Practice Test Flashcards

1
Q

Which of the following are the most important considerations when drafting IEP goals?

A Making sure the student has the same goal from year to year to promote consistency.

B Trying to write similar goals for groups of students who are in the same classes so it will be easier for the general education teachers to assess and report progress.

C Writing goals that will be easily attainable to build student confidence.

D Identifying data-supported weaknesses the student has displayed and aligning goals to curriculum standards that will allow the student to practice and improve on their deficiency at the appropriate rigor.

A

D

How well did you know this?
1
Not at all
2
3
4
5
Perfectly
1
Q

The Every Student Succeeds Act (ESSA) specifies that no more than 1 percent of all students should take alternate assessments not aligned to the academic standards of their grade level. For which of the following students would an alternate assessment be most appropriate?

A. a student with cerebral palsy who receives instruction in general-education classrooms but lacks the fine motor control to bubble in responses on an answer document

B. a student with autism spectrum disorder (ASD) who communicates verbally and receives instruction in general-education classrooms with paraprofessional support

C. a student with a severe intellectual disability who receives all instruction in a self-contained classroom

D. a student with an emotional disturbance (ED) who requires frequent breaks during tests and difficult assignments and constantly tries to engage other students in conversation

A

C

How well did you know this?
1
Not at all
2
3
4
5
Perfectly
2
Q

Which is the most appropriate section of a student’s IEP to include information about the student’s post-secondary goals as well as activities and services that the student can access now to prepare for post-secondary education and/or employment?

A. related services

B. least restrictive environment

C. accommodations and modifications

D. transition plan

A

D

How well did you know this?
1
Not at all
2
3
4
5
Perfectly
3
Q

Of the following, which parental concern is most likely to result in an evaluation of a child under the age of three?

A failure to reach national norms of height/weight ratio

B failure to recognize and identify letters of the alphabet

C failure to meet multiple milestones of adaptive development

D failure to complete toilet training

A

C

How well did you know this?
1
Not at all
2
3
4
5
Perfectly
4
Q

John is an incoming second-grade student with autism spectrum disorder (ASD) who is enrolled in a new district and school. John’s parents have expressed concerns about his transition to the new campus. Which of the following would be the best way to help John prepare for the new school year?

A. Schedule a phone call between John’s family and his new teacher so they can get to know each other.

B. a visual schedule of a typical school day for second-grade students so John can prepare for his new environment.

C. Mail John’s parents a map of the campus so they can review the layout and location of his classroom before school starts.

D. Offer to provide a private tour of the campus during which John and his parents can meet his teachers and the administrative staff.

A

D

How well did you know this?
1
Not at all
2
3
4
5
Perfectly
5
Q

Which of the following questions should take precedent when determining the least restrictive environment for any student during the initial referral process for special education services?

A Which instructional arrangement offers the greatest amount of support for students with disabilities?

B. Which instructional arrangement will provide the best opportunity for the student to demonstrate her strengths while receiving support for her weaknesses?

C. Which instructional arrangement will give the student the easiest route to annual promotion and eventual graduation?

D. Which instructional arrangement has the lowest student-to-teacher ratio and can therefore provide the greatest possible support?

A

B

How well did you know this?
1
Not at all
2
3
4
5
Perfectly
6
Q

Samuel is a third-grade student who is attentive and hardworking in class. He has good grades in math, science, and social studies, but struggles in reading. He has poor fluency, struggles to decode unfamiliar words, takes longer than other students to copy notes from the board, and struggles to answer questions that require a written response. Which of the following disabilities is Samuel most likely struggling with?

A. attention-deficit hyperactivity disorder (ADHD)

B. intellectual disability

C. dyslexia

D. autism spectrum disorder (ASD)

A

C

How well did you know this?
1
Not at all
2
3
4
5
Perfectly
7
Q

During a parent-teacher conference, the father of a non-disabled student in an inclusion class mentions that his child has had friction in the past with another student in the class who has an emotional disturbance. The father mentions several disagreements that the two have had and then asks if any progress has been made in the disabled student’s classroom behavior. How should the teacher respond?

A. Share relevant portions of the disabled student’s IEP to show that all necessary steps are being taken to provide a safe and comfortable educational environment for all.

B. Thank the father for his time and his concerns and firmly but politely indicate that you are only allowed to discuss his own child during the parent-teacher conference.

C. Assure the father that the disabled student in question will be kept away from his child to the maximum extent possible.

D. Remind the father that the student in question has a behavioral disability and ask that he encourage his own child to be more patient.

A

B

How well did you know this?
1
Not at all
2
3
4
5
Perfectly
8
Q

Lena is an eighth-grade student with ADHD. Which of the following would be the best accommodation to help Lena succeed?

Select all answers that apply.

A. having Lena take alternate tests with limited answer choices

B. allowing Lena to take breaks at specific stopping points during assignments

C. allowing Lena to work in a quiet area of the room

D. providing Lena with a variety of reference aids, such as a math fact sheet and formula sheet

A

B & C

How well did you know this?
1
Not at all
2
3
4
5
Perfectly
9
Q

Dominic is a two year-old boy who does not yet talk or make eye contact, hates loud noises, and prefers to play by himself, usually only with jungle animals. His teacher is concerned about his overall development and feels that he is not on the same level as his peers, especially socially. The next best step for Dominic’s teacher would be to:

The teacher should not presume anything and should keep the observations to herself. It is not her place to bring these concerns up if the parents haven’t said anything.

Discuss her observations and concerns with his parents and suggest next steps, such as a referral to an early intervention program.

Keep a record of her observations throughout the year and present the data to the parents at Dominic’s final conference.

Discuss the concerns with his parents and suggest that they try a new medical therapy that is supposed to help late talkers.

A

B

How well did you know this?
1
Not at all
2
3
4
5
Perfectly
10
Q

A sixth-grade social studies classroom is learning about the Stone Age and is going to be reading a text about hunters and gatherers and highlighting key ideas and details. Which of the following is a modification that might be appropriate for a boy in the class with a Specific Learning Disability (SLD) in reading who can read independently at a third-grade level?

A. The student completes a word search to familiarize himself with key vocabulary while the rest of the class completes the highlighting activity.

B. While the rest of the class works independently, the teacher sits with the student and reads the text to him, helping them understand the more complex sentences and identify key ideas and details to highlight.

C. The teacher allows the student to work in a different space to limit distractions and help the student focus on the activity.

D. The teacher creates a text that covers most of the same material but is written at a third-grade level to allow him to participate in the highlighting activity independently.

A

D The teacher creates a text that covers most of the same material but is written at a third-grade level to allow him to participate in the highlighting activity independently.

This modification is appropriate because it addresses the student’s specific learning disability in reading by providing a text that matches his reading proficiency level. By creating a text written at a third-grade level, the student can independently engage with the content about hunters and gatherers and participate in the highlighting activity alongside his classmates. This modification ensures that the student has access to the same material as his peers while accommodating his individual learning needs.

How well did you know this?
1
Not at all
2
3
4
5
Perfectly
11
Q

Pedro, a senior student with intellectual disabilities, will be graduating in the spring. He has been working on his money handling skills in his math classes. His goal is to work at his local grocery store. In order to facilitate this transition for the student, the special education teacher should:

A. Have Pedro complete a research project about what is required to work in a grocery store.

B. Assign Pedro to complete his resume and cover letter so that he is prepared to apply to all his local grocery stores upon graduation.

C. Collaborate with the local grocery store for Pedro to job train and shadow a couple hours a week while he is preparing for graduation.

D. Continue working with Pedro on his money handling skills to prepare him for the grocery job.

A

C Collaborate with the local grocery store for Pedro to job train and shadow a couple of hours a week while he is preparing for graduation.

This option directly addresses Pedro’s goal of working at a grocery store by providing him with practical, hands-on experience in a real-world setting. Job training and shadowing at the local grocery store will allow Pedro to familiarize himself with the tasks and responsibilities involved in working there, as well as develop essential skills required for the job. Additionally, this collaboration facilitates community involvement and creates opportunities for Pedro to build relationships with potential employers. It offers a meaningful transition from school to the workforce and supports Pedro’s goal of gaining employment after graduation.

How well did you know this?
1
Not at all
2
3
4
5
Perfectly
12
Q

Rachel, a high-school student who suffers from a traumatic brain injury, struggles with keeping track of her schedule and remembering her classes. Her reading skills were affected by the TBI, but she is able to understand everything when instructions are given orally. Give an example of a strategy that her special education teacher could use to help Rachel keep track of her schedule and remember her classes.

A. The teacher gives Rachel a large, numbered map, with highlighted classrooms, that she can follow throughout the day to get to each class

B. The teacher does not need to provide any strategies for Rachel, this is not an academic issue and her peers will be able to help her.

C. The only way for Rachel to gain back her reading skills is to practice, so her teacher decides to give her a written schedule to help her find her classes.

D. The teacher can provide Rachel with a visual schedule, paired with a recorded message, that both state the order of her classes.

A

D. The teacher can provide Rachel with a visual schedule, paired with a recorded message, that both state the order of her classes.

This strategy leverages Rachel’s strength in understanding instructions when given orally while also providing visual support to help her keep track of her schedule. The visual schedule offers a clear and accessible way for Rachel to see the order of her classes, while the recorded message reinforces the information orally, ensuring that she can comprehend and remember her schedule effectively. By combining both auditory and visual modalities, this strategy accommodates Rachel’s needs resulting from her traumatic brain injury and supports her in managing her schedule and remembering her classes.

How well did you know this?
1
Not at all
2
3
4
5
Perfectly
13
Q

What is the benefit of teaching expanded notation as an alternative method of calculating two- or three-digit addition problems?

A. Expanded notation addition requires extra steps, thus forcing students to concentrate more when performing calculations.

B. Expanded notation helps students understand decimals and fractions with greater clarity.

C. Expanded notation requires a calculator to complete, so students develop a useful technological skill at an earlier age.

D. Expanded notation helps students clearly delineate between place values and understand the value that each digit in a large number actually represents.

A

D

How well did you know this?
1
Not at all
2
3
4
5
Perfectly
14
Q

Which of the following situations would be the most logical instance to provide calculation aids to a student in a sixth-grade math inclusion class?

A. The student has a specific learning disability (SLD) in written expression and is struggling to maintain adequate organization when setting up and solving multi-step problems.

B. The student has a specific learning disability (SLD) in reading fluency and is struggling to identify key information in word problems.

C. The student has attention-deficit/hyperactivity disorder (ADHD) and struggles to maintain focus when completing multi-step problems.

D. The student has a specific learning disability (SLD) in math calculation and is struggling to complete multi-step problems.

A

D

How well did you know this?
1
Not at all
2
3
4
5
Perfectly
15
Q

Which of the following situations would not be beneficial for classroom management in a seventh-grade co-taught science classroom?

A. A teacher uses a chime to indicate when it is time to transition between stations during a lab.

B. A teacher allows the class to go outside because they have worked hard that day.

C. A teacher allows students to play a game at the start of class to incentivize hard work for the rest of the period.

D. The two teachers work together to model or “role play” appropriate and safe classroom behavior.

A

C, allowing students to play a game at the start of class to incentivize hard work for the rest of the period, might not be as beneficial for classroom management in a seventh-grade co-taught science classroom. While incentivizing hard work is a positive approach, starting the class with a game might lead to difficulty in transitioning from the game to the academic tasks, potentially causing disruptions and loss of focus. The other options all involve structured methods for managing classroom transitions, behavior modeling, or rewards that are more directly tied to the academic tasks at hand.

How well did you know this?
1
Not at all
2
3
4
5
Perfectly
15
Q

At the beginning of the year, during open house, a preschool teacher always welcomes her class by reading aloud one of her favorite books. She holds the book up for everyone to see the words and pictures, and she uses her finger to move along the text as she reads. She then encourages parents to also track their progress through books at home by tracing the line of text as they read. Which of the following is the teacher’s reason for this suggestion?

A. to encourage student structural analysis of words

B. to improve student recognition of sight words

C. to teach all print concepts

D. to help students develop an understanding of print orientation

A

D. to help students develop an understanding of print orientation

The teacher’s reason for suggesting that parents trace the line of text as they read at home is most likely to help students develop an understanding of print orientation. This practice supports children in recognizing that print runs from left to right and top to bottom, which is a fundamental print concept.

How well did you know this?
1
Not at all
2
3
4
5
Perfectly
16
Q

Which of the following best describes the degree to which an assessment accurately measures what it intends to measure?

A. validity

B. reliability

C. predictability

D. agility

A

A. Validity

How well did you know this?
1
Not at all
2
3
4
5
Perfectly
17
Q

When proposing an evaluation for initial eligibility, the Individuals with Disabilities Education Improvement Act requires that:

A. the evaluation is administered by qualified professionals

B. the same evaluations are administered to all students

C. parents provide verbal consent to the evaluation

D. all students are given speech/language evaluations

A

A. the evaluation is administered by qualified professionals

How well did you know this?
1
Not at all
2
3
4
5
Perfectly
18
Q

A high school special education teacher is preparing for an upcoming ARD meeting in which they will discuss a transition plan for a 16-year-old student. When considering this upcoming ARD committee meeting and IEP, which of the following is not required?

A. The IEP must include some form of post-secondary education.

B. The student must be invited to participate in the planning.

C. The IEP must address the student’s life after high school.

D. The IEP must include measurable goals.

A

A. The IEP must include some form of post-secondary education

How well did you know this?
1
Not at all
2
3
4
5
Perfectly
19
Q

Oppositional defiant disorder, or ODD, is a behavioral disorder that is NOT characterized by which of the following behaviors?

A. vindictive behavior, seeking revenge, and harsh words

B. focused and highly intense interests

C. constant blame on others for own mistakes and misbehavior

D. frequent temper tantrums and refusal to complete tasks

A

B. Focused and highly intense interests

How well did you know this?
1
Not at all
2
3
4
5
Perfectly
20
Q

A child with severe autism is determined by the Admission, Review, and Dismissal (ARD) committee to require a one-to-one aid, but the school district insists they cannot afford to hire someone for this role. As a result, the student does not make significant progress toward their IEP goals. The parents then sue the school on the grounds that:

A. the school is failing to provide Free Appropriate Public Education (FAPE) for their child.

B. the school is violating the guideline of “separate but equal” for their child.

C. the school is violating the guideline of “least restrictive environment” for their child.

D. the school is violating the No Child Left Behind act.

A

A. the school is failing to provide Free Appropriate Public Education (FAPE) for their child.

How well did you know this?
1
Not at all
2
3
4
5
Perfectly
21
Q

Which of the following best characterizes a student with an intellectual disability?

A. a student with more than one impairment, that so significantly impacts their access to the curriculum their educational needs cannot be met under one category of special education services

B. a student with notably below sub-average intelligence, presenting during the developmental period, and with adaptive delays

C. a student with communication delays or impairments, including, but not limited to, language or voice impairment, stuttering, and articulation issues

D. a student with a developmental disability, due to delays in communication and social interaction, often with perseverated, or very focused, interests, sensitivity to change in routine, and intense sensory response

A

B. a student with notably below sub-average intelligence, presenting during the developmental period, and with adaptive delays.

This description encapsulates the key aspects of intellectual disability, including cognitive functioning significantly below average, onset during the developmental period, and challenges with adaptive behavior.

How well did you know this?
1
Not at all
2
3
4
5
Perfectly
22
Q

Which of the following statements about Extended School Year (ESY) services is accurate according to the Individuals with Disabilities Education Act (IDEA)?

A. Students who are eligible for ESY must receive ESY for all of the goals on their IEPs.

B. Eligibility for ESY is determined annually.

C. Students who were found eligible for ESY must receive ESY in subsequent years.

D. ESY may only be provided to students with multiple disabilities.

A

B. Eligibility for ESY is determined annually

How well did you know this?
1
Not at all
2
3
4
5
Perfectly
23
Q

If a student with disabilities commits a crime that is a violation of federal and state law, the Local Education Agency (LEA) must provide the special education and discipline records for consideration to the maximum extent permitted by which of the following?

A. the committee meeting minutes

B. the IEP

C. Family Educational Rights and Privacy Act (FERPA)

D. the Local Education Agency (LEA) student code of conduct

A

C

How well did you know this?
1
Not at all
2
3
4
5
Perfectly
24
Q

A teacher wishes to check student comprehension of a concept taught in today’s math class. What would be the best type of assessment for him to use?

A. formative

B. norm-referenced

C. diagnostic

D. summative

A

A. Formative

How well did you know this?
1
Not at all
2
3
4
5
Perfectly
25
Q

Maria is concerned that when she brings her 2-and-a-half-year-old daughter to the park, her daughter is often very rough with the other children, does not respect their boundaries, and often struggles to end her playtime outside. The behavior has become extreme, and Maria needs help. Maria talks with her local early intervention coordinator who suggests that they refer her daughter for evaluation. The coordinator suggests that they place extra focus on what area of development?

A. adaptive development

B. social-emotional development

C. motor development

D. cognitive development

A

B. Social-Emotional development

How well did you know this?
1
Not at all
2
3
4
5
Perfectly
26
Q

As a special education teacher, Mr. Platz wants to use an assessment that accurately reflects his students’ vast diversity of skill and knowledge, and that uses multiple forms of data, student work, and learning material to measure progress. The best assessment style to recommend to Mr. Platz would be:

A. portfolio-based assessments.

B. standardized assessments.

C. observational assessments.

D. benchmark assessments.

A

A. portfolio-based assessments

How well did you know this?
1
Not at all
2
3
4
5
Perfectly
27
Q

A student with ADHD struggles to pay attention for long assessments such as state-wide standardized tests. Which of the following is a likely testing accommodation that might appear on that student’s IEP?

A. The student will take the test in a small group rather than in the general education classroom and may take movement breaks between sections.

B. The student will take the test in their general education classroom but may take movement breaks between sections.

C. The student takes a shorter version of the test than her peers due to her disability.

D. The teacher may provide specific help such as defining key words or helping identify the operations required in difficult word problems.

A

A. The student will take the test in a small group rather than in the general education classroom and may take movement breaks between sections.

How well did you know this?
1
Not at all
2
3
4
5
Perfectly
28
Q

A student with a sensory processing disorder has extreme sensitivity to loud noises and is easily distracted by environmental sounds. To best support this student, the teacher can:

A. play quiet background music at all times.

B. allow the student to wear noise-canceling headphones whenever needed.

C. have the student sit at a desk away from other students so that they aren’t distracted.

D. make the classroom a whisper-only zone and not allow any students to talk at a high volume.

A

B. Allow the student to wear noise-canceling headphones whenever needed.

How well did you know this?
1
Not at all
2
3
4
5
Perfectly
29
Q

Which of the following was NOT part of the Individuals with Disabilities Education Act (IDEA)?

A. the requirement that parents and teachers be involved in IEP decisions

B. the least restrictive environment (LRE) guideline

C. the establishment of IEPs as a federal regulation

D. the establishment of mandated state testing for special education students

A

D. The establishment of mandated state testing for special education students

How well did you know this?
1
Not at all
2
3
4
5
Perfectly
30
Q

In an IEP meeting, which of the following would likely be provided by the parents of the student?

A. the behavior plan that should be followed

B. a list of required accommodations for testing

C. the student’s grades from their previous school

D. any relevant medical information including contact information for doctors

A

D. any relevant medical information including contact information for doctors

How well did you know this?
1
Not at all
2
3
4
5
Perfectly
31
Q

Teresa is a special education student in third grade. She is a very talented artist and conversationalist but doesn’t like to let her peers know she struggles with a specific learning disability related to reading. She will often be disruptive and chatty during group projects and work to hide this fact, so she does not have to contribute. What strategy could the teacher use to integrate Teresa into her group so that she is able to do the work being asked of her and not distract her peers?

A. The teacher could privately talk with Teresa about behavior expectations during learning time. Together they can form a plan for future projects where Teresa can contribute in meaningful ways that she feels confident with, such as through art or some other form of creative expression.

B. The teacher can compliment the students who are on task in Teresa’s learning group, hoping to encourage Teresa to get back on task.

C. Give Teresa the ultimatum that if she can’t keep from distracting her peers during group work, then she will have to work alone.

D. Reprimand Teresa in front of her peers every time she becomes off task so that she learns to work in a group and not distract her peers.

A

A. The teacher could privately talk with Teresa about behavior expectations during learning time. Together they can form a plan for future projects where Teresa can contribute in meaningful ways that she feels confident with, such as through art or some other form of creative expression.

How well did you know this?
1
Not at all
2
3
4
5
Perfectly
32
Q

Juan is 5 feet tall and casts a shadow that is 10 feet long. If the flagpole casts a shadow that is 30 feet long, how tall is the flagpole?

A. 30 feet

B. 5 feet

C. 10 feet

D. 15 feet

A

D. 15 feet

How well did you know this?
1
Not at all
2
3
4
5
Perfectly
33
Q

A special education teacher is working with the language arts teacher to promote her students’ understanding of media literacy and advertising techniques. What activity would be most appropriate to use to achieve this learning objective?

A. reading a nonfiction article about the history of commercial advertising

B. viewing commercials and having discussions about the advertising techniques used

C. working with a partner to match advertisements to the correct products

D. creating a new product and then advertising this invention

A

B. Viewing commercials and having discussions about the advertising techniques used

How well did you know this?
1
Not at all
2
3
4
5
Perfectly
34
Q

Kenny is a kindergarten student with an IEP relating to executive functioning and sensory processing issues. His teacher has created a First/Then task chart for him to use during their learning center time to help with expectations, as visual schedules are too overwhelming. The main benefit of using a First/Then task chart for Kenny is:

A. it helps him learn to manage his time and task compliance.

B. it promotes Kenny’s independence and cognitive development.

C. it lets Kenny choose whatever two activities he wants to complete, giving him control over the task.

D. it helps Kenny to understand what is expected of him, focusing on two tasks at hand without being overwhelming.

A

D. It helps Kenny to understand what is expected of him, focusing on two tasks at hand without being overwhelming.

How well did you know this?
1
Not at all
2
3
4
5
Perfectly
35
Q

A fourth-grade student with attention-deficit/hyperactivity disorder (ADHD) has demonstrated the ability to accurately perform all four primary operations (addition, subtraction, multiplication, and division) up to three-digit numbers when given problems to complete on simple worksheets, but she struggles to correctly perform these same calculations when faced with word problems. Which of the following strategies is most likely to benefit the student when solving word problems?

A. Reduce the difficulty of the word problems by editing them to allow her to work with smaller numbers.

B. Emphasize key information and trigger words to help the student determine which operation to use, and teach her to identify the integers involved and eliminate unnecessary or distracting information before performing the calculations.

C. Accommodate the assignment to provide more space on the page between problems, so the student will have more room for her calculations.

D. Provide a calculator.

A

B. Emphasize key information and trigger words to help the student determine which operation to use, and teach her to identify the integers involved and eliminate unnecessary or distracting information before performing the calculations.

How well did you know this?
1
Not at all
2
3
4
5
Perfectly
36
Q

All of the following should be done for someone having a seizure except:

A. put something soft and flat under the person’s head.

B. try to reduce movement by gently holding the person down.

C. turn the person gently onto one side.

D. remove any eyeglasses from the person’s face.

A

B. Try to reduce movement by gently holding the person down

How well did you know this?
1
Not at all
2
3
4
5
Perfectly
37
Q

Which of the following would be an appropriate intervention to help fifth-grade students with deficits in reading identify the main idea in an informational text that is appropriate for their reading level?

A. Pre-teach important vocabulary and provide a word wall in the room to help students review and comprehend the new vocabulary words.

B. Provide a story structure graphic organizer that helps them think of the story as a hill with the climax at the top and the resolution at the bottom.

C. Provide a highlighter and ask students to highlight key facts so that they can easily find them later when they are taking notes.

D. Provide a “box and bullet” graphic organizer that helps them break down the article by listing important facts before trying to identify the main idea.

A

D

How well did you know this?
1
Not at all
2
3
4
5
Perfectly
38
Q

For students in fifth grade, which of the following literacy skills is likely the focus of reading instruction?

A. fluency

B. comprehension

C. literary analysis

D. decoding

A

B. comprehension.

At this stage, students are expected to comprehend more complex texts, infer meaning, make connections, and analyze information. While fluency, literary analysis, and decoding are also important, they often build upon a foundation of strong comprehension skills.

How well did you know this?
1
Not at all
2
3
4
5
Perfectly
38
Q

Which of the following would be an appropriate intervention to help fifth-grade students with deficits in reading identify the main idea in an informational text that is appropriate for their reading level?

A. Pre-teach important vocabulary and provide a word wall in the room to help students review and comprehend the new vocabulary words.

B. Provide a story structure graphic organizer that helps them think of the story as a hill with the climax at the top and the resolution at the bottom.

C. Provide a highlighter and ask students to highlight key facts so that they can easily find them later when they are taking notes.

D. Provide a “box and bullet” graphic organizer that helps them break down the article by listing important facts before trying to identify the main idea.

A

D. Provide a “box and bullet” graphic organizer that helps them break down the article by listing important facts before trying to identify the main idea.

This intervention helps students organize key information from the text, making it easier for them to analyze and determine the main idea. By identifying important facts first, students can then see patterns or recurring themes that contribute to understanding the main idea.

How well did you know this?
1
Not at all
2
3
4
5
Perfectly
39
Q

Ms. Chen, a third-grade special education teacher, notices that one of her students with ADHD seems to be struggling more than usual in the classroom lately. He seems dysregulated and has been having more tearful moments of frustration during independent work time. Which of the following options would be the best course of action for Ms. Chen?

A. Contact the school nurse and ask if there has been any change in the student’s medication lately.

B. Implement a behavior plan to encourage positive behavior and get the student back on track.

C. Contact the parents to discuss the behavior and ask for their input or advice.

D. Contact the principal and have her check in with the student about his behavior.

A

C. Contact the parents to discuss the behavior and ask for their input or advice.

How well did you know this?
1
Not at all
2
3
4
5
Perfectly
40
Q

A student with Cerebral Palsy is moving from his elementary school to the district’s middle school. In his elementary school, he was placed in a general education classroom with one-on-one paraprofessional support for some of the day and spent two hours each day receiving OT (occupational therapy) and PT (physical therapy) services outside of the classroom. Which is most likely to be true of his new placement?

A. He will likely be placed in a resource setting as the academic demands will become too rigorous for him, and he will need to focus more on OT and PT goals.

B. He will likely be placed in a resource setting as middle and high schools tend to employ fewer paraprofessionals and a self-contained placement will allow him to have the academic supports he needs.

C. He will likely continue to be placed in a general education setting for the majority of the day and will spend less time with the occupational and physical therapists as OT and PT supports are limited outside of an elementary school setting.

D. He will likely continue to be placed in a general education setting for some of the day and continue to receive OT and PT support as appropriate.

A

D. He will likely continue to be placed in a general education setting for some of the day and continue to receive OT and PT support as appropriate.

How well did you know this?
1
Not at all
2
3
4
5
Perfectly
41
Q

A student who uses screen magnification software most likely has a primary classification of:

A. hearing impairment.

B. intellectual disability.

C. specific learning disability.

D. vision impairment.

A

D. vision impairment.

How well did you know this?
1
Not at all
2
3
4
5
Perfectly
42
Q

Lizzie is a high-school student who recently acquired a Traumatic Brain Injury due to a horseback riding incident. One of the lingering issues that Lizzie is dealing with is uncontrolled seizures. It will be important for Lizzie’s teachers to understand which TWO of the following, when it comes to seizure management?

Select all answers that apply.

A. Lizzie’s seizures impact her mental health

B. possible seizure triggers for Lizzie

C. what Lizzie’s seizures look like and how long they last

D. Lizzie’s medication schedule and when the teacher will need to provide her medicine to her

A

B & C

How well did you know this?
1
Not at all
2
3
4
5
Perfectly
43
Q

Regarding FERPA and the purpose of student records, personal identifiable information refers to all of the following except:

A. the mother’s maiden name.

B. the parents’ names and family address.

C. the student’s interests and hobbies.

D. the student’s social security number.

A

C. the student’s interests and hobbies

How well did you know this?
1
Not at all
2
3
4
5
Perfectly
44
Q

A sixth-grade teacher is looking for a way to improve her students’ understanding of how visual images create meaning. Which activity below would be the best way to achieve this goal?

Have students work in pairs to discuss what they like and don’t like about presented images.

Have students tell a new story using only photographs from a magazine or newspaper.

Have students view famous paintings online and write a description of the painting’s meaning.

Have students draw an image to represent a section of a pre-existing short story after listening to the story in class.

A

B. Have students tell a new story using only photographs from a magazine or newspaper

How well did you know this?
1
Not at all
2
3
4
5
Perfectly
45
Q

Mel is an elementary student diagnosed with autism spectrum disorder. In order for her to be most successful in the classroom setting, her general education teacher can support her by:

A. keeping consistent routines and expectations that are clearly stated and posted in the classroom.

B. pairing Mel with a peer to support her and keep her on track.

C. sending her to the learning support teacher’s classroom.

D. having Mel check in with her teacher at the end of every day, after she has everything ready to go home.

A

A. Keeping consistent routines and expectations that are clearly stated and posted in the classroom

How well did you know this?
1
Not at all
2
3
4
5
Perfectly
46
Q

Dylan, a high-school senior diagnosed with severe ADHD, loves fashion. She plans to work at a local boutique when she graduates. Her teacher helps to prepare her for this role, pairing with a local shop owner to give Dylan real-world experience and to create a smooth transition for Dylan after graduation. The teacher practices other skills with Dylan in school, such as customer service, self-advocacy, and clear communication. By pairing with the shop owner and helping Dylan to develop the skills to function independently and confidently, the teacher is showing that she understands which best practice?

A. The importance of only focusing on and teaching students the skills that they will need to join the workforce after graduation.

B. The importance of teaching different skills across different learning environments and how these factors can impact student learning and behavior, as well as increase independence and competence.

C. The shop owner expects employees who are competent and highly skilled. The teacher knows this and is preparing Dylan for the role.

D. The importance of developing young students to join the workforce and be ready to participate in society.

A

B. The importance of teaching different skills across different learning environments and how these factors can impact student learning and behavior, as well as increase independence and competence.

How well did you know this?
1
Not at all
2
3
4
5
Perfectly
47
Q

Liam is a second-grade student with an intellectual disability. He spends approximately 25% of his day in a general education classroom. Liam is able to count ten, identify a square and a circle, and point to the larger set when shown two sets of counters up to five. Liam’s general education teacher is currently planning lessons on comparing three-digit numbers.

Which of the following modifications would be most appropriate for Liam?

A. allowing Liam extended time to complete assignments and tests

B. providing Liam with alternative assignments that cover the comparison of one-digit numbers using pictures

C. providing Liam with alternative assignments that cover counting to twenty

D. giving Liam the same assignments as other students, but administering alternative summative assessments

A

B. providing Liam with alternative assignments that cover the comparison of one-digit numbers using pictures

How well did you know this?
1
Not at all
2
3
4
5
Perfectly
48
Q

In developing a behavior intervention plan (BIP), it is essential that each identified behavior includes baseline data related to frequency, duration, intensity, and latency. This data will enable professionals implementing the BIP to

A. monitor progress in comparison to other students

B. measure the effectiveness of interventions

C. provide consequences

D. alter antecedent events

A

B. measure the effectiveness of interventions

How well did you know this?
1
Not at all
2
3
4
5
Perfectly
49
Q

Eleanor has recently been identified with a specific learning disability (SLD) in reading after struggling but receiving passing grades in a general education setting for several years. Testing reveals that she is reading three grade levels below benchmark and struggles with comprehension. Based on this recent diagnosis, how should her services for social studies, which has a heavy reading and writing component, change to better suit her needs?

A. In this case, the least restrictive environment guideline indicates that Eleanor should stay in her general education class and complete assignments to the best of her ability as she was not failing prior to her diagnosis.

B. If possible, Eleanor should be placed in a co-taught class or attend a general education class with assignments that are modified to be appropriate for her level.

C. Eleanor should be able to decide where she would like to be placed for Social Studies.

D. Eleanor should be moved to a self-contained or resource setting so that she will not fall further behind her peers.

A

B. If possible, Eleanor should be placed in a co-taught class or attend a general education class with assignments that are modified to be appropriate for her level.

This option provides Eleanor with additional support in a general education setting while ensuring that the assignments are adjusted to accommodate her reading difficulties. It aims to provide her with the necessary support to access the curriculum while still being included in the general education environment to the greatest extent possible.

How well did you know this?
1
Not at all
2
3
4
5
Perfectly
50
Q

What is the main advantage of joining a professional organization related to special education?

A. meeting new colleagues

B. reduced number of required hours for continuing education

C. advancing knowledge with opportunities for collaboration

D. opportunities to travel for conferences

A

C. Advancing knowledge with opportunities for collaboration

How well did you know this?
1
Not at all
2
3
4
5
Perfectly
51
Q

Lucy is a special education teacher in an elementary school. She is walking down the hallway and is stopped by Sam, a general education third-grade teacher who is wondering if Lucy could stop by the classroom at some point that week to informally observe a student who the teacher believes may have an unidentified disability. What would be the most appropriate response for Lucy in this situation?

A. Ask for more details during the conversation so that she can be more prepared when she visits the room.

B. Ask Sam to follow up with an email and remind them that they should not be discussing issues in the hallway as it is a FERPA violation.

C. Politely decline as special education teachers should only be asked to assist students on their caseload.

D. Offer to visit at a time when the student will be engaged in an activity that is likely to illustrate the issue and go from there.

A

D. Offer to visit at a time when the student will be engaged in an activity that is likely to illustrate the issue and go from there.

How well did you know this?
1
Not at all
2
3
4
5
Perfectly
52
Q

Which of the following is the most advanced language arts skill?

A. analyzing instances of figurative language

B. classifying types of figurative language

C. identifying examples of figurative language

D. defining types of figurative language

A

A. analyzing instances of figurative language

How well did you know this?
1
Not at all
2
3
4
5
Perfectly
53
Q

Aman is a third-grade student who gets pulled out of his general education classroom to receive 30 minutes of special education instruction in reading each day. He is independently reading at a first-grade level and often stops reading when he gets to longer or unfamiliar words. Which of the following would be the most appropriate activity for the special education teacher to do with Aman during his pull-out time?

A. Select an interesting book and do a picture walk. Then predict what might happen in the book based on the title and the pictures.

B. Read second-grade level books together to review and practice strategies like “chunking” to help Aman improve his decoding skills.

C. Help Aman improve his fluency by having him read 3rd-grade level texts aloud, offering help when needed.

D. Play a game using Scrabble tiles that requires Aman to name each letter and make its sound.

A

B. Read second-grade level books together to review and practice strategies like “chunking” to help Aman improve his decoding skills.

How well did you know this?
1
Not at all
2
3
4
5
Perfectly
54
Q

A student’s parents have requested testing for their child as they suspect that she might qualify for special education services. The district determined that testing was appropriate and the results have just been finalized. Which of the following scenarios would be the most appropriate next step?

A. The principal calls to schedule a meeting with the parents to go over the results of the testing.

B. The special education teacher sends the results to the parents as a PDF via email or in a paper packet that is mailed to their home.

C. The special education teacher calls the parents to let them know the testing has been completed and schedules an eligibility meeting to go over the results and discuss next steps.

D. The student’s general education teacher is given the results and calls the parents to let them know whether or not their child qualifies for services based on the results.

A

C. The special education teacher calls the parents to let them know the testing has been completed and schedules an eligibility meeting to go over the results and discuss next steps.

How well did you know this?
1
Not at all
2
3
4
5
Perfectly
55
Q

Penelope’s parents meet with the ARD committee to review her initial evaluation results and discuss what happens next. They are nervous about the outcome and unsure of how to move forward based on the assessment and evaluation information. The best way for the special education teacher to support Penelope’s parents would be to:

A. explain the results of the assessments and evaluations in a thorough way that her parents understand, and then present the next steps for the parents to consider based on the information they have about Penelope.

B. take no action. The parents were told the assessment and evaluation results over the phone and they don’t need to be discussed or clarified at the meeting.

C. expect the parents to do their own research and voice what they want for Penelope, but help if they ask for it.

D. explain the results and explain that Penelope will be enrolled in a developmental preschool classroom.

A

A. explain the results of the assessments and evaluations in a thorough way that her parents understand, and then present the next steps for the parents to consider based on the information they have about Penelope.

How well did you know this?
1
Not at all
2
3
4
5
Perfectly
56
Q

An inclusion teacher is planning support for the special education students in a general education classroom for an upcoming writing assignment. During the writing process, which of the following best demonstrate the most appropriate way to support these students?

A. After writing the first draft, require the students to transfer the content of their essay into a graphic organizer to determine the best organizational structure for their writing.

B. Increase the amount of class time allotted for each stage.

C. Allow students to skip the revision and editing stage knowing that they will not be graded on spelling and grammar errors.

D. Provide students with a checklist that includes each stage in the writing process and steps to take during each stage. Stop after each stage to discuss the next stage and prompt students to take steps to engage in the next stage of writing.

A

D. Provide students with a checklist that includes each stage in the writing process and steps to take during each stage. Stop after each stage to discuss the next stage and prompt students to take steps to engage in the next stage of writing.

How well did you know this?
1
Not at all
2
3
4
5
Perfectly
57
Q

Emily, a general education social studies teacher, and Thomas, a special education teacher, are teaching a co-taught seventh-grade social studies class. They are designing a research project to teach and assess students’ inquiry skills which will ask students to select a current event to investigate from a list of options. Which of the following would NOT be an appropriate method of differentiation to include in this project?

A. providing a menu of different types of final product they could create

B. assigning topics to some students based on difficulty level

C. creating graphic organizers that are available to help students take notes as they conduct research

D. allowing students to use videos instead of written texts as resources

A

B. assigning topics to some students based on difficulty level

How well did you know this?
1
Not at all
2
3
4
5
Perfectly
58
Q

Which of the following would not be appropriate for teaching math to students with emotional disturbance?

A. creating opportunities for real-world applications of skills

B. building in rewards for positive behavior

C. modifying the pacing of units

D. involving hands-on activities

A

C. modifying the pacing of units.

Modifying the pacing of units might not be the best approach because it could inadvertently single out students or lead to feelings of inadequacy if they perceive themselves as falling behind their peers. It’s essential to provide appropriate support and scaffolding without compromising the rigor of the curriculum or making students feel isolated. Therefore, modifying pacing should be done discreetly and with consideration for each student’s needs. The other options (A, B, and D) are generally effective strategies for teaching math to students with emotional disturbance, as they promote engagement, positive reinforcement, and hands-on learning experiences.

How well did you know this?
1
Not at all
2
3
4
5
Perfectly
59
Q

A preschool aged child points to the sign that says “Target” and identifies the letters in the word, but tells her mom that the sign says “store.” What does this demonstrate about the child?

A. The child lacks and understanding of print orientation.

B. The child lacks alphabetic knowledge.

C. The child lacks phoneme awareness.

D. The child has developed print awareness.

A

D. The child has developed print awareness

This scenario demonstrates that the child has developed print awareness. Print awareness refers to the understanding that print carries meaning and is used for various purposes in our environment, such as labeling, informing, and communicating. The child correctly identifies the letters in the word “Target” and associates it with the concept of a store, indicating an understanding of the connection between printed words and their meanings, even if the exact word is not accurately decoded.

How well did you know this?
1
Not at all
2
3
4
5
Perfectly
60
Q

Which of the following is an accurate statement about the use of social skills for students with disabilities?

A. Social skills instruction is only appropriate for students with autism.

B. Social skills instruction is only provided to younger students.

C. Social skills instruction is appropriate for improving students’ interactions with peers and adults.

D. Social skills instruction must be provided by a speech/language therapist or a social worker.

A

C. Social skills instruction is appropriate for improving students’ interactions with peers and adults

How well did you know this?
1
Not at all
2
3
4
5
Perfectly
61
Q

Which of the following describes the outcome and relevance of the Supreme Court case Board of Education of the Hendrick Hudson Central School District v. Rowley (Hudson v. Rowley)?

A. The Court ruled that schools may not segregate students and overturned the previous standard of “separate but equal.” This ruling determined that racial segregation is unconstitutional.

B. The Court ruled that, because the student was reasonably successful in her classes, the school had provided a Free Appropriate Public Education (FAPE) and did not need to provide the additional services the parents were requesting. This was the court’s first ruling that defined FAPE.

C. The Court ruled that a school may not expel or indefinitely suspend a handicapped student for behavior related to their disability even if it endangers the student or others. This was a challenge of the “stay-put” provision of the Education of the Handicapped Act.

D. The Court ruled that, even if a student is making minimal progress toward IEP goals, schools must provide services that allow the student to make progress that is appropriate for their circumstances. This ruling established a guideline for how much progress is necessary in order for a FAPE to have been provided.

A

B. The Court ruled that, because the student was reasonably successful in her classes, the school had provided a Free Appropriate Public Education (FAPE) and did not need to provide the additional services the parents were requesting. This was the court’s first ruling that defined FAPE.

This ruling established a precedent for defining what constitutes a Free Appropriate Public Education (FAPE) under the Individuals with Disabilities Education Act (IDEA). It clarified that FAPE does not require schools to maximize a student’s potential but rather to provide an education that is reasonably calculated to confer educational benefit.

How well did you know this?
1
Not at all
2
3
4
5
Perfectly
62
Q

Sasha is an eighth-grade student with an intellectual disability who learns in Mr. Abanza’s resource room. During a creative writing unit, Sasha is struggling to understand the concept of narrative structure. She often says she cannot think of anything to write about and when she does write, her “stories’’ are often one to two sentences long. For example, she may write “one day a girl got a pony. She loved the pony. The End.”

Which of the following interventions should Mr. Abanza try to help Sasha understand how to write a creative story?

A. reading a mentor text aloud and allowing her to use the story she heard as inspiration

B. allowing Sasha to use pictures to tell a story instead of requiring her to write

C. providing Sasha with sentence frames to help her structure her sentences

D. allowing Sasha to use speech-to-text technology to help her write more

A

A. reading a mentor text aloud and allowing her to use the story she heard as inspiration.

By exposing Sasha to well-written narratives, she can gain a better understanding of how stories are structured and how characters, settings, and events are developed. Allowing her to use the story she heard as inspiration can provide a scaffold for her own writing and help spark her creativity. Additionally, discussing the elements of the mentor text and how they contribute to the story can further support Sasha in developing her narrative skills.

How well did you know this?
1
Not at all
2
3
4
5
Perfectly
63
Q

A special education teacher is communicating with a high-need parent of one of her students. The parent is frustrated that the teacher sometimes does not respond to emails sent later in the day until the following school day and feels that her concerns are not being respected. The parent requests the teacher’s cell phone number so that she can text for daily updates on her child’s behavior during each day. Which of the following options would be the most appropriate course of action for the teacher?

A. Provide the parent with the cell phone number but remind her that texts will not be answered outside of business hours.

B. Tell the parent that it is inappropriate to text but agree to respond more quickly to emails sent later in the day.

C. Acknowledge the parent’s concerns and bring in an administrator to help solve the problem.

D. Suggest an in-person meeting to address the parent’s concerns.

A

C. Acknowledge the parent’s concerns and bring in an administrator to help solve the problem.

How well did you know this?
1
Not at all
2
3
4
5
Perfectly
64
Q

Which of the following assessment methods would be the most appropriate way for a teacher to quickly gauge understanding toward the end of a mini-lesson?

A. asking students to hold up their thumbs (up, down, or sideways) to rate how well they understand what was just taught

B. asking students if they have any questions following the lesson

C. monitoring student participation during a review game

D. asking students to quickly complete an exit slip

A

A. asking students to hold up their thumbs (up, down, or sideways) to rate how well they understand what was just taught

This method provides immediate feedback from all students, allowing the teacher to quickly assess overall comprehension. It also allows students to anonymously indicate their level of understanding, reducing potential social pressure. While the other options (B, C, and D) can also provide valuable information about student understanding, the thumb rating method is particularly efficient and straightforward for a quick check of understanding.

How well did you know this?
1
Not at all
2
3
4
5
Perfectly
65
Q

The LEA has the responsibility to identify, locate, and evaluate all students who reside in the state of Texas who are in need of special education or related services, including students enrolling in private schools. At what age can children begin receiving services?

A. Birth

B. Once they begin speaking

C. 5 years old or enrolled in elementary school

D. 3 years old or enrolled in preschool

A

A. Birth

How well did you know this?
1
Not at all
2
3
4
5
Perfectly
66
Q

A special education teacher has incorporated time in their 7th-grade resource class to develop the student’s study skill using materials from their core class. The teacher notices that the students need assistance comprehending non-literary texts. Which of the following strategies will support these students’ ability to understand and more easily interact with these texts?

A. mark any turning points in the text

B. identify the type of narrator in the text

C. highlight and identify the types literary devices included

D. pre-read text features and scan for organizational patterns

A

D. pre-read text features and scan for organizational patterns

How well did you know this?
1
Not at all
2
3
4
5
Perfectly
67
Q

Based on a recent Ages and Stages Questionnaire, 1-year-old Penny’s doctor is concerned about her cognitive and motor development. The doctor brings the concerns to Penny’s mom, who has also noticed these concerns and delays. Penny’s mom is unsure of what to do next. The doctor suggests:

A. Telling mom to wait it out. She feels that Penny is behind, but she should grow out of it.

B. Looking up strategies online and trying them at home. If Penny’s mom doesn’t see improvement, she can come back in six months.

C. Contacting the state early intervention program for children from birth to age three that provides free developmental evaluations and services to families.

D. Referring Penny to a new doctor who is more comfortable seeing children with cognitive and motor delays.

A

C. Contacting the state early intervention program for children from birth to age three that provides free developmental evaluations and services to families.

How well did you know this?
1
Not at all
2
3
4
5
Perfectly
68
Q

A special education teacher is teaching students to improve their understanding of texts by scanning the text to preview text features and graphic elements. Which of the following genres is best served by this reading comprehension strategy?

A. a personal journal entry

B. a persuasive letter

C. a historical document

D. an informational text

A

D. an informational text

How well did you know this?
1
Not at all
2
3
4
5
Perfectly
69
Q

Mary is a student in Ms. Jones’s special education math class. She has been asked to complete an assignment that is at her level but that requires prolonged mental effort. The teacher notices that she is near the end of the assignment but hasn’t made progress in several minutes and is instead staring out the window. Ms. Jones knows Mary can finish if she can just keep at it. Which of the following comments would be the most appropriate and beneficial way to help motivate Mary to finish the assignment?

A. “You’re so close to the end! Why don’t you go get a quick drink from your water bottle and come back and finish this up? I bet it will feel so good to finish this in class so it won’t be homework tonight!”

B. “Wow, you’ve already done so many problems! I’ll be so proud of you if you can finish before class ends!”

C. “Look! Julia is finished with her assignment and now she gets to draw! Don’t you want to finish your work, too?”

D. “You usually do such a good job finishing your work. I know you can do better!”

A

A. “You’re so close to the end! Why don’t you go get a quick drink from your water bottle and come back and finish this up? I bet it will feel so good to finish this in class so it won’t be homework tonight!”

This response acknowledges Mary’s progress and offers a positive reinforcement by reminding her of the benefits of completing the assignment in class. It also suggests a brief break to refresh and refocus her attention, which can help her regain the mental energy needed to finish the task. This approach encourages Mary to persist while providing support and motivation.

How well did you know this?
1
Not at all
2
3
4
5
Perfectly
70
Q

A student’s IEP requires access to an adapted pencil or pen and a slant board. These are best described as which type of assistive technology?

A. aids for daily living

B. mobility aids

C. low-tech assistive technology

D. high tech assistive technology

A

C. low-tech assistive technology.

Low-tech assistive technology refers to devices or tools that are simple and easy to use, typically not reliant on electronics or advanced technology. In this case, the adapted pencil or pen and the slant board are basic tools that provide physical support and aid in handwriting tasks without requiring complex technology. They are designed to address specific needs related to writing and positioning and are considered low-tech solutions.

How well did you know this?
1
Not at all
2
3
4
5
Perfectly
71
Q

Jacob is a second-grade student with a mild intellectual disability. He is able to meet most grade-level standards when the appropriate accommodations are provided. Jacob is currently having difficulty adding two-digit numbers. Which of the following would be the most appropriate step to take to support Jacob?

A. requesting a meeting with Jacob’s parents to discuss concerns

B. providing in-class one-on-one instruction reviewing place value

C. allowing Jacob to use a calculator to solve two-digit addition problems

D. providing pull-out support by the campus math interventionist

A

B. providing in-class one-on-one instruction reviewing place value.

This option addresses Jacob’s specific learning need by providing targeted instruction on place value, which is fundamental to understanding two-digit addition. By offering one-on-one instruction within the classroom setting, Jacob can receive personalized support tailored to his learning pace and needs while remaining engaged with his peers. This approach aligns with the principles of inclusive education and individualized support for students with disabilities.

How well did you know this?
1
Not at all
2
3
4
5
Perfectly
72
Q

Which of the following activities would be most effective in helping first-graders understand partitioning 2-dimensional shapes into equal parts?

A. use a ruler to measure the perimeter of different 2-dimensional shapes

B. placing cubes on top of 2-dimensional shapes to see how many cubes it takes to fill the shape

C. cutting out different shapes and having students fold them into 2 or 4 equal parts

D. watching the teacher draw a line on a 2-dimensional shape to divide it into 2 equal parts

A

C. cutting out different shapes and having students fold them into 2 or 4 equal parts.

This hands-on activity allows students to physically manipulate shapes and directly observe how they can be divided into equal parts through folding. It provides a concrete and visual representation of partitioning, which is particularly beneficial for young learners. Additionally, by cutting out and folding shapes, students engage in a tactile and interactive experience that promotes deeper understanding and retention of the concept.

How well did you know this?
1
Not at all
2
3
4
5
Perfectly
73
Q

Elijah is a twelve year old student who uses a human scribe or speech-to-text software for written responses. Elijah tested above grade level for reading and mathematics. Elijah tested slightly below grade level for writing and provided limited written responses due to difficulty with handwriting and typing skills. His speech/language therapy goal focuses on improving social interactions with peers. His occupational therapy goal is to improve typing speed and identify and utilize sensory strategies.

Which of the following eligibility classifications is most likely to be documented on Elijah’s Individualized Education Program (IEP)?

A. generalized anxiety disorder

B. orthopedic impairment

C. specific learning disability (SLD)

D. autism spectrum disorder (ASD)

A

D. autism spectrum disorder (ASD)

  • Sensory strategies
  • Social interactions
How well did you know this?
1
Not at all
2
3
4
5
Perfectly
74
Q

Which of the following would NOT be an appropriate classroom support for teaching the alphabetic principle in an inclusion kindergarten room?

A. creating a station that allows students to manipulate some letters in words to create other words (eg. c-at becomes b-at)

B. creating a chalkboard station where students can practice writing each letter

C. creating a sight word word wall with words and pictures to represent each word

D. labeling items in the classroom (eg. labeling the classroom door with “door”)

A

C. creating a sight word word wall with words and pictures to represent each word.

While a sight word word wall is a valuable resource for building sight word recognition, it does not directly support teaching the alphabetic principle. The alphabetic principle focuses on the understanding that letters represent sounds in spoken language, and that these sounds combine to form words. Sight words, on the other hand, are typically irregular words that do not follow phonetic patterns and cannot be sounded out using letter-sound correspondence rules. Therefore, while a sight word word wall may be helpful for other literacy skills, it does not specifically target the alphabetic principle.

How well did you know this?
1
Not at all
2
3
4
5
Perfectly
75
Q

Dylan, a first-grade student with severe medical needs related to DIPG, a form of brain cancer, has an IEP in place to help her best meet her educational goals. Due to the constant transitioning between school, home, and medical environments, Dylan has a hard time keeping up with her peers and classwork. How can Dylan’s new teacher help her to be most successful?

A. The teacher can provide additional work and activities because Dylan misses so much school to ensure that she has all the resources that she needs to be successful and meet her educational goals.

B. The teacher can review the IEP, but due to the frequent transitioning between home, school, and medical environments, the teacher does not need to prioritize Dylan’s educational goals when she is not in school.

C. Due to Dylan’s high medical needs, it is not the teacher’s responsibility to help. Dylan is better off getting support from the hospital staff since that is where she spends so much of her time.

D. The teacher can review Dylan’s IEP to ensure that she understands exactly how to support Dylan and help her reach her educational goals, applying all the appropriate accommodations and modifications that Dylan needs to succeed.

A

D. The teacher can review Dylan’s IEP to ensure that she understands exactly how to support Dylan and help her reach her educational goals, applying all the appropriate accommodations and modifications that Dylan needs to succeed.

Reviewing Dylan’s Individualized Education Program (IEP) is crucial for the teacher to understand Dylan’s specific needs, accommodations, and goals. Given Dylan’s medical condition and frequent transitioning between school, home, and medical environments, it’s essential for the teacher to prioritize supporting Dylan’s educational goals even when she is not in school. This includes providing continuity of instruction, coordinating with other professionals involved in Dylan’s care, and implementing appropriate accommodations and modifications to address her unique needs. This approach ensures that Dylan receives consistent support across all environments and maximizes her chances of academic success despite her medical challenges.

How well did you know this?
1
Not at all
2
3
4
5
Perfectly
76
Q

Which of the following is the most appropriate use of assistive technology that a teacher can implement for a student with significant fine motor deficits?

A. enlarging font size on computers and presentations to reduce eye strain

B. providing extended time to complete written assignments

C. allowing the student to keep a copy of textbooks at home

D. providing speech to text software to record written responses

A

D. providing speech to text software to record written responses.

Speech-to-text software allows students to verbally dictate their responses, which are then transcribed into written text by the software. This accommodation is particularly beneficial for students with fine motor deficits who may struggle with handwriting or typing. It enables them to express their ideas without the physical demands of writing, thereby reducing barriers to participation and promoting independence in completing written assignments.

How well did you know this?
1
Not at all
2
3
4
5
Perfectly
77
Q

Mateo uses a screen reader to complete his assignment during his social studies class. Which of the following disabilities might Mateo be diagnosed with?

A. specific learning disability in math calculation

B. hearing impairment

C. visual impairment

D. specific learning disability in written expression

A

C. visual impairment

How well did you know this?
1
Not at all
2
3
4
5
Perfectly
78
Q

A teacher had implemented the RTI process with a student who was failing to make progress on grade-level math skills, but the student did not respond to the interventions. Which of the following is the next step in the process to determine whether or not the student will receive special education services?

A. develop an IEP to support the student’s learning

B. develop a transition plan to support the student as they move from the general education setting

C. schedule a Review of Existing Evaluation Data (REED)

D. ask the students’ parent or guardian for consent to have the student evaluated

A

D. ask the student’s parent or guardian for consent to have the student evaluated.

Once it has been determined through the RTI process that the student is not making adequate progress despite intervention, the school district must seek parental consent to conduct a comprehensive evaluation to determine if the student qualifies for special education services. This evaluation, known as a Full and Individual Evaluation (FIE) or a Multidisciplinary Evaluation, assesses the student’s strengths and weaknesses in various areas of functioning, including academic, developmental, and behavioral domains. Based on the results of this evaluation, an Individualized Education Program (IEP) team, which includes the student’s parents, will determine eligibility for special education services and develop an appropriate educational plan, if necessary.

How well did you know this?
1
Not at all
2
3
4
5
Perfectly
79
Q

Having students participate in a community and repair harm through inclusive practices is known as:

A. reality therapy.

B. ecological assessment.

C. restorative justice.

D. logical consequences.

A

C. restorative justice.

Restorative justice focuses on repairing harm caused by misconduct or conflict by involving all affected parties in a collaborative process aimed at understanding the impact of actions, taking responsibility, and restoring relationships. This approach emphasizes accountability, empathy, and reconciliation rather than punitive measures. Therefore, option C is the correct term for the described practice.

How well did you know this?
1
Not at all
2
3
4
5
Perfectly
80
Q

Miss Kalie, the school secretary, gets to know Jordan, a new elementary student who has transferred to the district. The new student has obvious behavioral struggles coming to school in the morning, and has already been sent to the office many times to sit with Miss Kalie due to oppositional behaviors in the classroom. The student receives IEP services. Miss Kalie and the special education teacher have been in close contact in regards to tracking Jordan’s behaviors and time in the office to best help support Jordan. Jordan has moved into the house next door to the school secretary’s best friend, Amy. Amy keeps trying to talk to Miss Kalie about the student and what’s happening in the school. The best response Miss Kalie can give to her friend is:

A. Miss Kalie should give Amy every detail that she knows about Jordan and tell her all of the stories of his misbehavior.

B. “Jordan struggles with some behavioral disorder but I’m not really sure. I can just tell you that he receives special education services from our teacher.”

C. “Jordan is a great kid! You’re lucky to be his neighbor!”

D. “Jordan is in the office EVERY day. Sometimes multiple times per day. He has an IEP for oppositional defiant disorder and he is a handful!”

A

C. “Jordan is a great kid! You’re lucky to be his neighbor!”

-CONFIDENTIALITY!

How well did you know this?
1
Not at all
2
3
4
5
Perfectly
81
Q

When discussing a student’s performance in a research-based reading intervention program at an IEP meeting, team members may ask, “Is the student making progress?” Which of the following questions is most important to consider when discussing progress in the program?

A. Has the student increased his/her reading level by at least two grade levels in the calendar year?

B. What do peers think of the intervention program?

C. Is the student now reading on grade level?

D. Has the student increased reading fluency and/or reading comprehension skills?

A

D. Has the student increased reading fluency and/or reading comprehension skills?

This question directly addresses the core goals of a reading intervention program, which typically focus on improving reading fluency (speed and accuracy of reading) and reading comprehension skills (understanding and interpreting text). Monitoring progress in these areas is essential for evaluating the effectiveness of the intervention and determining whether the student’s needs are being met. While questions about reading level (option A) and reading on grade level (option C) are also important, they may not capture the full picture of the student’s progress, as reading level alone does not necessarily indicate fluency or comprehension abilities. The question about peers’ opinions (option B) is less relevant to assessing the effectiveness of the intervention program and addressing the student’s individual needs.

How well did you know this?
1
Not at all
2
3
4
5
Perfectly
82
Q

Which of the following interventions would be most appropriate for a student who continues to struggle to learn how to read grade-level texts fluently?

A. Working with the teacher once a week to practice sight word flash cards.

B. Reading a story with a peer and answering questions together.

C. Joining a small group session twice weekly after school for homework assistance.

D. Working with a reading specialist daily to practice decoding skills and reading strategies.

A

D. Working with a reading specialist daily to practice decoding skills and reading strategies.

This intervention provides intensive and targeted support directly addressing the student’s specific difficulties with reading fluency. Daily sessions with a reading specialist allow for consistent practice and feedback, which are essential for skill development and improvement. Additionally, the reading specialist can tailor instruction to the student’s individual needs, focusing on decoding skills and implementing effective reading strategies. This intensive intervention approach is likely to be most effective in addressing the student’s reading difficulties and helping them progress toward reading fluency at grade level.

How well did you know this?
1
Not at all
2
3
4
5
Perfectly
83
Q

A student with an IEP for an intellectual disability (ID) has been struggling in her general education science classroom. All of the appropriate accommodations and modifications listed on her current IEP have been tried by her general education teacher, but she is not making progress toward her goals and has been acting out due to frustration, which is a distraction to other students. Which of the following would be an appropriate next step for her case manager?

A. Try pulling the student into a self-contained science classroom on a trial basis and track the results to share with the Admission, Review, and Dismissal (ARD) committee at the next meeting.

B. Ask an administrator to speak to her about her behavior and explain why she shouldn’t be distracting others.

C. Communicate concerns with the Admission, Review, and Dismissal (ARD) committee members and schedule an IEP meeting to discuss data and next steps.

D. Assign a paraprofessional to work with her one-on-one in her science classroom to help her be more successful.

A

C. Communicate concerns with the Admission, Review, and Dismissal (ARD) committee members and schedule an IEP meeting to discuss data and next steps.

Communicating concerns with the ARD committee members and scheduling an IEP meeting allows for a comprehensive review of the student’s current situation, including her academic progress, behavioral challenges, and the effectiveness of existing accommodations and modifications. During the meeting, the committee can analyze data, discuss potential reasons for the lack of progress, and collaboratively develop new strategies or adjustments to the student’s IEP to better address her needs. This approach ensures that the student’s educational plan is regularly reviewed and adjusted as necessary to support her academic and behavioral success.

How well did you know this?
1
Not at all
2
3
4
5
Perfectly
84
Q

Mrs. Spisak’s goal in this lesson is to have her students use calculators to develop financial literacy. Which of the following activities best addresses this goal?

A. Have students calculate sales tax and discounts on grocery store items.

B. Multiply a monthly salary by 12 using pencil and paper for an annual budget.

C. Write out fractions on paper with the amount of a monthly bill on top and monthly income on the bottom, which students then put in simplest form.

D. Give students a checkbook register and have a race to see how fast they can find the balance.

A

A. Have students calculate sales tax and discounts on grocery store items.

This activity directly involves students in using calculators to perform real-world financial calculations related to sales tax and discounts, which are important aspects of financial literacy. By using calculators to determine the final cost of items after applying sales tax or discounts, students gain practical experience in using calculators as tools for financial management. Additionally, this activity reinforces concepts such as percentages and calculations involving money, further enhancing students’ understanding of financial literacy. Therefore, option A is the most appropriate activity for achieving Mrs. Spisak’s goal.

How well did you know this?
1
Not at all
2
3
4
5
Perfectly
85
Q

The following project would be appropriate for teaching reading at which grade level?

Students select a character from their independent reading books and complete a character analysis on that character. They must select a trait possessed by the character and find three pieces of evidence from the text to support their thesis. They use sticky notes to mark each piece of evidence and later use their analysis to write an essay.

A. 3rd grade

B. 8th grade

C. 10th grade

D. 6th grade

A

D. 6th grade.

At this grade level, students are often expected to analyze characters in more depth, including identifying character traits and providing evidence from the text to support their analysis. Additionally, writing essays based on textual evidence is commonly introduced and practiced in 6th-grade curriculum. While the project involves higher-order thinking skills, it is structured in a way that is achievable for students at this grade level, making option D the most appropriate choice.

How well did you know this?
1
Not at all
2
3
4
5
Perfectly
86
Q

A 4th-grade class is comparing and contrasting two articles, one about bats and one about birds. Which of the following strategies can the inclusion teacher implement to help his students compare and contrast these articles?

A. completing a Venn Diagram

B. discussing using think-pair-share

C. summarize each article

D. writing annotations on the articles

A

A. completing a Venn Diagram.

A Venn Diagram is an effective graphic organizer for comparing and contrasting two or more topics. Students can use the overlapping circles to identify similarities between bats and birds in the shared area and differences in the separate areas. This visual representation helps students organize their thoughts and see connections between the information presented in the articles. It encourages critical thinking and active engagement with the texts. Therefore, option A is the most appropriate strategy for facilitating comparison and contrast in this context.

How well did you know this?
1
Not at all
2
3
4
5
Perfectly
87
Q

Violet is a fifth-grader with an IEP for ADHD. She is at benchmark in all core subjects but struggles with executive functioning skills, which can affect her work completion and hurt her grades. She needs frequent check-ins to make sure she is on-task during work time and reminders about deadlines. Which of the following is the most appropriate setting for Violet?

A. a general education setting with a paraprofessional to support her

B. a special education setting

C. a self-contained resource setting

D. a general education classroom

A

D. a general education classroom.

Violet’s academic performance indicates that she is capable of keeping up with grade-level content in a general education setting. However, she requires additional support for her executive functioning skills, such as frequent check-ins and reminders about deadlines. These accommodations can be provided within a general education classroom through the implementation of her Individualized Education Program (IEP). Additionally, being in a general education classroom allows Violet to access the same curriculum as her peers and participate in inclusive learning experiences, which can promote her academic and social growth. Therefore, option D is the most suitable setting for Violet.

How well did you know this?
1
Not at all
2
3
4
5
Perfectly
88
Q

Ms. Lopez, a special education teacher, receives a concerned email from Laura, the parent of one of her students, stating that she does not want her daughter to be required to sit through the statewide standardized test due to her ADHD diagnosis. She cites her daughter’s difficult behavior and extreme stress leading up to the test and asks for her to be exempt from testing for the year. What is the best course of action for Ms. Lopez?

A. Call the parent to address her concerns. Explain that all students are required to take the statewide standardized test and offer to schedule an emergency IEP meeting to discuss testing accommodations that might help ease her daughter’s difficulty with the test.

B. Send an email reply explaining that all students are required to take the statewide standardized test and remind her that her daughter’s scores will not affect any academic grades.

C. Send an email stating that she understands Laura’s concerns and forward her email to the principal who can send the appropriate paperwork to opt-out of the test.

D. Call the parent to address the concerns and get verbal approval to change the IEP to reflect that the student will not be required to sit for the statewide standardized test.

A

A. Call the parent to address her concerns. Explain that all students are required to take the statewide standardized test and offer to schedule an emergency IEP meeting to discuss testing accommodations that might help ease her daughter’s difficulty with the test.

It’s important for Ms. Lopez to communicate directly with the parent to address her concerns and provide information about the importance of the statewide standardized test. Ms. Lopez can explain that the test is a requirement for all students and that accommodations can be made to support her daughter during the testing process. Offering to schedule an emergency IEP meeting demonstrates a proactive approach to addressing the student’s needs and finding appropriate accommodations that can help alleviate her stress and difficulties with the test. This approach allows for collaboration between the school and the parent to find solutions that meet the student’s needs while fulfilling testing requirements. Therefore, option A is the most appropriate course of action.

How well did you know this?
1
Not at all
2
3
4
5
Perfectly
89
Q

Which of the following strategies is most likely to effectively support an eighth-grade student with autism spectrum disorder (ASD) who is struggling to understand the concept of maintaining balance when solving a single-variable equation?

A. Assign the student to work with a peer partner who understands the process and can explain it in more relatable language.

B. Teach the student to input and solve the equation with a graphing calculator.

C. Use a simple balance scale with amounts of mass on each side to demonstrate that when anything is added or removed from one side, the same must be done on the opposite side to keep the scale in balance.

D. Provide the student with a laminated “cheat sheet” showing the steps to follow when solving an algebraic equation.

A

C. Use a simple balance scale with amounts of mass on each side to demonstrate that when anything is added or removed from one side, the same must be done on the opposite side to keep the scale in balance.

This strategy utilizes a concrete and hands-on approach to teach the abstract concept of maintaining balance in equations, which can be particularly effective for students with autism spectrum disorder (ASD) who may benefit from visual and tactile learning experiences. By using a balance scale with physical objects representing the terms of the equation, the student can directly observe the effects of adding or removing mass from one side and the need to maintain balance by making corresponding changes on the other side. This concrete representation helps make the abstract concept of equation solving more tangible and accessible for the student. Therefore, option C is the most likely strategy to effectively support the student in understanding and mastering the concept.

How well did you know this?
1
Not at all
2
3
4
5
Perfectly
90
Q

A student in fifth grade attends a life skills class, is preverbal, has dysphagia, has multiple orthopedic impairments, uses a wheelchair, and she also uses a communication board. What is a potential bias that educators must be aware of when working with this student?

A. the assumption that the student needs assistance to eat

B. the assumption that the student requires robust support to access education

C. the assumption that the student needs to enter and exit the school using doors near wheelchair ramps

D. the assumption that the student does not understand the communication around her

A

D. the assumption that the student does not understand the communication around her.

It’s important for educators to avoid making assumptions about a student’s understanding or comprehension based on their preverbal status or use of alternative communication methods such as a communication board. Just because a student is preverbal or uses alternative communication does not necessarily mean they do not understand the communication around them. Students with disabilities may have varying levels of comprehension and understanding, and it’s essential for educators to recognize and respect their individual abilities and strengths. Assuming that the student does not understand communication can lead to underestimating their capabilities and may result in missed opportunities for meaningful engagement and learning. Therefore, option D highlights a potential bias that educators must be vigilant about and strive to address through inclusive and supportive practices.

How well did you know this?
1
Not at all
2
3
4
5
Perfectly
91
Q

An eighth-grade special education teacher is planning a unit on argumentative writing which will require students to select a topic to study and will culminate in a podcast written and recorded by each student in which they take a stance on a controversial issue (eg. Are school sports beneficial for kids? Should uniforms be required? Are beauty pageants harmful?, etc.) and support their position with evidence from their research. Which of the following lessons would be the most appropriate start to this unit?

A. a lesson reviewing how to write a thesis statement followed by asking each student to complete a graphic organizer that provides a sentence frame for them to write their own thesis statement

B. a lesson that asks students to listen to an argumentative podcast and discuss as a whole group how the author proved their point

C. a game played as a whole group that asks students to determine if a statement is a fact or an opinion

D. a lesson in which students watch “trailers” or videos on each of the potential topics and consider which issue might interest them

A

A. a lesson reviewing how to write a thesis statement followed by asking each student to complete a graphic organizer that provides a sentence frame for them to write their own thesis statement

How well did you know this?
1
Not at all
2
3
4
5
Perfectly
92
Q

A student identified with emotional disturbance (ED) is transitioning from elementary school to middle school. In her elementary school, she spent a portion of the day working with the custodial staff collecting recycling from classrooms to work toward a behavioral goal in her IEP. This was a successful intervention for the student, so the new team at the middle school is hoping to do something similar in the new environment. Which of the following would be the most appropriate course of action for the transition team?

A. nterventions listed on the IEP must be administered by special education staff according to IDEA regulations.

B. Ask the parents what they would prefer and follow their recommendations.

C. The student should not be given the option to spend time out of the classroom as that will detract from her academic time and cause her to fall behind.

D. Give her the option to work with custodial or food preparation staff for a portion of the day on a trial basis to see if the intervention continues to be helpful for her.

A

D. Give her the option to work with custodial or food preparation staff for a portion of the day on a trial basis to see if the intervention continues to be helpful for her.

This option acknowledges the success of the intervention in the student’s previous school environment and allows for continuity of support in the new middle school setting. It provides the student with a familiar and effective strategy for addressing her behavioral needs while transitioning to a new school. Additionally, implementing the intervention on a trial basis allows the transition team to assess its effectiveness in the new context and make adjustments as needed. By offering the student the option to continue working with custodial or food preparation staff, the transition team can support her transition to middle school while addressing her behavioral goals outlined in her IEP. Therefore, option D is the most appropriate course of action.

How well did you know this?
1
Not at all
2
3
4
5
Perfectly
93
Q

A special education teacher is planning a lesson on the parts of speech. She will end the lesson with an activity in which students will make sentences independently by putting cards with words on them in an order specified by the teacher. For example, she will call for a noun followed by a verb and students will select one of each from their set to use in their sentence. Which of the following would be the most appropriate way to differentiate for this lesson?

A. Create some sets of cards that are color coded (green for verbs, red for nouns, etc.) and some that are white and allow students to choose between them.

B. Reduce the amount of different types of speech that some students are asked to learn.

C. Allow some students who may struggle to focus during the activity to use fidgets as necessary.

D. Allow students who may struggle with the task to work with a partner who is stronger with the skill.

A

A. Create some sets of cards that are color coded (green for verbs, red for nouns, etc.) and some that are white and allow students to choose between them.

How well did you know this?
1
Not at all
2
3
4
5
Perfectly
94
Q

During independent reading time, Mrs. Clarke walks around and conducts quick, individual readers’ conferences. After talking to Stephanie, Mrs. Clarke is concerned because it is clear that she is not comprehending the story. To help Stephanie read more confidently, which step should Mrs. Clarke take first?

A. Ask Stephanie to switch to a text with a lower reading level, then perform another reader’s conference.

B. Have Stephanie answer a series of multiple-choice reading comprehension questions about the story.

C. Contact Stephanie’s parents to discuss how much time is spent at home reading.

D. Perform an Informal Reading Inventory and provide a remediation lesson based on these results.

A

D. Perform an Informal Reading Inventory and provide a remediation lesson based on these results.

Conducting an Informal Reading Inventory (IRI) will allow Mrs. Clarke to gather detailed information about Stephanie’s reading abilities, including strengths and areas of difficulty. Based on the results of the IRI, Mrs. Clarke can identify specific skills or strategies that Stephanie needs support with and tailor her instruction accordingly. Providing a remediation lesson targeted at addressing Stephanie’s specific needs will be more effective in improving her comprehension and confidence than simply switching to a text with a lower reading level or having her answer multiple-choice comprehension questions. Therefore, option D is the most appropriate step for Mrs. Clarke to take first.

How well did you know this?
1
Not at all
2
3
4
5
Perfectly
95
Q

Ms. Cearing has observed that one of her students with a specific learning disability is struggling to comprehend stories read aloud in class. Which activity could she choose to support this student’s comprehension during whole-class read-alouds?

A. Ask the student to repeat new or challenging words after she says them.

B. Ask the student to respond to discussion questions after the reading is done.

C. Ask the student to visualize what is being read aloud by the teacher.

D. Ask the student to spell familiar and unfamiliar words from the reading.

A

C. Ask the student to visualize what is being read aloud by the teacher.

Visualizing involves creating mental images or “mind movies” based on the text being read or heard. For students with specific learning disabilities who may struggle with comprehension, visualizing can be an effective strategy to enhance understanding and engagement with the text. By asking the student to visualize the events, characters, settings, and actions described in the story, Ms. Cearing can help the student create a mental picture of the text, which can aid in comprehension and retention. This strategy encourages active engagement with the text and can improve the student’s overall comprehension skills. Therefore, option C is the most appropriate activity to support the student’s comprehension during whole-class read-alouds.

How well did you know this?
1
Not at all
2
3
4
5
Perfectly
96
Q

Jordan is a fourth-grade student with dyslexia who struggles with sounding out unfamiliar words. Which of the following would be the most helpful strategy to use to help Jordan get more comfortable with sounding out new words?

A. reviewing sight words with him and making a book of sight words that he can use to study at home

B. holding up flashcards with letters or words on them and asking Jordan to write them out in shaving cream

C. creating cards with phonemes and graphemes that he can manipulate to make words and work together to sound them out

D. using flashcards to review basic punctuation to help Jordan review how to read sentences based on the punctuation used

A

C. creating cards with phonemes and graphemes that he can manipulate to make words and work together to sound them out.

For a student with dyslexia like Jordan who struggles with sounding out unfamiliar words, a multisensory approach that involves manipulating phonemes and graphemes can be highly beneficial. This strategy aligns with the principles of structured literacy, which emphasizes explicit instruction in phonemic awareness, phonics, and decoding skills. By creating cards with phonemes and graphemes, Jordan can practice blending sounds together to form words, which can improve his ability to sound out unfamiliar words. Additionally, working together with the teacher or a peer provides opportunities for guided practice and feedback, further supporting Jordan’s learning. Therefore, option C is the most appropriate strategy to help Jordan improve his skills in sounding out new words.

How well did you know this?
1
Not at all
2
3
4
5
Perfectly
97
Q

Which of the following situations is a violation of FERPA (Family Educational Rights and Privacy Act)?

A. A teacher shares some details of a student’s IEP with a classroom volunteer who will be working with the student in a small group.

B. A teacher speaks with an occupational therapist about a student’s IEP goal progress during class.

C. A teacher discusses details of a student’s IEP with a teacher from another school without disclosing the student’s name.

D. A teacher lists some details of a student’s IEP in the classroom’s substitute teacher folder.

A

D. A teacher lists some details of a student’s IEP in the classroom’s substitute teacher folder.

FERPA protects the privacy of student education records, including Individualized Education Program (IEP) details. Sharing details of a student’s IEP, such as accommodations or goals, in a substitute teacher folder would violate FERPA because it exposes sensitive information to individuals who do not have a legitimate educational interest in the student’s records. It is important to maintain the confidentiality of student records and only share information on a need-to-know basis with individuals directly involved in the student’s education. Therefore, option D represents a violation of FERPA.

How well did you know this?
1
Not at all
2
3
4
5
Perfectly
98
Q

A student who uses a wheelchair and needs healthcare services related to a breathing machine is enrolled in special education services. He is considered medically fragile and requires nursing care 24 hours a day. The school would like to share the cost of the nurse with the family. How the cost be legally shared?

A. The school can work with the insurance company to share costs, but there cannot be a cost to the family.

B. The school covers the same percentage amount as the family’s insurance company.

C. The school can share costs with the family if it allows them to select the nurse who provides services.

D. The school requires the family to apply for Medicaid services in order to share costs with another state fund.

A

A. The school can work with the insurance company to share costs, but there cannot be a cost to the family.

How well did you know this?
1
Not at all
2
3
4
5
Perfectly
99
Q

As part of her first-week student survey, Mrs. Walker asks students to provide the titles of two books they enjoyed, one book they started and never finished, one book they finished but didn’t love, and their favorite genre(s) to read. Which of the following would be the most reasonable purpose for gathering this information?

A. assessing student reading level in order to ensure you assign appropriately leveled books in the future.

B. evaluating student mastery of previous year’s reading goals

C. gaining an impression of student reading habits and interests for consideration when choosing class readings

D. gaining a first impression of student writing and spelling skills

A

C. gaining an impression of student reading habits and interests for consideration when choosing class readings.

This information would help Mrs. Walker understand what types of books and genres her students enjoy reading, as well as their preferences and experiences with different books. With this insight, she can select class readings that are more likely to engage her students and cater to their interests. Understanding students’ reading habits and interests can also help Mrs. Walker create a more personalized and effective reading curriculum that meets the needs and preferences of her students. Therefore, option C is the most reasonable purpose for gathering this information.

How well did you know this?
1
Not at all
2
3
4
5
Perfectly
100
Q

A first grade student is continuing to struggle with blending onsets and rimes. Most of the class has grasped this skill and is ready to move on. The teacher recognizes the need to back up and review a previous skill with the student before continuing with instruction on blending onsets and rimes. Which of the following would be the most reasonable skill for her to back up and review with the student?

A. syllable awareness

B. word awareness

C. letter to sound correspondence

D. phoneme substitution

A

A. syllable awareness

How well did you know this?
1
Not at all
2
3
4
5
Perfectly
101
Q

A new eighth-grade teacher is finding that her class gets rowdy quickly, and she has a hard time keeping them focused during direct instruction. Which of the following factors is likely to be the root cause of her issue?

A. She often builds opportunities for movement into her lessons.

B. She often moves around the classroom as she is teaching a lesson.

C. She often speaks quickly and in an excited tone.

D. She often allows students to eat the school snack at the beginning of class.

A

C. She often speaks quickly and in an excited tone.

Speaking quickly and in an excited tone can inadvertently contribute to a classroom environment that becomes rowdy quickly and has a hard time staying focused. When a teacher speaks quickly, it can be difficult for students to process the information being presented, leading to confusion and distraction. Additionally, an excited tone can inadvertently convey a sense of urgency or excitement that may energize students and lead to increased chatter and restlessness. Therefore, option C is likely the root cause of the teacher’s issue in maintaining focus during direct instruction.

How well did you know this?
1
Not at all
2
3
4
5
Perfectly
102
Q

A 4th-grade student has recently begun taking ADHD medicine. The student’s parents have asked the teacher to let them know if she notices any changes in the student throughout the school day. Which of the following is a common side effect associated with common ADHD medicine?

A. dizziness

B. night terrors

C. seizures

D. lack of hunger

A

D. lack of hunger

One common side effect associated with many ADHD medications is a decrease in appetite or lack of hunger. This side effect can result in reduced food intake, which may affect a student’s energy levels and overall well-being throughout the school day. Therefore, if the teacher notices changes in the student’s appetite or eating habits, it could be a potential side effect of the ADHD medication.

How well did you know this?
1
Not at all
2
3
4
5
Perfectly
103
Q

Scarlett is a student in sixth grade who qualifies for special education as having an intellectual disability. To be academically successful, Scarlett requires her curriculum to be modified to a kindergarten level in all content areas.

What would be an appropriate goal in Scarlett’s IEP?

A. By the end of the second grading period, when given objects and a visual for a number, Scarlett will demonstrate an understanding of the quantities that numbers to 10 represent by counting out the designated number of objects with 90% accuracy in four out of five trials over four sessions, as measured by observation, data, and work samples.

B. Scarlett will identify groups of ten similar concrete objects, such as ten coins, ten bricks, or ten pencils.

C. the end of the second grading period, Scarlett will demonstrate an understanding of multiplication fact families with 80% accuracy in four out of five trials over four sessions, as measured by observation, data, and work samples.

D. By the end of the second grading period, Scarlett will demonstrate an understanding of commutative property with 80% accuracy in four out of five trials over four sessions, as measured by observation, data, and work samples.

A

A. By the end of the second grading period, when given objects and a visual for a number, Scarlett will demonstrate an understanding of the quantities that numbers to 10 represent by counting out the designated number of objects with 90% accuracy in four out of five trials over four sessions, as measured by observation, data, and work samples.

Option A presents a goal that aligns with Scarlett’s academic level, focusing on her ability to understand quantities represented by numbers up to 10. This goal is appropriate because it is tailored to Scarlett’s current level of functioning and provides a specific target for her to achieve within a defined timeframe. The goal also includes measurable criteria for determining Scarlett’s progress, which is essential for effective monitoring and adjustment of her educational program.

How well did you know this?
1
Not at all
2
3
4
5
Perfectly
104
Q

A special education teacher is struggling with the classroom behavior of one of her students who is identified as having an emotional disturbance (ED). She feels she needs advice but would need to share specific identifying details about the student and his IEP to best convey the situation. Who would it be most appropriate for her to seek advice from?

A. another special education teacher at a different school

B. to adhere to FERPA, she may not discuss details of the student’s IEP with any other staff

C. the student’s parent

D. the in-school social worker that works with the student and attends all of the IEP meetings

A

D. the in-school social worker that works with the student and attends all of the IEP meetings

The in-school social worker is already involved in the student’s case and is likely familiar with the student’s needs, behaviors, and IEP goals. Seeking advice from the social worker would be appropriate because they have firsthand knowledge of the situation and can provide guidance based on their professional expertise and experience working with the student. Additionally, since the social worker attends IEP meetings, they are aware of the student’s individualized plan and can offer insights into how to effectively support the student within the framework of their IEP.

How well did you know this?
1
Not at all
2
3
4
5
Perfectly
105
Q

A life skills classroom with several non-verbal students and students with significant motor-control deficits is beginning a unit on reading comprehension. The teacher will provide a 2-3 sentence story and ask two simple questions about what the students have read. Which of the following would NOT be an appropriate support to include in the instructional design?

A. one-on-one support from a paraprofessional to help students indicate their answers

B. picture supports in multiple-choice answers

C. large print on white paper to make the writing as clear as possible

D. dobbers (large circular stamps) to allow them to easily select their answers

A

C. large print on white paper to make the writing as clear as possible

While large print can be helpful for students with visual impairments or those who may have difficulty reading small text, it may not be beneficial for students with significant motor-control deficits or non-verbal students. These students may require alternative methods of accessing and interacting with the text, such as using assistive technology, communication devices, or tactile materials. Large print alone may not adequately address their needs and may not be the most effective support for facilitating reading comprehension in this context.

How well did you know this?
1
Not at all
2
3
4
5
Perfectly
106
Q

All of the following are assessed using a functional behavior assessment (FBA) except:

A. the cause of negative behavior.

B. skills that can be taught to help lessen the negative behavior.

C. what negative behaviors are being exhibited.

D. rewards that might help motivate the student to lessen the negative behavior.

A

D. rewards that might help motivate the student to lessen the negative behavior.

A functional behavior assessment (FBA) is primarily focused on understanding the function or purpose of the behavior, identifying the antecedents (triggers) and consequences that maintain the behavior, and determining the environmental factors that may influence the behavior. It seeks to answer questions about the function of the behavior (i.e., why the behavior is occurring), the specific behaviors being exhibited, and the skills or interventions that can be implemented to address the behavior. While identifying potential rewards or incentives to encourage positive behavior change may be part of the behavior intervention planning process that follows the FBA, it is not typically a component of the FBA itself.

How well did you know this?
1
Not at all
2
3
4
5
Perfectly
107
Q

At an initial Admission Review Dismissal (ARD) committee meeting for a 3-year-old who was referred to the team for suspected developmental delays in the cognitive, communication, and adaptive domains, the evaluation team must make sure to obtain the parents’ informed written consent before they begin the evaluation process. What would the next step be for the team to take?

A. Establish a time and location for the developmental evaluation.

B. Determine what areas the team will evaluate.

C. Send the parents an invitation to an early-enrollment event offered for their preschool.

D. Determine who will complete the assessment and provide services to the child.

A

A. Establish a time and location for the developmental evaluation.

How well did you know this?
1
Not at all
2
3
4
5
Perfectly
108
Q

A 2nd-grade teacher and a special education teacher are planning a co-taught reading and writing block. They are planning to partner students for buddy reading and use those same partners for the first project in which students will read a book and retell the story with puppets. Which of the following would be the most appropriate way to group the students?

A. Use the data from a student interest survey collected at the beginning of the unit to indicate what the students are interested in reading.

B. Use the data from the previous year-end developmental reading assessments.

C. Use the data collected from parents about their child’s needs and preferences for partnerships.

D. Use the data from the first developmental reading assessment of the year administered by the teacher.

A

D. Use the data from the first developmental reading assessment of the year administered by the teacher.

The most appropriate way to group the students for buddy reading and the puppet project would be to use the data from the first developmental reading assessment of the year administered by the teacher (Option D). This assessment would provide insights into each student’s current reading level, strengths, and areas for growth. Grouping students based on their reading abilities ensures that each pair is balanced and that students can support each other effectively during buddy reading sessions and collaborative projects.

How well did you know this?
1
Not at all
2
3
4
5
Perfectly
109
Q

Ben’s parents are struggling with how they are going to support their son, who suffers from a mild intellectual disability, in his transition from high school to post-high school life. Ben has been enrolled in vocational training and is prepared to work after graduation. He would like to move out of his parents’ home, but struggles to keep to a routine and remember to do daily care tasks. Which of the following is a helpful solution that his teacher might be able to suggest to this family?

A. Hire an in-home care assistant to help Ben with his daily tasks so his parents won’t have to worry about him.

B. Call a local group home that is designed for individuals with disabilities to live as independently as possible.

C. It is not the teacher’s place to make these types of suggestions to the family.

D. Ben is not able to care for himself independently. He must live with his parents.

A

B. Call a local group home that is designed for individuals with disabilities to live as independently as possible.

Group homes often provide structured environments where individuals with disabilities can receive support with daily tasks while also fostering independence. This option allows Ben to live semi-independently while still receiving the necessary support to maintain a routine and complete daily care tasks. It’s important to consider all available options that meet Ben’s needs and preferences while also ensuring his safety and well-being.

How well did you know this?
1
Not at all
2
3
4
5
Perfectly
110
Q

Mr. Amendola teaches an inclusive ninth-grade science class where he has weekly quizzes on learning material but uses a flexible approach to grading. He provides opportunities for additional instruction and allows all of his students to repeat quizzes multiple times. Mr. Amendola most likely takes this approach because:

A. it allows him to have the same procedures and expectations for the general education and special education students in the class.

B. it gives him a way to continuously collect data on student progress in order to differentiate his instructional methods based on his students’ learning needs.

C. he has heard the best way to build rapport with students is to let them repeat quizzes to raise their grades.

D. he has a really big heart and does not want to see any of his students fail.

A

B: it gives him a way to continuously collect data on student progress in order to differentiate his instructional methods based on his students’ learning needs.

By allowing students multiple attempts to demonstrate their understanding, Mr. Amendola can gather valuable information about their areas of strength and areas needing improvement. This data allows him to tailor his instruction to better meet the diverse needs of all students in his inclusive classroom.

How well did you know this?
1
Not at all
2
3
4
5
Perfectly
111
Q

A first-grade teacher is finishing a unit on place value and composing/decomposing numbers using hundreds, tens, and ones. Which of the following would help to ensure that students continue practicing this skill even after the unit is finished?

A. a “number of the day” that students model using hundreds, tens, and ones

B. an online game in which students identify the hundreds, tens, and ones place

C. counting each day of school by adding a popsicle stick to a jar and making groups of tens when applicable

D. all of the above

A

Option D, “all of the above,” would help ensure that students continue practicing place value skills even after the unit is finished.

A. A “number of the day” activity allows students to regularly engage with place value concepts by modeling numbers using hundreds, tens, and ones.

B. An online game that requires students to identify the hundreds, tens, and ones place reinforces their understanding of place value in a fun and interactive way.

C. Counting each day of school and grouping objects into tens when applicable provides ongoing practice with place value as students see how numbers are composed of hundreds, tens, and ones.

By incorporating these activities, students can continue to reinforce and apply their understanding of place value beyond the specific unit on the topic.

How well did you know this?
1
Not at all
2
3
4
5
Perfectly
112
Q

Kylie is a second-grade student who is struggling with fine motor tasks such as holding a pencil, writing her name, and cutting with scissors. Her parents suspect that she may have a disability. Which of the following professionals would be most likely to evaluate Kylie?

A. school psychologist

B. second grade teacher

C. occupational therapist

D. school nurse

A

C. occupational therapist

How well did you know this?
1
Not at all
2
3
4
5
Perfectly
113
Q

A first grade student is struggling to develop reading skills. A teacher noted the student’s reading challenges while screening first grade students for possible learning disabilities. Challenges with which of the following specific reading skills is most likely to signal to the teacher the student may have a reading disability?

A. reading fluency

B. phonological awareness

C. morphology

D. listening comprehension

A

Option B, phonological awareness, is most likely to signal to the teacher that the student may have a reading disability.

Phonological awareness refers to the ability to recognize and manipulate the sounds of spoken language, including identifying and blending individual sounds (phonemes) to form words. Difficulties with phonological awareness, such as struggling to identify or manipulate sounds, are often early indicators of potential reading difficulties or dyslexia.

While reading fluency, morphology, and listening comprehension are all important components of reading development, challenges with phonological awareness are particularly significant in identifying potential reading disabilities, especially in early elementary grades like first grade.

How well did you know this?
1
Not at all
2
3
4
5
Perfectly
114
Q

Which of the following would NOT be an appropriate use of formative assessment data?

A. determining student comprehension of a skill or concept taught

B. using the data to group students into small instructional groups

C. gathering self-reported data on how comfortable students feel with a concept or unit

D. determining mastery of unit skills

A

Option D, determining mastery of unit skills, would NOT be an appropriate use of formative assessment data.

Formative assessment is designed to provide feedback on student learning while instruction is still ongoing, helping teachers adjust their teaching to better meet students’ needs. It is typically used to monitor student progress, identify areas of strength and weakness, inform instructional decisions, and guide ongoing learning. Formative assessment is not typically used to determine mastery of unit skills; instead, summative assessments are generally used for that purpose. Summative assessments are administered at the end of a unit or period of instruction to evaluate student learning and determine mastery of specific skills or standards.

How well did you know this?
1
Not at all
2
3
4
5
Perfectly
115
Q

An occupational therapist may be asked to consult with teachers and share recommendations for which of the following situations?

A. A student who is wheelchair-bound is learning to independently transfer to the toilet.

B. A student with an emotional disability was hospitalized due to depression and is preparing to return to school.

C. A student with sound errors is reluctant to participate in class for fear of teasing from peers.

D. A student who has arthritis is having difficulty holding a pencil and writing legibly.

A

D. A student who has arthritis is having difficulty holding a pencil and writing legibly.

Occupational therapists specialize in helping individuals develop, recover, or maintain daily living and work skills. In an educational setting, they often work with students who have physical, developmental, or cognitive challenges that affect their ability to participate in school activities. For a student with arthritis who is struggling with handwriting due to difficulty holding a pencil and writing legibly, an occupational therapist can provide recommendations for adaptive tools, strategies, and interventions to help improve the student’s handwriting and overall participation in classroom activities.

How well did you know this?
1
Not at all
2
3
4
5
Perfectly
116
Q

Hanna is a ninth grade student who has been diagnosed with attention deficit hyperactivity disorder (ADHD) and asthma. Hanna takes medication for both conditions inconsistently. Hanna is failing all classes despite accommodations. Which of the following statements best describes the most appropriate services for Hanna?

A. Hanna has a physical impairment which only allows eligibility under Section 504.

B. Hanna’s performance requires referral to the special education team for consideration of eligibility for an Individualized Education Program (IEP).

C. Hanna is ineligible for additional support until she agrees to take medication as directed by her physician.

D. Hanna requires homebound instruction to prevent exposure to illness from peers.

A

B. Hanna’s performance requires referral to the special education team for consideration of eligibility for an Individualized Education Program (IEP).

Given Hanna’s diagnoses of ADHD and asthma, along with her inconsistent medication use and academic struggles despite accommodations, it’s likely that she requires more targeted support beyond what is typically provided through Section 504 accommodations. An IEP would allow for a more comprehensive evaluation of her needs and the development of specific interventions and services to address her academic, behavioral, and health-related challenges. This process would involve the special education team, including assessments by relevant professionals, to determine eligibility and appropriate services tailored to Hanna’s needs.

How well did you know this?
1
Not at all
2
3
4
5
Perfectly
117
Q

An Admission, Review, and Dismissal (ARD) committee is meeting for a fourth-grade student’s annual IEP review in September. The case manager notices that many of the listed accommodations have been in place without any change for several years but the student has matured significantly over the past year and many of the accommodations may be outdated. Which of the following would be the best course of action for the case manager?

A. Ask the parents to look through the list and remove any accommodations that they do not feel are appropriate for their child at this time.

B. Review each listed accommodation and remove any that are no longer appropriate. Ask the special education administrator to review all changes before finalizing them.

C. While facilitating the meeting, make sure to take time to list all of the accommodations and get input from the student, parents, and general education teachers about which may be appropriate and which can be removed.

D. Prior to the meeting, remove all accommodations that seem outdated and use the data from the most recent exams to compile a list of possible accommodations to be reviewed by the ARD committee.

A

C. While facilitating the meeting, make sure to take time to list all of the accommodations and get input from the student, parents, and general education teachers about which may be appropriate and which can be removed.

This option involves a collaborative approach where all relevant stakeholders, including the student, parents, and general education teachers, provide input on the effectiveness and relevance of the accommodations. By engaging in discussion and considering different perspectives, the ARD committee can make informed decisions about which accommodations are still necessary and which ones may need to be updated or removed based on the student’s current needs and progress. This ensures that the student’s IEP reflects their current level of functioning and provides appropriate support for their continued growth and success.

How well did you know this?
1
Not at all
2
3
4
5
Perfectly
118
Q

Liam is a 5th-grade student who is twice-exceptional; he has an ADHD diagnosis and a gifted and talented placement. His teacher notices that he is struggling with writing, often missing words or letters in words, leaving out spaces, and leaving out punctuation. She reaches out to the special education teacher to ask for advice. Which of the following would NOT be a beneficial intervention for Liam?

A. allowing him to use speech-to-text software on big assignments instead of handwriting

B. encouraging him to organize his thoughts on a graphic organizer before beginning to write

C. encouraging him to skip lines when he is writing to help slow him down and give him room to edit

D. encouraging him to read his writing out loud and editing before moving on to a new idea

A

B. encouraging him to organize his thoughts on a graphic organizer before beginning to write

How well did you know this?
1
Not at all
2
3
4
5
Perfectly
119
Q

Sebastian, a second-grade student, struggles with an auditory processing disorder and attention issues during learning periods. The IEP team has decided to try an assistive technology device to see if it improves Sebastian’s attention and ability to process what he is hearing. The assistive technology device that would best suit the student’s need is a(n):

A. Dynavox

B. FM Listening system

C. graphic organizer

D. audio recorder

A

B. FM Listening system

An FM Listening system would be particularly beneficial for Sebastian’s auditory processing disorder and attention issues during learning periods. This system helps by reducing background noise and amplifying the teacher’s voice directly into Sebastian’s ears, making it easier for him to focus on and process spoken information. This can enhance his ability to comprehend what is being said in the classroom despite his auditory processing challenges and attention issues.

How well did you know this?
1
Not at all
2
3
4
5
Perfectly
120
Q

Dominique is a sixth-grade student with a specific learning disability (SLD) in math calculation. His teachers have noticed that he struggles to solve problems involving proportion, despite the use of calculation aids specified in his IEP. Which of the following is the logical next step to diagnose the source of Dominique’s trouble with proportions?

A. Assess his understanding of the relationship between fractions, decimals, and ratios and his ability to combine and compare them.

B. Assess his ability to solve one-variable equations.

C. Assess his ability to identify the correct operation to perform when solving word problems.

D. Assess his ability to graph the slope of a line represented by an equation.

A

A. Assess his understanding of the relationship between fractions, decimals, and ratios and his ability to combine and compare them.

Since Dominique is struggling with proportions, which involve the comparison of different quantities, assessing his understanding of fractions, decimals, and ratios, as well as his ability to combine and compare them, would be the logical next step. This assessment would help pinpoint any specific areas of weakness or misunderstanding that may be contributing to his difficulties with proportions. By evaluating his foundational understanding of these mathematical concepts, educators can better tailor interventions and support to address Dominique’s needs.

How well did you know this?
1
Not at all
2
3
4
5
Perfectly
121
Q

A middle-school student with a mild intellectual disability and hearing impairment spends her language arts and math blocks in a special education setting and is in the general education classroom for science and social studies. Which of the following aspects of her IEP would be most important for the special education teacher to review with the general education teachers prior to the start of classes at the beginning of the year?

A. special education teachers should not discuss a student’s IEP with others

B. her academic goals for the upcoming year

C. her most recent academic grades

D. the list of classroom accommodations and modifications

A

D. the list of classroom accommodations and modifications

Reviewing the list of classroom accommodations and modifications with the general education teachers would be the most important aspect of the student’s IEP to discuss prior to the start of classes. These accommodations and modifications are designed to support the student’s access to the general education curriculum and ensure that she can participate and make progress in the classroom. By understanding the specific supports outlined in the student’s IEP, the general education teachers can effectively implement strategies to meet her individual needs and provide appropriate accommodations during instruction.

How well did you know this?
1
Not at all
2
3
4
5
Perfectly
122
Q

A middle school language arts teacher has received notification a new special education student has transferred to the school and will be joining her class. As the teacher prepares lesson plans for the upcoming week, which of the following is the most important resource for the teacher to use when planning for the new student’s learning goals in language arts?

A. the student’s IEP

B. a conversation with the student’s tracking teacher

C. a conversation with the student’s preview ELA teacher

D. the state standards for ELA

A

A. the student’s IEP

The student’s Individualized Education Program (IEP) is the most important resource for the teacher to use when planning for the new student’s learning goals in language arts. The IEP outlines the student’s specific learning needs, accommodations, modifications, and goals related to language arts instruction. By reviewing the student’s IEP, the teacher can gain valuable insights into how to best support the student’s learning and ensure that instruction is aligned with the student’s individual needs and goals.

How well did you know this?
1
Not at all
2
3
4
5
Perfectly
123
Q

Mariam, a fourth-grader, arrives in her classroom and informs Mr. Greene, her homeroom teacher, that she is fasting for Ramadan and she may not have any food or water during the day. Which of the following would be the most appropriate course of action for Mr. Greene?

A. Announce to the class that, out of respect for Mariam’s religious beliefs, all students should avoid eating or drinking in front of Mariam.

B. Express to the student that not drinking water all day is unhealthy and suggest that she drink some water when she feels she needs to.

C. Politely explain to Mariam that discussing religious beliefs in school is inappropriate and that she should not discuss Ramadan with her peers while at school.

D. Thank Mariam for letting him know and show her an alternative place she can go during the class’s snack time if she wishes.

A

D. Thank Mariam for letting him know and show her an alternative place she can go during the class’s snack time if she wishes.

In this situation, it’s important for Mr. Greene to acknowledge and respect Mariam’s religious beliefs and practices while also ensuring her well-being during fasting. Offering her an alternative place to go during snack time allows Mariam to observe her fast while also providing her with a comfortable environment away from food and drink. This approach demonstrates sensitivity to Mariam’s religious needs while also accommodating her within the classroom environment.

How well did you know this?
1
Not at all
2
3
4
5
Perfectly
124
Q

A fifth-grade teacher is preparing to launch a unit focused on multiplying and dividing fractions. Which of the following concepts should he include on the pre-unit diagnostic test?

A. FOIL

B. decimal place value

C. PEMDAS

D. finding simplest form

A

D. finding simplest form

Including questions on finding the simplest form of fractions would be most relevant to assess students’ readiness for the upcoming unit on multiplying and dividing fractions. This concept is foundational to understanding fractions and aligns directly with the skills needed to work with fractions effectively. It serves as a good indicator of whether students have grasped basic fraction concepts before moving on to more complex operations like multiplication and division of fractions.

How well did you know this?
1
Not at all
2
3
4
5
Perfectly
125
Q

Phonemic awareness is a strong predictor of future reading success and is, therefore, a key part of literacy instruction. Based on current research, which of the following best describes a successful approach to phonemic awareness instruction for elementary-aged students?

A. Students at all levels of elementary should be receiving explicit instruction on phonemic awareness skills.

B. PreK teachers should expose their young students to phonemic skills, and explicit instruction should be added to exposure activities beginning in kindergarten.

C. Teachers should explicitly teach phonemic awareness to young students, and teachers of older students should continue to directly teach phonemic awareness as needed for students still struggling with the skill.

D. Many students will generally develop phonemic awareness naturally and explicit instruction should not begin until second grade for those who have not developed it on their own.

A

C. Teachers should explicitly teach phonemic awareness to young students, and teachers of older students should continue to directly teach phonemic awareness as needed for students still struggling with the skill.

Phonemic awareness instruction should begin early in a child’s education and continue as needed throughout elementary school. Explicit instruction in phonemic awareness is crucial for young students to develop foundational literacy skills. While some students may develop phonemic awareness naturally, many will benefit from direct instruction to strengthen these skills. Teachers should regularly assess students’ phonemic awareness and provide targeted instruction to support their development.

How well did you know this?
1
Not at all
2
3
4
5
Perfectly
126
Q

A special education teacher is planning a lesson for her first-grade resource ELA class to support the development of phonemic awareness amongst her students. Which of the following students is demonstrating phonemic awareness?

A. a student who is asked to name a rhyming word for sweep, and says heap

B. a student who can identify the number of words in a sentence that is read aloud

C. a student who recognizes that happiness contains 3 syllables

D. a student who can listen to the word sit read aloud and identify that the first sound is /s/

A

D. a student who can listen to the word sit read aloud and identify that the first sound is /s/

Phonemic awareness involves the ability to identify and manipulate individual sounds (phonemes) in spoken words. In this scenario, the student demonstrating phonemic awareness is the one who can listen to the word “sit” read aloud and identify the first sound as /s/. This shows an understanding of the individual phonemes within the word.

How well did you know this?
1
Not at all
2
3
4
5
Perfectly
127
Q

Which of the following activities is most likely to be used to teach reading comprehension?

A. asking students to take turns reading a story out loud

B. asking students to participate in a trivia game about the current read-aloud book

C. asking students to draw a picture of the main character in the class read-aloud book based on how the author describes them

D. asking students to identify difficult words in a story and use them to make an individualized spelling list that they can practice

A

C. asking students to draw a picture of the main character in the class read-aloud book based on how the author describes them.

This activity encourages students to visualize and understand the details of the story, which is a key aspect of reading comprehension. It also helps students engage with the text on a deeper level by connecting visual representations with textual descriptions.

How well did you know this?
1
Not at all
2
3
4
5
Perfectly
128
Q

Mrs. Dubarry has a second-grade student with oppositional defiant disorder (ODD) in her classroom and has been struggling to find effective strategies to help him join in play with peers. To better understand the student and to come up with more effective strategies, with parent permission, Mrs. Dubarry pairs with the school counselor to help brainstorm effective strategies to help the student engage with his classmates. What are some activities the teacher and counselor could practice with the student?

A. activities that reinforce teamwork and cooperation within the classroom

B. activities that reinforce positive self-image and incorporate relaxation techniques

C. activities that focus on developing flexibility and using appropriate language

D. activities that focus on recognizing when to take a break and building emotional resilience

A

A. activities that reinforce teamwork and cooperation within the classroom

How well did you know this?
1
Not at all
2
3
4
5
Perfectly
129
Q

A middle school student who has an orthopedic impairment, impacting his ability to walk and use one arm, will be transferring to a new classroom. In order for his teacher to best prepare the classroom environment she can:

A. make a few minor changes to the environment, but really it is the parents’ responsibility to inform the teacher of what the student will need.

B. prepare the classroom based on what the teacher knows about the incoming student’s needs, with plans to adapt after the student joins the classroom. The teacher creates clear accessways, gathers adapted materials, learns how to use appropriate assistive technology devices, and creates an inclusive environment.

C. observe how the student interacts the first few weeks of school, then make any changes based on observations. It is best to wait to see how successful the student will be so the teacher doesn’t need to make unnecessary changes.

D. do nothing. The student will learn to navigate the environment, just like they would in the real world. The teacher does not need to make changes.

A

B. prepare the classroom based on what the teacher knows about the incoming student’s needs, with plans to adapt after the student joins the classroom. The teacher creates clear accessways, gathers adapted materials, learns how to use appropriate assistive technology devices, and creates an inclusive environment.

Preparing the classroom in advance based on the known needs of the student demonstrates proactive and inclusive teaching practices. This approach ensures that the student can access the learning environment from the start and minimizes potential barriers to their participation. Additionally, it shows a commitment to creating an inclusive and supportive learning environment for all students.

How well did you know this?
1
Not at all
2
3
4
5
Perfectly
130
Q

Public Law 94-142:

A. is a piece of American legislation that ensures students with a disability are provided with a Free Appropriate Public Education that is tailored to their individual needs.

B. prohibits discrimination against people with disabilities in programs that receive federal financial assistance and was a predecessor of the Americans with Disabilities Act.

C. is a 1990 civil rights law that prohibits discrimination based on disability.

D. was signed into law in 1975 and requires all public schools accepting federal funds to provide equal access to education and one free meal a day for children with physical and mental disabilities.

A

A. is a piece of American legislation that ensures students with a disability are provided with a Free Appropriate Public Education that is tailored to their individual needs.

Public Law 94-142, also known as the Individuals with Disabilities Education Act (IDEA), was signed into law in 1975. It mandates that children with disabilities are entitled to a free and appropriate public education (FAPE) in the least restrictive environment possible. This law ensures that students with disabilities receive special education and related services tailored to their individual needs.

How well did you know this?
1
Not at all
2
3
4
5
Perfectly
131
Q

There is an upcoming annual Admission Review Dismissal (ARD) committee meeting for eighth-grade student Navian. Navian struggles with a language processing disorder. He will turn fourteen at the end of the week and attend the meeting for the first time. They will discuss his transition to high school and class schedule for the fall. The meeting will include all other participants except:

A. a special education teacher.

B. a peer of Navian’s choosing.

C. Navian’s parents.

D. a speech-language pathologist.

A

B. a peer of Navian’s choosing.

Typically, an ARD committee meeting for a student with special needs includes key stakeholders such as a special education teacher, the student’s parents, and relevant specialists like a speech-language pathologist. While students are encouraged to participate in their ARD meetings, they typically don’t have the authority to invite peers to the meeting. Therefore, the option that does not belong is “B. a peer of Navian’s choosing.”

How well did you know this?
1
Not at all
2
3
4
5
Perfectly
132
Q

A student who has recently arrived in America after a long immigration process and periods living in refugee camps often gets up and wanders around the room during instruction and continually blurts out answers without waiting to be called on. Which of the following would be the most appropriate course of action for the teacher to take?

A. Make a connection with the student and review classroom procedures and norms with him and confer with the ELL case manager to share information.

B. Call the student’s parents and inform them that his behavior in class has been inappropriate.

C. Refer the student for ADHD testing and provide fidgets to help the student focus during class.

D. Set up a behavior plan with the student with the goal of reducing disruptive behavior.

A

A. Make a connection with the student and review classroom procedures and norms with him and confer with the ELL case manager to share information.

Given the student’s recent immigration and potential exposure to trauma, it’s important for the teacher to approach the situation with empathy and understanding. Building a connection with the student and reviewing classroom procedures and norms can help clarify expectations and provide structure. Additionally, conferring with the ELL case manager can provide insights into the student’s background and potential support strategies. This approach prioritizes building a supportive relationship with the student while addressing any underlying issues that may be contributing to the behavior.

How well did you know this?
1
Not at all
2
3
4
5
Perfectly
133
Q

Mr. Hutchison, a learning support teacher, and Mrs. Weber, a general education teacher, will be co-teaching a fourth-grade social studies course together. Mrs. Weber has already prepared the curriculum mapping for the year, and has the lessons mapped out for the first quarter. What could Mr. Hutchison do to best support Mrs. Weber in their planning and co-teaching?

A. After reviewing his caseload and the needs of the students in the classroom, Mr. Hutchison can prepare instructional strategies and supports that align with Mrs. Weber’s plans, checking in with her to make sure that they are on the same page in regards to planning and instructional strategies.

B. Mr. Hutchison can prepare a series of lessons that coincide with Mrs. Weber’s lessons for the first quarter, so that he can pull out the learning support students and make sure they are getting the support they need in an outside setting.

C. Mr. Hutchison can check in with Mrs. Weber to see what instructional strategies she suggests and make his plans based on what she knows about the students.

D. Mr. Hutchison decides to wait to begin his planning until after observing how Mrs. Weber works with the students. He feels he can best support her as a co-teacher after he gets an idea of the learning environment and what strategies are already in place.

A

A. After reviewing his caseload and the needs of the students in the classroom, Mr. Hutchison can prepare instructional strategies and supports that align with Mrs. Weber’s plans, checking in with her to make sure that they are on the same page in regards to planning and instructional strategies.

Collaboration between Mr. Hutchison and Mrs. Weber is essential for effective co-teaching. By reviewing his caseload and understanding the needs of the students, Mr. Hutchison can tailor instructional strategies and supports that align with Mrs. Weber’s curriculum plans. Regular communication and checking in with Mrs. Weber ensure that both teachers are working towards common goals and utilizing cohesive instructional strategies to support all students in the classroom.

How well did you know this?
1
Not at all
2
3
4
5
Perfectly
134
Q

Each week, Mr. Rivera asks his students to reflect on their reading for the week. They rate the books on a scale of 1-5 and write down one topic that they liked and one topic that they did not like. What can Mr. Rivera do with the information he is collecting?

A. He can use this information to develop reading quizzes.

B. He can use this information to discover students’ hobbies and interests for personalized writing assignments.

C. He can use this information to select future independent reading books for his students.

D. He can use this information to determine whether or not the student understood their reading.

A

C. He can use this information to select future independent reading books for his students.

How well did you know this?
1
Not at all
2
3
4
5
Perfectly
135
Q

Adrian is a 7-year-old boy who receives weekly services from a speech-language pathologist. He has trouble producing speech sounds and struggles with tongue and jaw movement. What is the most likely reason that Adrian is seeing a speech-language pathologist?

A. orthopedic impairment

B. dyscalculia

C. attention deficit hyperactivity disorder

D. apraxia

A

D. apraxia

Apraxia refers to a motor speech disorder where the brain has difficulty coordinating the movements necessary for speech. Adrian’s challenges with tongue and jaw movement and difficulty producing speech sounds suggest symptoms consistent with apraxia. Seeing a speech-language pathologist is common for individuals with apraxia to receive targeted therapy to improve their speech production skills.

How well did you know this?
1
Not at all
2
3
4
5
Perfectly
136
Q

A first-grader with autism spectrum disorder is in a general education classroom all day except for 30 minutes a day when she is pulled out for occupational therapy to work on sensory and motor function. She is above benchmark in her reading skills but lately has been adding in extra details when reading aloud that are not written in or depicted in the book. They are often the same details from book to book and she “reads” them as if they are written on the page. Which of the following would be an appropriate next step to help with this behavior?

A. Ask the special education teacher to informally observe to see if additional services might be appropriate.

B. Call an IEP meeting to discuss the behavior and consider working a reading goal into the plan.

C. Remind her of the difference between reality and make believe and ask her to only read what is on the page.

D. Encourage her to use her finger to follow the words that she is reading when she reads aloud and point out when she is adding details that are not written down.

A

D. Encourage her to use her finger to follow the words that she is reading when she reads aloud and point out when she is adding details that are not written down.

Using her finger to track the words while reading aloud can help the student focus on the text itself and reduce the tendency to add extra details. Pointing out when she adds details that are not written in the book can also help her become more aware of this behavior and learn to distinguish between the text and her own additional interpretations.

How well did you know this?
1
Not at all
2
3
4
5
Perfectly
137
Q

Eduardo is a kindergarten student with a production-related communication disability. He often struggles to explain himself and is unable to retell stories. Which of the following would be the most appropriate intervention to help Eduardo improve his communication skills?

A. setting up a role-playing station with puppets and encouraging students to work together to retell the day’s read-aloud book

B. avoiding calling on Eduardo during whole-group instruction so that he will not feel embarrassed if he struggles to explain himself

C. modifying the writing assignments to allow Eduardo to draw pictures instead of writing

D. pairing Eduardo with a more advanced student in his class during play time to help him develop social skills

A

A. Setting up a role-playing station with puppets and encouraging students to work together to retell the day’s read-aloud book.

Using puppets and role-playing can provide a supportive and interactive environment for Eduardo to practice communication skills in a fun and engaging way. It allows him to act out scenarios, express himself, and retell stories without the pressure of direct questioning or writing tasks. Additionally, working collaboratively with peers provides opportunities for social interaction and peer modeling, which can further support his communication development.

How well did you know this?
1
Not at all
2
3
4
5
Perfectly
138
Q

Jen, a seventh-grade language arts teacher, and Zach, a special education teacher, are working together to co-teach for the first time this year. Each of the following are activities that Jen typically includes in her practice to gauge student comprehension. Which assessment method should Zach encourage her to move away from as it would not be equitable for students with disabilities?

A. silent sustained reading using their independent reading book for an entire 40 minute class period

B. using reading logs that require brief written summaries as a weekly grade to measure progress toward the standard: “range of reading and level of text complexity”

C. an exit ticket that checks for comprehension by asking students to summarize the events of that day’s read-aloud by writing one sentence

D. a partner activity that teaches grammar by having students use colored strips of paper to create silly sentences that include different parts of speech

A

B. using reading logs that require brief written summaries as a weekly grade to measure progress toward the standard: “range of reading and level of text complexity”

How well did you know this?
1
Not at all
2
3
4
5
Perfectly
139
Q

At an annual Admission Review Dismissal (ARD) meeting, the school psychologist referenced the results from a BASC test administered to both teacher and parents. The Behavioral Assessment System for Children (BASC) is a standardized assessment used to monitor emotional and behavioral changes in children. The psychologist can best explain a standardized assessment to the family as:

A. a formal type of assessment that requires all participants to answer the same questions, usually in the same conditions, is widely used, and is scored against a standard and consistent set of data or criterion.

B. a form of assessment that a teacher created to use in the classroom to provide information on how a student is doing.

C. a form of assessment that the school psychologist created and uses when a child is struggling with behavioral issues.

D. a form of assessment that uses both formal and informal data to measure and provide information relating to assessment.

A

A. a formal type of assessment that requires all participants to answer the same questions, usually in the same conditions, is widely used, and is scored against a standard and consistent set of data or criterion.

How well did you know this?
1
Not at all
2
3
4
5
Perfectly
140
Q

Which of the following statements is true of the Every Student Succeeds Act (ESSA)?

A. 1% of students per state may be eligible for alternative assessment.

B. Students must participate in standardized testing annually between grades 3 and 12.

C. Special education teachers must have multiple areas of certification.

D. The federal government will determine supports and consequences for low-performing schools.

A

A. 1% of students per state may be eligible for alternative assessment.

How well did you know this?
1
Not at all
2
3
4
5
Perfectly
141
Q

Mrs. Morgan has been working to teach her kindergarten students alphabetic principles. Monday, she began by describing the sound made by the letter s. Which of the following would be the most logical next step of instruction?

A. Have all students write a list of words that start with the letter “s.”

B. Describe the sounds made by the rest of the letters in the alphabet.

C. Read a book where many words start with the letter “s” and ask students to tally how many times they hear it.

D. Point out examples of “s” in familiar words and names.

A

D. Point out examples of “s” in familiar words and names.

How well did you know this?
1
Not at all
2
3
4
5
Perfectly
142
Q

Nevaeh is a young girl with an IFSP. She will be turning three in four months, and a transition meeting is being held to determine her current educational needs and special education options for the future. When holding this transition meeting, it is important for the service coordinator, Janell, to explain which THREE of the following items?

Select all answers that apply.

A. the different options available to Nevaeh for preschool programs

B. how services will be provided to Nevaeh, from what providers, and at what frequency and duration

C. the transition process and the difference between early intervention and special education services

D. what medical appointments she will need to schedule and make before entering a school program

A

A. the different options available to Nevaeh for preschool programs

B. how services will be provided to Nevaeh, from what providers, and at what frequency and duration

C. the transition process and the difference between early intervention and special education services

How well did you know this?
1
Not at all
2
3
4
5
Perfectly
143
Q

In a fourth-grade inclusion classroom, which of the following would NOT be appropriate for helping students transition from snack time back into academic time?

A. Create an individualized incentive system that allows students to get a point toward a reward if they are cleaned up and ready for learning at the end of snack time.

B. At the end of snack time, set a timer and challenge students to try to clean up as quickly as they can.

C. Create an anchor chart with three expectations for what it looks like to be ready for learning time (eg. food away, in your seat, book ready).

D. Provide five-minute and two-minute warnings before the end of snack time to help students prepare to transition.

A

B. At the end of snack time, set a timer and challenge students to try to clean up as quickly as they can.

How well did you know this?
1
Not at all
2
3
4
5
Perfectly
144
Q

Which of the following behaviors can likely be addressed by a functional behavior assessment (FBA)?

A. Sean has difficulty interpreting social cues and engaging in conversation with peers.

B. Sean has difficulty solving math problems without the use of a calculator.

C. Sean breaks pencils, tears his paper, and leaves without permission the classroom during science class.

D. Sean is frequently absent from class due to hospitalizations and is falling behind in school work.

A

C. Sean breaks pencils, tears his paper, and leaves without permission the classroom during science class.

How well did you know this?
1
Not at all
2
3
4
5
Perfectly
145
Q

Which of the following scenarios would suggest that a student needs additional instruction or remediation in phonemic awareness?

A. A first-grader is frequently leaving vowels out of words (eg. writing “cat” as “ct.”).

B. A fourth-grader is writing letters in the incorrect order when spelling a word (eg. spelling “does” as “odes”).

C. A kindergartener is writing letters backwards (eg. writing “b” instead of “d”).

D. A second-grader is struggling to decode unfamiliar words when reading aloud.

A

A. A first-grader is frequently leaving vowels out of words (eg. writing “cat” as “ct.”).

How well did you know this?
1
Not at all
2
3
4
5
Perfectly
146
Q

Which of the following is NOT a required component of an IEP?

A. measurable annual goals, both academic and functional

B. a list of accommodations to be provided during assessments

C. the most recent academic grades for the student

D. a schedule of all services including start date, location, duration, and frequency

A

C. the most recent academic grades for the student

How well did you know this?
1
Not at all
2
3
4
5
Perfectly
147
Q

Which of the following activities to build understanding of the alphabetic principle would be the most advanced?

A. as a whole group, students list words that all begin with the same phoneme

B. as a whole group, the class lists the syllables in words

C. in independent small groups, students play a memory game in which they need to match letters with an image of something that starts with that letter

D. in independent small groups, students use block manipulatives with letters and graphemes on each of the sides to make and sound out new words

A

D. in independent small groups, students use block manipulatives with letters and graphemes on each of the sides to make and sound out new words

How well did you know this?
1
Not at all
2
3
4
5
Perfectly
148
Q

A fourth-grade student who has attention-deficit/hyperactivity disorder (ADHD) has a testing accommodation of extra time. What could be the reasoning behind this accommodation?

A. The student frequently asks for breaks during testing to go to the bathroom or get a drink of water to refocus, and extra time allows the student to take the breaks without being penalized for time running out during a test.

B. All students with ADHD need extra time as a testing accommodation.

C. The student has to be frequently reminded to stay on task, and extra time allows the teacher to talk to the student more often during the test to give the reminders to stay on task.

D. The student is often off task and requires an extended amount of time to complete work and tests.

A

A. The student frequently asks for breaks during testing to go to the bathroom or get a drink of water to refocus, and extra time allows the student to take the breaks without being penalized for time running out during a test.

How well did you know this?
1
Not at all
2
3
4
5
Perfectly
149
Q

Jayce is a first-grade student struggling with comparing two-digit numbers. Which two of the following manipulatives could his teacher use to provide support for Jayce?

Select all answers that apply.

A. Unifix cubes

B. counters

C. base ten blocks

D. Cuisenaire rods

A

A. Unifix cubes
C. base ten blocks

How well did you know this?
1
Not at all
2
3
4
5
Perfectly
150
Q

Abby is a high school student with a learning disability. She is enrolled in a general education English class with Mrs. Yuhn. Her special education teacher, Mr. Scotty, wants to ensure a successful year for Abby. What is one way that Mr. Scotty could create effective collaboration with Mrs. Yuhn to best support his student?

A. Mr. Scotty can check in with Abby to see how she feels about everything, but doesn’t need to worry about speaking with Mrs. Yuhn directly.

B. Mr. Scotty believes it is Mrs. Yuhn’s responsibility to ask for help if she needs it.

C. Mr. Scotty can send out a monthly newsletter about Abby’s progress towards her IEP goals to all of Abby’s teachers, including Mrs. Yuhn.

D. Mr. Scotty can make sure that Mrs. Yuhn understands Abby’s learning goals and needs at the start of the year, and regularly partner with her throughout it to best support Abby.

A

D. Mr. Scotty can make sure that Mrs. Yuhn understands Abby’s learning goals and needs at the start of the year, and regularly partner with her throughout it to best support Abby.

How well did you know this?
1
Not at all
2
3
4
5
Perfectly
151
Q

Mr. Parsons will be teaching a special education class composed of a small group of students who attend an alternative school for part of the school day. Mr. Parsons is planning on meeting with the teacher at the alternative school to help address his students’ needs and make sure that they have a successful year. What would be the best way for Mr. Parsons to prepare for the meeting?

A. Mr. Parsons should call the alternative school teacher to discuss instructional plans and strategies for each student and get an idea of the teacher’s instructional methods.

B. Mr. Parsons should prepare a list of common instructional strategies and tactics that the alternative school teacher could use throughout the year.

C. Mr. Parsons should meet with his students’ past teachers to discuss what instructional methods work best for his students.

D. Mr. Parsons should review all of his students’ IEPs so that he is aware of their current levels of performance, goals, and progress towards their outcomes to help determine instructional goals.

A

D. Mr. Parsons should review all of his students’ IEPs so that he is aware of their current levels of performance, goals, and progress towards their outcomes to help determine instructional goals.

How well did you know this?
1
Not at all
2
3
4
5
Perfectly
152
Q

A first-grade student is struggling to decode words containing multiple syllables. While investigating what could be causing the problem, the teacher notices that she is able to read and spell single-syllable words accurately most of the time. What skill should the teacher revisit to help improve this student’s decoding skills?

A. phoneme recognition

B. structural analysis

C. blending and segmenting phonemes

D. rhyme recognition

A

B. structural analysis

How well did you know this?
1
Not at all
2
3
4
5
Perfectly
153
Q

The Every Student Succeeds Act (ESSA) specifies that no more than 1 percent of all students should take alternate assessments not aligned to the academic standards of their grade level. For which of the following students would an alternate assessment be most appropriate?

A. a student with cerebral palsy who receives instruction in general-education classrooms but lacks the fine motor control to bubble in responses on an answer document

B. a student with autism spectrum disorder (ASD) who communicates verbally and receives instruction in general-education classrooms with paraprofessional support

C. a student with a severe intellectual disability who receives all instruction in a self-contained classroom

D. a student with an emotional disturbance (ED) who requires frequent breaks during tests and difficult assignments and constantly tries to engage other students in conversation

A

C. a student with a severe intellectual disability who receives all instruction in a self-contained classroom

How well did you know this?
1
Not at all
2
3
4
5
Perfectly
154
Q

Which of the following are the most important considerations when drafting IEP goals?

A. Making sure the student has the same goal from year to year to promote consistency.

B. Trying to write similar goals for groups of students who are in the same classes so it will be easier for the general education teachers to assess and report progress.

C. Writing goals that will be easily attainable to build student confidence.

D. Identifying data-supported weaknesses the student has displayed and aligning goals to curriculum standards that will allow the student to practice and improve on their deficiency at the appropriate rigor.

A

D. Identifying data-supported weaknesses the student has displayed and aligning goals to curriculum standards that will allow the student to practice and improve on their deficiency at the appropriate rigor.

This option aligns most closely with best practices in drafting Individualized Education Program (IEP) goals. It emphasizes the importance of basing goals on specific, measurable data about the student’s needs and focusing on areas where the student requires support and improvement. Goals should be tailored to the individual student’s abilities and challenges and should align with curriculum standards to ensure they are meaningful and relevant to the student’s educational progress.

How well did you know this?
1
Not at all
2
3
4
5
Perfectly
155
Q

Which is the most appropriate section of a student’s IEP to include information about the student’s post-secondary goals as well as activities and services that the student can access now to prepare for post-secondary education and/or employment?

A. related services

B. least restrictive environment

C. accommodations and modifications

D. transition plan

A

D. transition plan

The transition plan section of a student’s Individualized Education Program (IEP) is the most appropriate place to include information about the student’s post-secondary goals as well as activities and services to prepare for post-secondary education and/or employment. This section specifically focuses on planning for the student’s transition from school to adult life and typically includes goals, activities, and services related to post-secondary education, vocational training, employment, independent living skills, and community involvement.

How well did you know this?
1
Not at all
2
3
4
5
Perfectly
156
Q

Of the following, which parental concern is most likely to result in an evaluation of a child under the age of three?

A. failure to reach national norms of height/weight ratio

B. failure to recognize and identify letters of the alphabet

C. failure to meet multiple milestones of adaptive development

D. failure to complete toilet training

A

C. failure to meet multiple milestones of adaptive development

Concerns about a child’s failure to meet multiple milestones of adaptive development are most likely to result in an evaluation of a child under the age of three. Adaptive development involves skills related to daily living activities such as feeding, dressing, and interacting with others. Delays in adaptive development can be indicative of developmental delays or disabilities and may prompt early intervention services to support the child’s development. Early intervention services are often provided for children under the age of three who are identified as having developmental delays or disabilities, so concerns related to adaptive development are particularly relevant in this context.

How well did you know this?
1
Not at all
2
3
4
5
Perfectly
157
Q

John is an incoming second-grade student with autism spectrum disorder (ASD) who is enrolled in a new district and school. John’s parents have expressed concerns about his transition to the new campus. Which of the following would be the best way to help John prepare for the new school year?

A. Schedule a phone call between John’s family and his new teacher so they can get to know each other.

B. Send a visual schedule of a typical school day for second-grade students so John can prepare for his new environment.

C. Mail John’s parents a map of the campus so they can review the layout and location of his classroom before school starts.

D. Offer to provide a private tour of the campus during which John and his parents can meet his teachers and the administrative staff.

A

D. Offer to provide a private tour of the campus during which John and his parents can meet his teachers and the administrative staff.

For a student with autism spectrum disorder (ASD) like John, a private tour of the campus would likely be the most beneficial option to help him prepare for the new school year. This allows John and his parents to familiarize themselves with the physical layout of the school, including locating his classroom, the cafeteria, restrooms, and any other important areas. Meeting his teachers and administrative staff in person during the tour can help John feel more comfortable and ease his transition. Additionally, it provides an opportunity for John’s parents to ask questions and share information about John’s needs and preferences, fostering a collaborative relationship with the school staff from the beginning.

How well did you know this?
1
Not at all
2
3
4
5
Perfectly
158
Q

Which of the following is an appropriate use of a behavioral redirection placement?

A. Carol, a seventh-grade student with attention-deficit/hyperactivity disorder (ADHD), is sent to the redirection room when she is too disruptive with inappropriate comments during whole-group instruction.

B. Maverick, an eighth-grade student with an intellectual disability is allowed to visit the redirection room when he gets frustrated by the tasks he’s expected to complete in his math resource class.

C. Marisol, a sixth-grade student with an emotional disturbance, is allowed to visit the redirection room when she gets overwhelmed by anxiety due to the amount of activity and noise in her general-education classes.

D. Alex, a fifth-grade student with autism spectrum disorder (ASD) is sent to the redirection room when he finishes his assignments early because he is restless and distracting when he is not occupied.

A

C. Marisol, a sixth-grade student with an emotional disturbance, is allowed to visit the redirection room when she gets overwhelmed by anxiety due to the amount of activity and noise in her general-education classes.

This option describes an appropriate use of a behavioral redirection placement. The redirection room serves as a supportive environment where Marisol can go when she becomes overwhelmed by anxiety in her general-education classes. It provides her with a quieter, calmer space where she can regulate her emotions and regain control before returning to her classroom. This use of the redirection room aligns with the goal of providing appropriate support for students with emotional disturbances to help them manage their emotions and succeed in the classroom.

How well did you know this?
1
Not at all
2
3
4
5
Perfectly
159
Q

Which of the following questions should take precedent when determining the least restrictive environment for any student during the initial referral process for special education services?

A. Which instructional arrangement offers the greatest amount of support for students with disabilities?

B. Which instructional arrangement will provide the best opportunity for the student to demonstrate her strengths while receiving support for her weaknesses?

C. Which instructional arrangement will give the student the easiest route to annual promotion and eventual graduation?

D. Which instructional arrangement has the lowest student-to-teacher ratio and can therefore provide the greatest possible support?

A

B. Which instructional arrangement will provide the best opportunity for the student to demonstrate her strengths while receiving support for her weaknesses?

How well did you know this?
1
Not at all
2
3
4
5
Perfectly
160
Q

Samuel is a third-grade student who is attentive and hardworking in class. He has good grades in math, science, and social studies, but struggles in reading. He has poor fluency, struggles to decode unfamiliar words, takes longer than other students to copy notes from the board, and struggles to answer questions that require a written response. Which of the following disabilities is Samuel most likely struggling with?

A. attention-deficit hyperactivity disorder (ADHD)

B. intellectual disability

C. dyslexia

D. autism spectrum disorder (ASD)

A

C. dyslexia

How well did you know this?
1
Not at all
2
3
4
5
Perfectly
161
Q

Charles is a high school junior with autism spectrum disorder (ASD). He performs above grade level in all core subjects, but struggles with decision-making in non-school settings and suffers extreme anxiety when dealing with the unexpected. During transition planning prior to his annual IEP meeting, Charles has expressed interest in web development as a career, even though he has not gained any real-world training or experience in this pursuit. Which of the following resources would be most helpful for a member of the admission, review, and dismissal (ARD) committee to provide to Charles and his family?

A. enrollment in a seminar teaching attendees how to start and grow their own web-development business

B. a list of colleges and universities that offer online degrees in website development

C. a list of junior college campuses within a 60-mile radius of Charles’s home, so he can attend college and still have the support of living at home

D. provide a wireless hotspot and laptop for Charles’s home so he can use district-managed technology to pursue his interests.

A

B. a list of colleges and universities that offer online degrees in website development

Based on the available information, Charles has a career goal that could be aided by continuing his education after high school. Additionally, an online degree program would allow Charles to access the post-secondary curriculum without the added stress and anxiety of leaving home and living independently before he is ready to do so.

How well did you know this?
1
Not at all
2
3
4
5
Perfectly
162
Q

During a parent-teacher conference, the father of a non-disabled student in an inclusion class mentions that his child has had friction in the past with another student in the class who has an emotional disturbance. The father mentions several disagreements that the two have had and then asks if any progress has been made in the disabled student’s classroom behavior. How should the teacher respond?

A. Share relevant portions of the disabled student’s IEP to show that all necessary steps are being taken to provide a safe and comfortable educational environment for all.

B. Thank the father for his time and his concerns and firmly but politely indicate that you are only allowed to discuss his own child during the parent-teacher conference.

C. Assure the father that the disabled student in question will be kept away from his child to the maximum extent possible.

D. Remind the father that the student in question has a behavioral disability and ask that he encourage his own child to be more patient.

A

B. Thank the father for his time and his concerns and firmly but politely indicate that you are only allowed to discuss his own child during the parent-teacher conference.

How well did you know this?
1
Not at all
2
3
4
5
Perfectly
163
Q

What is the primary benefit of having students paraphrase or summarize a text after they’ve read it?

A. It increases metacognition and is the purest way for a student to prove he or she understands the text.

B. It improves fluency by requiring students to prove that they have read the passage when they write their summaries.

C. It forces students to slow down while reading and pay more attention to unfamiliar vocabulary.

D. It helps students connect the new text to other passages they’ve read, solidifying the new information in their memory.

A

A. It increases metacognition and is the purest way for a student to prove he or she understands the text.

Paraphrasing or summarizing a text requires students to engage with the material at a deeper level, as they must not only comprehend the content but also rephrase it in their own words. This process enhances metacognition, which is the awareness and understanding of one’s own thought processes. By summarizing or paraphrasing, students must reflect on what they have read, identify the main ideas, and articulate them in a coherent manner. This active engagement fosters a deeper understanding of the text and allows students to demonstrate their comprehension in a meaningful way.

How well did you know this?
1
Not at all
2
3
4
5
Perfectly
164
Q

Which of the following difficulties displayed by a third-grade student with a specific learning disability (SLD) in math calculation could be supported by allowing the student to complete his work on graph paper?

A. neglecting to complete all steps of a calculation

B. incorrectly calculating basic math facts

C. performing the wrong operation

D. keeping place value consistent when performing calculations

A

D. keeping place value consistent when performing calculations

Allowing the student to complete his work on graph paper can support difficulties related to keeping place value consistent when performing calculations. Graph paper provides a structured layout with clearly defined squares, which can assist the student in aligning numbers properly and maintaining place value. The grid lines can help the student organize digits in columns, making it easier to perform multi-digit calculations accurately. This accommodation can help mitigate difficulties related to spatial organization and place value understanding, which are common challenges for students with specific learning disabilities in math calculation.

How well did you know this?
1
Not at all
2
3
4
5
Perfectly
165
Q

Which landmark special education court decision held that school districts may be required to reimburse parents of students with special needs for the costs of private education in certain situations?

A. Burlington School Committee v. Massachusetts Board of Education

B. Irving Independent School District v. Amber Tatro

C. Honig v. Doe

D. Schaffer v. West

A

A. Burlington School Committee v. Massachusetts Board of Education

Burlington School Committee v. Massachusetts Board of Education is the landmark special education court decision that held that school districts may be required to reimburse parents of students with special needs for the costs of private education in certain situations. This case established the principle of “tuition reimbursement,” which allows parents to seek reimbursement for the cost of private education if the school district fails to provide a free appropriate public education (FAPE) to their child and the private placement is determined to be appropriate.

How well did you know this?
1
Not at all
2
3
4
5
Perfectly
166
Q

All of the following are benefits of using pre-planned visual cues to remind a student of available behavior supports EXCEPT:

A. it reminds the student that she is in control of her choices and that she has options for managing her emotions and behavior.

B. it can help the student recognize that she is engaging in behavior that may result in consequences.

C. it allows the teacher to give notice that behavior needs correction without calling the student out in front of the class.

D. it allows the teacher to remove the student from class if the behavior continues because the visual cue counts as a correction.

A

D. it allows the teacher to remove the student from class if the behavior continues because the visual cue counts as a correction.

How well did you know this?
1
Not at all
2
3
4
5
Perfectly
167
Q

A teacher is planning a formative assessment to determine how well his students can differentiate between the concepts of area and volume. Which of the following formative assessments would be the most appropriate for this topic?

A. sorting drawings of shapes into area and volume categories

B. writing a short essay describing areas and volumes in daily life

C. defining area and volume

D. calculating the areas and volumes of figures

A

A. sorting drawings of shapes into area and volume categories

How well did you know this?
1
Not at all
2
3
4
5
Perfectly
168
Q

Callie is a 4-year-old who has been diagnosed with autism spectrum disorder (ASD) by a private diagnostician at her parents’ expense. Callie has not previously been enrolled in preschool or daycare because both her parents work from home and have provided care themselves. After receiving the diagnosis, Callie’s parents want to have her enrolled in the Preschool Programs for Children with Disabilities (PPCD) to help prepare for her transition to public school. Which of the following is the logical next step in this process?

A. Callie can be enrolled immediately based on the recommendation of her licensed private diagnostician.

B. Callie is not eligible for PPCD because her parents are able to provide care at home.

C. Callie must be enrolled in a private daycare or preschool to allow for a complete evaluation in a school-like environment before determining her eligibility for PPCD.

D. Callie needs a full individual evaluation (FIE) to determine if she is eligible for PPCD before being enrolled.

A

D. Callie needs a full individual evaluation (FIE) to determine if she is eligible for PPCD before being enrolled.

How well did you know this?
1
Not at all
2
3
4
5
Perfectly
169
Q

Mr. Jackson is a seventh-grade science teacher who teaches multiple inclusion classes. Before beginning a unit of study on cell structure and function, Mr. Jackson instructs his students to combine their knowledge and complete a chart detailing what they already know and questions they have about specific cell organelles. The primary benefit of this type of frontloading activity is that:

A. it shows students that they can use one another as a resource when they don’t understand something.

B. it allows students to work collaboratively to develop original classroom materials.

C. it lets Mr. Jackson know which parts of the unit he can skip over.

D. it stimulates background knowledge and gives Mr. Jackson insight into the level of understanding his students possess.

A

D. it stimulates background knowledge and gives Mr. Jackson insight into the level of understanding his students possess.

How well did you know this?
1
Not at all
2
3
4
5
Perfectly
170
Q

A special education teacher in a first-grade inclusion class uses index cards to label places and objects in the room, like doors, windows, shelves, and desks. Which of the following concepts is the teacher most likely trying to reinforce?

A. consonant blends

B. the idea that words (symbols) represent real-world objects

C. phonemic awareness

D. parts of speech

A

B. the idea that words (symbols) represent real-world objects

How well did you know this?
1
Not at all
2
3
4
5
Perfectly
171
Q

Alfredo is a fourth-grade student with a specific learning disability (SLD) in reading comprehension. During a small-group activity that incorporates oral reading, Alfredo’s special education teacher notes that while he correctly pronounces most grade-level words, he reads in a robotic, monotonal voice and that his pauses and pacing do not match the punctuation of the text. Which of the following would be the best way for Alfredo’s teacher to help him improve his reading comprehension during future small-group work?

A. Give Alfredo the option of taking his turn to read or following along silently while others read.

B. Utilize dramas with stage directions that cue emotional affect and break the speaking parts into short lines among multiple characters.

C. Offer the choice of several high-interest texts and let Alfredo pick which passages the group will read.

D. Allow Alfredo to pre-read the text selections, so he will feel more prepared during small-group work.

A

B. Utilize dramas with stage directions that cue emotional affect and break the speaking parts into short lines among multiple characters.

How well did you know this?
1
Not at all
2
3
4
5
Perfectly
172
Q

Which of the following functional goals would most likely be found in the IEP of a student on the autism spectrum who receives speech therapy once per week?

A. The student will wait for his turn without complaint in 3 out of 5 tries while participating in a preferred activity with a teacher and peer.

B. The student will complete 10 assisted (modified resting position) pushups without stopping.

C. The student will button and unbutton a dress shirt without missing any buttons in 7 out of 10 attempts.

D. The student will understand new vocabulary and use it correctly when speaking and writing.

A

A. The student will wait for his turn without complaint in 3 out of 5 tries while participating in a preferred activity with a teacher and peer.

How well did you know this?
1
Not at all
2
3
4
5
Perfectly
173
Q

Students in Mrs. Wilson’s class have mastered multiplication and have been introduced to division. Mrs. Wilson gave a test over introductory concepts in division and found that a number of students struggled. Which of the following strategies is best to help improve the students’ understanding of division?

A. Use manipulatives to model division and connect it to multiplication.

B. Teach that multiplication and division are opposites, and have students memorize times tables to make division easier.

C. Provide a new study guide with division problems and give a new test again in two days.

D. Separate the class into groups and have at least one student that understands division in each group. The higher-level students can reteach the struggling students the concept through peer tutoring.

A

A. Use manipulatives to model division and connect it to multiplication.

How well did you know this?
1
Not at all
2
3
4
5
Perfectly
174
Q

Ms. Echeveria is a special education case manager for a group of fifth-grade students. A social studies teacher who works with several of the students on Ms. Echeveria’s caseload is absent due to the flu for several consecutive days. Ms. Echeveria is concerned that the substitute teachers may not understand how to provide necessary supports without explicit instructions, including IEP information specific to each student’s qualifying disability. Which of the following would be the best way for her to approach the situation?

A. Provide relevant sections of each student’s IEP to any substitute who will work with the class and make sure they understand how to implement appropriate accommodations.

B. Ms. Echeveria should take no action because the contents of student IEPs are confidential information not to be shared with non-certified district personnel.

C. Ms. Echeveria should pull all students with an IEP during social studies class and provide instruction herself until the general-education teacher returns.

D. Assign a paraprofessional to provide support to the substitutes until the general-education teacher returns.

A

A. Provide relevant sections of each student’s IEP to any substitute who will work with the class and make sure they understand how to implement appropriate accommodations.

How well did you know this?
1
Not at all
2
3
4
5
Perfectly
175
Q

If a student with epilepsy suffers a grand mal seizure during class, the best thing the teacher can do is:

A. position the affected student on the floor with support for his head and neck and send a classmate to the nearest adult to get help.

B. put something hard between the student’s teeth so he doesn’t swallow his tongue.

C. immediately go get the school nurse.

D. hold the student firmly with both arms wrapped around his chest so his spasms are minimized.

A

A. position the affected student on the floor with support for his head and neck and send a classmate to the nearest adult to get help.

How well did you know this?
1
Not at all
2
3
4
5
Perfectly
176
Q

A kindergarten teacher is planning a unit on financial literacy. What are some elements that the teacher should plan to include in this unit?

A. money models (pretend money), children’s books, guest speakers

B. at-home projects, independent work, money-counting activities

C. posters, worksheets, videos

D. student-created savings plans, class reward system, show-and-tell

A

A. money models (pretend money), children’s books, guest speakers

How well did you know this?
1
Not at all
2
3
4
5
Perfectly
177
Q

Which of the following would be the most appropriate use of wall space for a sixth-grade inclusion classroom consisting of multiple students with various disabilities?

A. Post pictures of the teachers and their families on the front wall to encourage students to get to know their instructors better.

B. Keep the space around the primary focal point of the room (smartboard/whiteboard) mostly clear and utilize space on the side walls toward the rear for anchor charts that reinforce key concepts and processes.

C. Post the best examples of student-created projects on the front wall as motivation for students to put forth their best efforts.

D. Keep all walls as bare as possible to limit potential distractions.

A

B. Keep the space around the primary focal point of the room (smartboard/whiteboard) mostly clear and utilize space on the side walls toward the rear for anchor charts that reinforce key concepts and processes.

How well did you know this?
1
Not at all
2
3
4
5
Perfectly
178
Q

Marvin, a sixth-grade student with a specific learning disability (SLD) in reading fluency also receives speech therapy for a slight stutter and is hesitant to read aloud during class. Which of the following strategies would best help Marvin gain confidence for oral reading and allow him to work on his fluency?

A. Provide Marvin with a preview of any text to be read aloud in class so he can practice the night before. Then, encourage him to volunteer to read as much or as little of the text as he’s comfortable with.

B. Provide a blank notecard as a placeholder, so Marvin won’t get lost during whole-group oral reading.

C. Allow Marvin to choose a seat near friends in class so that he feels supported and comfortable.

D. Excuse Marvin from any whole-group oral reading activities to avoid making him uncomfortable.

A

A. Provide Marvin with a preview of any text to be read aloud in class so he can practice the night before. Then, encourage him to volunteer to read as much or as little of the text as he’s comfortable with.

How well did you know this?
1
Not at all
2
3
4
5
Perfectly
179
Q

Which of the following would be an appropriate method of assessment when reporting progress on an IEP goal that measures a student’s ability to understand and use newly acquired vocabulary?

A. Ask the student to choose from a word bank the vocabulary word that best fits the blanks in a series of sentences.

B. Ask the student to identify the meanings of various prefixes and suffixes and explain how they change the meanings of the words to which they are affixed.

C. Give the student a matching activity that requires them to connect vocabulary words to their definitions.

D. Ask the student to provide two synonyms and two antonyms for each vocabulary word.

A

A. Ask the student to choose from a word bank the vocabulary word that best fits the blanks in a series of sentences.

How well did you know this?
1
Not at all
2
3
4
5
Perfectly
180
Q

Mr. Sanders, a special education teacher in a language arts inclusion classroom, is working to improve the reading comprehension of several students in his class. Which of the following strategies is Mr. Sanders most likely to use?

A. emphasizing the use of context clues to help decode text

B. diagramming sentences from the text

C. increasing the rate at which the students read

D. identifying figurative language when reading

A

A. emphasizing the use of context clues to help decode text

How well did you know this?
1
Not at all
2
3
4
5
Perfectly
181
Q

Which of the following supports would be most beneficial in helping a fifth-grade student who is deaf participate in an activity that requires students to listen to selected song lyrics to identify and decode examples of figurative language?

A. Provide a transcript of the lyrics with multiple choice options to help identify the figurative language in each song.

B. Turn on the video’s subtitles so the student can read along with the songs.

C. Have a deaf education specialist deliver the lyrics using sign language.

D. Plan an alternative activity for the student.

A

A. Provide a transcript of the lyrics with multiple choice options to help identify the figurative language in each song.

How well did you know this?
1
Not at all
2
3
4
5
Perfectly
182
Q

Which of the following supports would be most beneficial in helping a fifth-grade student who is deaf participate in an activity that requires students to listen to selected song lyrics to identify and decode examples of figurative language?

A. Provide a transcript of the lyrics with multiple choice options to help identify the figurative language in each song.

B. Turn on the video’s subtitles so the student can read along with the songs.

C. Have a deaf education specialist deliver the lyrics using sign language.

D. Plan an alternative activity for the student.

A

A. Provide a transcript of the lyrics with multiple choice options to help identify the figurative language in each song.

How well did you know this?
1
Not at all
2
3
4
5
Perfectly
183
Q

Malcolm is a third-grade student with a mild intellectual disability. He reads with fluency that is only slightly below grade level but struggles to demonstrate comprehension of texts. Malcolm is significantly below grade level in written expression and often guesses at answers when given multiple-choice questions. Which of the following alternative means of assessment could best help Malcolm’s teachers establish a baseline for his reading comprehension level?

A. Have Malcolm retell the story to a peer partner after reading.

B. Allow Malcolm to draw pictures representing important elements of the texts he reads.

C. Provide Malcolm with lower-level texts to build his confidence.

D. Have Malcolm write a summary immediately after reading a text, while it is still fresh in his mind.

A

B. Allow Malcolm to draw pictures representing important elements of the texts he reads.

How well did you know this?
1
Not at all
2
3
4
5
Perfectly
184
Q

Which of the following would likely be most important for a sixth-grade English language arts teacher to keep in mind when planning instruction for a class that includes a student with muscular dystrophy?

A. Make sure all lessons and assignments allow for the use of text-to-speech software.

B. Be prepared to assign homework when in-class assignments are not completed.

C. Build in breaks during written assignments to allow for muscle fatigue and recovery.

D. Arrange for the student to receive audio versions of all class novels, since reading for extended periods is likely to be tiring.

A

C. Build in breaks during written assignments to allow for muscle fatigue and recovery.

How well did you know this?
1
Not at all
2
3
4
5
Perfectly
185
Q

Special education teacher Mr. Brown has been invited to observe Amanda, a sixth-grade student with attention deficit hyperactivity disorder (ADHD), who is on his caseload. Amanda’s science teacher reports that Amanda and several other children are disruptive and unruly during whole-group instruction and asks for Mr. Brown’s help in changing the behavior. The first thing Mr. Brown notices is that the front wall of the classroom is covered with vibrant, colorful, motivational posters, as well as student work. Additionally, when the teacher is delivering direct instruction, she positions herself at the front of the room near the smartboard and rarely moves. What advice should Mr. Brown offer after his observation?

A. The science teacher should rearrange her room so that the desks all face the side wall.

B. The science teacher should remove some of the clutter that decorates the front wall of the room and try to move around more as she delivers instruction.

C. The science teacher should make sure to always have plenty of tasks at the ready to keep her students busy.

D. The science teacher should change her seating chart to separate Amanda and some of the other students from one another.

A

B. The science teacher should remove some of the clutter that decorates the front wall of the room and try to move around more as she delivers instruction.

How well did you know this?
1
Not at all
2
3
4
5
Perfectly
186
Q

Which of the following could be considered assistive technology for a physical education class that includes several students with disabilities that impair their movement and force generation?

A. letting students walk when the curriculum calls for running in an activity

B. lowering the basketball hoop

C. using a beep-ball for kickball

D. using the competition scoreboard to track completed exercises

A

B. lowering the basketball hoop

How well did you know this?
1
Not at all
2
3
4
5
Perfectly
187
Q

According to the Individuals with Disabilities Education Act (IDEA), IEP goals must meet all of the following criteria except:

A. IEP goals must include a standard of performance against which progress will be measured.

B. IEP goals must be applicable to a trade or vocation.

C. IEP goals must include a clear statement of when progress will be reported.

D. IEP goals must be measurable.

A

B. IEP goals must be applicable to a trade or vocation.

How well did you know this?
1
Not at all
2
3
4
5
Perfectly
188
Q

Who should be interviewed when collecting data for a functional behavior assessment (FBA)?

A. only the student’s special education teachers, as they should be well-versed in behavior analysis

B. the parents of the student in question

C. multiple people who have observed the target behavior in a variety of settings and conditions

D. the individual who wrote the most recent discipline referral related to the target behavior

A

C. multiple people who have observed the target behavior in a variety of settings and conditions

How well did you know this?
1
Not at all
2
3
4
5
Perfectly
189
Q

Bryce is a student with autism spectrum disorder (ASD) who performs at grade level in math but whose reading ability is multiple grade levels below his current fifth-grade placement. Bryce has received instruction in a co-taught classroom for the last four years. At his annual IEP meeting, in preparation for his transition to middle school, his special education case manager recommends that Bryce be placed in a resource class for language arts to accommodate his lower reading level. His parents argue against this placement, saying that they don’t want Bryce to feel singled out. Which of the following is the most appropriate response to this concern?

A. Emphasize to Bryce’s parents that their input will always be a valuable component to any decisions regarding their child’s education and explain how the lower student-to-teacher ratio in resource classes allows for more personalized instruction and support.

B. Have the office staff call Bryce out of class to participate in the meeting and get his input regarding next year’s placement.

C. Show the parents Bryce’s language arts assessment data and compare it to the performance of general education students who read at grade level.

D. Table the discussion and suspend the IEP meeting so that both sides can gather data to support their opinions, then schedule a mediation to determine Bryce’s placement.

A

A. Emphasize to Bryce’s parents that their input will always be a valuable component to any decisions regarding their child’s education and explain how the lower student-to-teacher ratio in resource classes allows for more personalized instruction and support.

How well did you know this?
1
Not at all
2
3
4
5
Perfectly
190
Q

Which of the following is the best activity for reviewing percentages with fifth-grade students?

A. coloring in 100-blocks to represent percent

B. using a variety of methods and scenarios to determine percentage

C. writing percentages from decimal or fraction conversions

D. comparing percentages from their test scores throughout the year

A

B. using a variety of methods and scenarios to determine percentage

How well did you know this?
1
Not at all
2
3
4
5
Perfectly
191
Q

Which of the following are the best ways for a teacher to help students develop independent reading skills?

Select all answers that apply.

A. Pick all student reading for them to be sure it is a good fit.

B. Provide in-class time for students to choose independent reading books and some time for quiet reading.

C. Facilitate peer conversations about their reading.

D. Only allow students to choose their independent reading books from teacher-selected lists.

A

B. Provide in-class time for students to choose independent reading books and some time for quiet reading.

C. Facilitate peer conversations about their reading.

How well did you know this?
1
Not at all
2
3
4
5
Perfectly
192
Q

Which of the following accommodations would NOT be helpful for a medically fragile sixth-grade student who struggles with mobility and gross motor control?

A. adaptive physical education

B. paraprofessional support during transitions

C. transcription support

D. large-print textbooks and assignments

A

D. large-print textbooks and assignments

How well did you know this?
1
Not at all
2
3
4
5
Perfectly
193
Q

When helping a general education teacher plan instruction for a student with a specific learning disability, the first thing the special education teacher is likely to share is:

A. the student’s testing accommodations in the IEP.

B. the student’s most recent state assessment data.

C. the relevant sections of the student’s present levels of academic achievement and functional performance (PLAAFP) in the IEP.

D. the student’s most recent testing data from the last full individual evaluation (FIE).

A

C. the relevant sections of the student’s present levels of academic achievement and functional performance (PLAAFP) in the IEP.

How well did you know this?
1
Not at all
2
3
4
5
Perfectly
194
Q

Brandi, a seventh-grade student with autism spectrum disorder (ASD) has violated the district code of student conduct, and the campus discipline matrix stipulates that she will be placed in the district’s disciplinary alternative education program for a minimum of 30 days. Since this placement would take Brandi out of the general education setting for more than 10 consecutive days, the admission, review, and dismissal (ARD) committee must hold a manifestation determination review (MDR). The purpose of an MDR is to prevent:

A. students from being punished for behaviors that are a direct result of their disability.

B. students with disabilities from being disciplined in the same manner as non-disabled peers.

C. students with disabilities from being removed from the least restrictive environment.

D. students with disabilities from having discipline referrals added to their permanent records.

A

A. students from being punished for behaviors that are a direct result of their disability.

How well did you know this?
1
Not at all
2
3
4
5
Perfectly
195
Q

Which of the following activities is most likely to aid a seventh-grade student with a specific learning disability (SLD) in written expression who frequently writes incomplete sentences?

A. Work to increase the student’s vocabulary so that he has more words to choose from when composing sentences.

B. Have the student copy complete sentences repeatedly until he understands the structure and grammar.

C. Teach the proper conventions of punctuation so that the student can correctly punctuate his sentences.

D. Teach the student to diagram his sentences and identify which parts of speech are missing when he writes sentence fragments.

A

D. Teach the student to diagram his sentences and identify which parts of speech are missing when he writes sentence fragments.

How well did you know this?
1
Not at all
2
3
4
5
Perfectly
196
Q

Mrs. Smallwood is a third-grade teacher in an inclusion classroom. Which of the following activities introducing her students to the concept of fractions is most likely to engage kinesthetic learners of all ability levels?

A. Show a short video demonstrating how to add and subtract fractions with different denominators.

B. Have the students color pie charts on a worksheet to represent different fractional relationships.

C. Have the students stand up and arrange themselves into groups of varying sizes to represent fractions related to the makeup of the class.

D. Have the students complete a worksheet requiring them to identify fractions depicted by graphics.

A

C. Have the students stand up and arrange themselves into groups of varying sizes to represent fractions related to the makeup of the class.

How well did you know this?
1
Not at all
2
3
4
5
Perfectly
197
Q

A special education teacher is working on reinforcing foundational reading skills in order to support her student’s literacy development. She writes the letter H on the board and makes the /h/ sound. She asks the class to name as many words as they can that start with the letter H. Which of the following is this activity most focused on improving?

A. print awareness

B. letter recognition

C. phonemic awareness

D. word awareness

A

C. phonemic awareness

How well did you know this?
1
Not at all
2
3
4
5
Perfectly
198
Q

Which of the following practices by a first-grade teacher is the most important to developing independent readers?

A. facilitating a variety of literary experiences

B. requiring high level texts for independent reading

C. maintaining consistently accountable expectations that are monitored through the use of reading logs

D. sharing the teacher’s own love of reading

A

A. facilitating a variety of literary experiences

How well did you know this?
1
Not at all
2
3
4
5
Perfectly
199
Q

To help encourage a third-grade student with attention-deficit hyperactivity disorder (ADHD) to maintain focus and complete assignments, which of the following would be the best approach?

A. Allow him to choose which assignments he will complete and in which order.

B. Set a goal for days on task during a six-week grading period with a reward at the end.

C. Remind the student to focus and stay on task with constant verbal reminders.

D. Offer him short-term incremental rewards that add up for the opportunity to earn larger rewards.

A

D. Offer him short-term incremental rewards that add up for the opportunity to earn larger rewards.

How well did you know this?
1
Not at all
2
3
4
5
Perfectly
200
Q

Following the distribution of periodic IEP goal progress reports, the parent of a seventh-grade student contacts Ms. Simpson, a special education case manager, to express concern over her child’s lack of progress in her math goal. How should Ms. Simpson respond?

A. Forward the parent’s concerns to a campus administrator and request that they respond.

B. Schedule an admission, review, and dismissal (ARD) committee meeting to reevaluate and revise the appropriateness of the student’s IEP goals and supports.

C. Provide copies of assignments and tasks used to compile the data for the progress report and explain how the work relates to the student’s goal while soliciting parent input for how to provide better support for the student.

D. Send a detailed report of the student’s performance in the relevant domains on state assessments over the last three years to emphasize that this lack of progress is not uncommon.

A

C. Provide copies of assignments and tasks used to compile the data for the progress report and explain how the work relates to the student’s goal while soliciting parent input for how to provide better support for the student.

How well did you know this?
1
Not at all
2
3
4
5
Perfectly
201
Q

Which of the following is the first step in drafting an effective behavior intervention plan (BIP)?

A. charting behavior to identify its antecedent

B. identifying the behavior that is to be replaced

C. selecting an appropriate substitute for the behavior that will serve the same function

D. identifying the function of the behavior

A

B. identifying the behavior that is to be replaced

How well did you know this?
1
Not at all
2
3
4
5
Perfectly
202
Q

Following the second reading fluency assessment of the school year, a third-grade teacher noticed that a few of her students have shown no or very little fluency growth from the first assessment nine weeks before. Which of the following would be the least effective way to support their fluency development?

A. facilitate opportunities for the students to read aloud to themselves or with a partner

B. increase the difficulty of their independent reading

C. provide them opportunities to reread texts

D. incorporate choral reading into the classroom routines

A

B. increase the difficulty of their independent reading

How well did you know this?
1
Not at all
2
3
4
5
Perfectly
203
Q

What is the primary benefit of educating students with disabilities in general education classrooms alongside non-disabled peers?

A. Inclusion classrooms reduce undesirable behaviors from special education students by allowing them to observe and imitate the conduct of non-disabled peers.

B. Special education students in general education classes benefit from accessing the gen-ed curriculum and gaining the same social and emotional experiences as their non-disabled peers.

C. Teachers are able to enlist the help of non-disabled peers to assist special education students with academic and social situations.

D. Inclusion classrooms provide an enriching experience for non-disabled students, who get to learn about the challenges and unique abilities of their special education peers.

A

B. Special education students in general education classes benefit from accessing the gen-ed curriculum and gaining the same social and emotional experiences as their non-disabled peers.

How well did you know this?
1
Not at all
2
3
4
5
Perfectly
204
Q

Caleb is a tenth-grade student with a learning disability and currently reads at a second-grade level. Which of the following is the most appropriate accommodation for Caleb’s math teacher to provide during assessments?

A. alternate questions that reflect Caleb’s reading level

B. oral administration of tests and assignments

C. extended time to complete tests and assignments

D. frequent breaks to prevent Caleb from become overwhelmed

A

B. oral administration of tests and assignments

How well did you know this?
1
Not at all
2
3
4
5
Perfectly
205
Q

As part of the prereferral intervention in cases where a disability is suspected but not yet confirmed, all of the following are acceptable means of gathering data EXCEPT:

A. engaging in an informal conversation with the student to assess his level of comfort with a range of academic skills.

B. placing the student in a resource class for a brief time to see if his performance improves.

C. giving the student a variety of standard accommodations across a range of tasks to analyze which are likely to be helpful.

D. allowing the student to complete written assignments using a word processor to assess the potential benefit of assistive technology.

A

B. placing the student in a resource class for a brief time to see if his performance improves.

How well did you know this?
1
Not at all
2
3
4
5
Perfectly
206
Q

A first-grade student with a visual impairment has been added to an inclusion class. Which of the following would be most important for her teachers to keep in mind when arranging the classroom?

A. The student should be seated near non-impaired peer partners who can help explain content that is shared visually.

B. The student should be seated near enough important visuals for her to engage with material that is shared visually.

C. The student should be seated near the back of the classroom so that she will not be embarrassed when the teachers help her complete assignments.

D. The student should be seated near the front of the room so that she can hear the teacher during whole-group instruction.

A

B. The student should be seated near enough important visuals for her to engage with material that is shared visually.

How well did you know this?
1
Not at all
2
3
4
5
Perfectly
207
Q

Student groups are given a six-sided die, with each side labeled a number 1 through 6. Each student group rolls the die 75 times and records the number that is rolled. If there are 8 groups of students participating in this activity, which of the following is most likely the total number of times a 4 was rolled?

A. 13

B. 98

C. 75

D. 154

A

B. 98

How well did you know this?
1
Not at all
2
3
4
5
Perfectly
208
Q

Which of the following procedural safeguards is included in the Individuals with Disabilities Education Act (IDEA)?

A. Students may not be removed to an interim alternative educational setting due to weapons, drugs, or serious bodily injury if the behavior was a manifestation of the child’s disability.

B. Parents may not pursue mediation prior to requesting a due process hearing.

C. Parents have the right to obtain an Individual Education Evaluation (IEE) if they are in disagreement with the school district’s evaluation.

D. Parents are required to be present at all IEP meetings.

A

C. Parents have the right to obtain an Individual Education Evaluation (IEE) if they are in disagreement with the school district’s evaluation.

How well did you know this?
1
Not at all
2
3
4
5
Perfectly
209
Q

According to the Individuals with Disabilities Education Act (IDEA), which of the following must be communicated to parents prior to the implementation of an IEP?

Select all answers that apply.

A. data on present academic performance

B. detailed descriptions of all special education services to be received by the child

C. medical records that are relevant to the diagnosis or services to be provided

D. identifying information about the student’s classroom teacher

A

A. data on present academic performance

B. detailed descriptions of all special education services to be received by the child

How well did you know this?
1
Not at all
2
3
4
5
Perfectly
210
Q

A kindergarten teacher begins each class by reciting the Pledge of Allegiance. The teacher has a poster with the words to the Pledge of Allegiance and taps each word as the class recites the Pledge. By tapping the word on the poster in correlation with reciting the word, the teacher is best demonstrating which of the following?

A. phonological awareness

B. alphabetic principle

C. decoding

D. concepts of print

A

D. concepts of print

How well did you know this?
1
Not at all
2
3
4
5
Perfectly
211
Q

If the parents of a student with an IEP express to the student’s special education case manager that they are dissatisfied with the progress their child is making, which of the following would be the best response for the case manager to make?

A. Share informal reports from the child’s teachers about his classroom behavior and habits that may explain why he is not making progress.

B. Offer to change some of the supports in the student’s IEP to see if his performance improves.

C. Assure the parents that they will be able to voice their concerns at the child’s next annual IEP meeting.

D. Ask the parents if they would like to request an admission, review, and dismissal (ARD) committee meeting to discuss potential changes to the IEP.

A

D. Ask the parents if they would like to request an admission, review, and dismissal (ARD) committee meeting to discuss potential changes to the IEP.

Transparency and open communication between district representatives and the parents of students with special needs is vital to helping the students succeed. Therefore, it is important to make sure parents know what their options are when requesting information or changes to their child’s IEP. A parent can request an admission, review, and dismissal (ARD) committee meeting at any time for any reason, and concerns over their student’s progress is certainly an acceptable reason to request such a meeting.

How well did you know this?
1
Not at all
2
3
4
5
Perfectly
212
Q

During its financial literacy unit of study, a seventh-grade math resource class has been introduced to and mastered the concept of computing credits and debits to keep a balanced checking account. Which of the following would be the logical next step in teaching this unit?

A. Teach the process for computing and planning for the cost of attending college.

B. Teach the difference between static expenses and variable expenses.

C. Teach the difference between simple and compound interest.

D. Teach the process for making change as part of a cash purchase.

A

B. Teach the difference between static expenses and variable expenses.

Once students understand how to balance a checking account, the logical next step is to teach them how to plan and budget for various types of expenses that they may incur.

How well did you know this?
1
Not at all
2
3
4
5
Perfectly
213
Q

Maxwell is a fifth-grade student with attention-deficit hyperactivity disorder (ADHD). During his annual IEP meeting, the special education teacher shares the results of Maxwell’s most recent unit tests in the four core subjects. This data falls under which type of assessment?

A. summative assessment

B. formative assessment

C. norm-referenced assessment

D. informal assessment

A

A. summative assessment

A summative assessment evaluates student learning at the end of a unit of study grading each student’s work based on a rubric or aligned standard.

How well did you know this?
1
Not at all
2
3
4
5
Perfectly
214
Q

Which of the following actions by a teacher is most likely to help a second-grade student with attention-deficit hyperactivity disorder (ADHD) maintain focus and limit impulsive behaviors?

A. Assigning the student small chores that allow her to assist the teacher and require her to get up and move around at regular intervals during the school day.

B. Moving the student’s desk next to a window so that she can take occasional sensory breaks to check on what is going on outside.

C. Arranging the student’s desk so that she faces a blank wall to limit the likelihood of her becoming distracted.

D. Assigning a peer partner to redirect the student and help keep her on task.

A

A. Assigning the student small chores that allow her to assist the teacher and require her to get up and move around at regular intervals during the school day.

Turning a fidgety student into a teacher aide can be an effective way to build in time for movement and talking during the day without disrupting the flow of instruction. Students with ADHD often need frequent breaks during the school day to move around and engage with others in their environment to help them reset their ability to focus.

How well did you know this?
1
Not at all
2
3
4
5
Perfectly
215
Q

Which skill is the focus of an activity in a second-grade resource class requiring students to look at animal pictures and say the letter sound that begins each animal name?

A. phonemic awareness

B. sight word practice

C. reading fluency

D. vocabulary expansion

A

A. phonemic awareness

Phonemic awareness is the ability to identify and manipulate individual sounds in spoken words. The activity as described would be focused on improving phonemic awareness.

How well did you know this?
1
Not at all
2
3
4
5
Perfectly
216
Q

What is the value of allowing students in a fourth-grade language arts inclusion class to select books from their favorite genres during silent sustained reading?

A. Students are more likely to engage meaningfully with a text if it matches their personal interests.

B. Silent sustained reading with a self-selected book is likely to increase fluency by giving the students a text they are excited to read.

C. Freedom of choice makes it more likely that students will select a book that they find challenging.

D. Silent sustained reading with a self-selected book allows the students to connect their text to personal experiences.

A

A. Students are more likely to engage meaningfully with a text if it matches their personal interests.

Meaningful engagement with a text leads to greater metacognition through skills like inferencing and questioning. Allowing students to choose their own books for this activity makes it more likely that they will care about what they are reading and spend more time thinking about it.

How well did you know this?
1
Not at all
2
3
4
5
Perfectly
217
Q

Transcription support, such as having an adult bubble in an answer document, would most likely benefit a student with which of the following disabilities?

A. specific learning disability (SLD) in math problem-solving

B. attention-deficit hyperactivity disorder (ADHD)

C. emotional disturbance

D. specific learning disability (SLD) in written expression

A

D. specific learning disability (SLD) in written expression

Students with a specific learning disability in written expression typically suffer from poor fine motor control, so transcription support would likely be a helpful accommodation for such students.

How well did you know this?
1
Not at all
2
3
4
5
Perfectly
218
Q

A key consideration when determining the least restrictive environment for a student is to consider:

A. the number of times restraint holds must be used to therapeutically de-escalate a student.

B. whether the student is able to receive education to the maximum extent appropriate alongside peers who are not disabled.

C. whether the student is able to move in the room with little permission from the teacher.

D. the number of staff members assisting the student at any given time.

A

B. whether the student is able to receive education to the maximum extent appropriate alongside peers who are not disabled.

A key consideration when determining the least restrictive environment is to consider whether the student is able to receive education to the maximum extent appropriate alongside peers who are not disabled.

How well did you know this?
1
Not at all
2
3
4
5
Perfectly
219
Q

A teacher in the district’s Disciplinary Alternative Education Program (DAEP) is preparing to receive a new student with a specific learning disability (SLD) in reading comprehension. What steps should the teacher take to prepare for the student’s arrival?

A. Request and review the student’s most recent full individual evaluation (FIE), gather low-level, high-interest texts, adjust the classroom lighting to be softer.

B. Request discipline records from administration, adjust the classroom to a more calming arrangement, develop a reward system to encourage positive behaviors.

C. Request behavior reports from the student’s teachers, draft new IEP goals for the DAEP placement, contact the student’s parents to set behavior expectations.

D. Contact the case manager, request and review the IEP, request and review the student’s most recent assessment data in core classes.

A

D. Contact the case manager, request and review the IEP, request and review the student’s most recent assessment data in core classes.

When receiving a new special education student who is experiencing a change in placement, it is crucial for the teacher of record to access and utilize existing IEP paperwork and assessment data to drive instruction and maintain academic progress.

How well did you know this?
1
Not at all
2
3
4
5
Perfectly
220
Q

Regan is a ninth-grade student with a specific learning disability (SLD) in reading comprehension. During Regan’s annual IEP meeting, her father requests that she be dismissed from special education services since her last report card showed As and Bs in all core classes. The most appropriate response to Regan’s father is to:

A. refer him to Regan’s most recent full individual evaluation (FIE) to review all her G scores.

B. agree with him and recommend that Regan no longer receive special education services.

C. point out that the classroom accommodations Regan receives as part of her IEP are likely contributing to her academic success and that she may struggle or regress without these supports.

D. recommend moving Regan out of her inclusion classes and into advanced classes so she will have access to more challenging curriculum.

A

C. point out that the classroom accommodations Regan receives as part of her IEP are likely contributing to her academic success and that she may struggle or regress without these supports.

It is important to communicate clearly with parents when discussing IEP supports and the impact they have on student success. A more appropriate first step–that may also serve as a compromise–would be to evaluate which accommodations Regan no longer needs and analyze her progress without these supports.

How well did you know this?
1
Not at all
2
3
4
5
Perfectly
221
Q

Matilda is a fourth-grade student who has a specific learning disability (SLD). Matilda’s parents have asked the special education teacher to provide some suggestions for how to make homework completion more manageable. Which of the following recommendations is most appropriate for the special education teacher to advise Matilda’s parents to try at home?

A. Encourage Matilda’s older siblings to check her homework for accuracy.

B. Allow Matilda to take ownership of homework completion. If she chooses not to do it, she will face the consequences.

C. Read aloud with Matilda and ask questions to aid in comprehension.

D. Offer Matilda a five minute screen-time break for every 10 minutes of homework she completes.

A

C. Read aloud with Matilda and ask questions to aid in comprehension.

Reading aloud with Matilda will help her to focus on comprehension of the content. By asking questions, her parents will be able to keep her engaged and determine her level of understanding.

How well did you know this?
1
Not at all
2
3
4
5
Perfectly
222
Q

Which part of the transition planning process is most valuable in helping students establish a purpose for education as they move into high school and toward post-secondary life?

A. Providing opportunities for community members to meet with and encourage students.

B. Creating the transition plan so that it matches the student’s academic strengths.

C. Encouraging all students to continue their formal education following high school graduation.

D. Creating the transition plan with the student so that it focuses on and reflects the student’s true interests and goals for their future.

A

D. Creating the transition plan with the student so that it focuses on and reflects the student’s true interests and goals for their future.

Student buy-in is a crucial element of successful transition planning; therefore, any good transition plan should start with an honest and open inventory of the student’s interests and goals. These interests and goals should also be reevaluated annually while making appropriate adjustments to the transition plan.

How well did you know this?
1
Not at all
2
3
4
5
Perfectly
223
Q

Which of the following is not a component of print concepts?

A. identifying basic punctuation

B. understanding the difference between letters, words, spaces, and sentences

C. phonemic awareness

D. awareness of the left-to-right, top-to-bottom progression of text

A

C. phonemic awareness

Phonemic awareness is the recognition of individual sounds that make up words. It is not a print concept.

How well did you know this?
1
Not at all
2
3
4
5
Perfectly
224
Q

Richard is an eighth-grade student with autism spectrum disorder (ASD). At his annual IEP meeting, Richard’s parents expressed a desire to dismiss him from speech therapy services since he has no trouble speaking clearly, and they are concerned that his weekly sessions with the campus speech therapist are causing him to fall behind in his classes. The speech therapist has previously told Richard’s case manager that their sessions focus on intentionally teaching Richard how to interpret the facial expressions and body language of others and how to positively engage in social interactions through reciprocal communication. What is the best response to Richard’s parents’ concerns?

A. Produce data that proves Richard’s academic performance is not suffering due to his weekly speech therapy sessions.

B. Accept the parents’ recommendation without dissent, since they know their child better than anyone else.

C. Share anecdotal evidence from personal observations that prove that Richard is struggling to make friends and, therefore, needs to continue speech therapy.

D. Encourage the speech therapist to share data that measures Richard’s ability to engage in and maintain a conversation and how their sessions aim to help him build this skill.

A

D. Encourage the speech therapist to share data that measures Richard’s ability to engage in and maintain a conversation and how their sessions aim to help him build this skill.

All admission, review, and dismissal (ARD) committee decisions should be data-backed and supported by research. Students with ASD often require direct instruction to learn social behaviors that developmentally typical students absorb and implement naturally.

How well did you know this?
1
Not at all
2
3
4
5
Perfectly
225
Q

The parents of an eighth-grade student with autism spectrum disorder (ASD) email their son’s language arts teacher with concerns about his lack of progress in writing. They request that he be allowed to type his essays on a computer instead of writing them by hand. Which of the following would be the most logical course of action for the teacher to take before drafting a reply?

A. Evaluate the student’s available written work in various formats to determine if he is more successful on written assignments when using word processing software.

B. Review the student’s list of state assessment accommodations to evaluate the appropriateness of using word processing software for class writing assignments.

C. Compare the student’s essay grades to those of his non-disabled peers to determine the need for additional support on written assignments.

D. Evaluate the state standards related to writing to determine if use of word processing software is allowed.

A

A. Evaluate the student’s available written work in various formats to determine if he is more successful on written assignments when using word processing software.

Gathering data to compare the student’s written work when typing vs. writing by hand is the logical step to take before replying to the parents’ request. It is important to honor parent concerns and use data to drive decision-making when it comes to accommodations.

How well did you know this?
1
Not at all
2
3
4
5
Perfectly
226
Q

Which of the following activities is most likely to help assess and build reading comprehension among students in a fifth-grade language arts inclusion class?

A. In small groups of three to four, have students independently read a passage, then work together to answer a brief series of related multiple-choice questions.

B. In small groups of three to four, have students read and discuss a passage, then develop a group summary to share with the class.

C. Have students engage in silent sustained reading of a common text, then allow volunteers to share important observations they made about the passage.

D. Have students independently read and annotate a text passage and then write their own questions related to the passage.

A

B. In small groups of three to four, have students read and discuss a passage, then develop a group summary to share with the class.

Discussion of a text with peers increases metacognition during reading, and collaborating to construct a suitable summary requires the use of higher-level thinking to weed out unimportant details and prove that the students understand the main ideas or themes of the passage.

How well did you know this?
1
Not at all
2
3
4
5
Perfectly
227
Q

A special education teacher has been working with a student on phonological awareness skills. Which of the following skills is most likely to be the final phonological awareness skills that the student will master before the teacher transitions from phonological awareness to explicit phonics instruction?

A. identifying syllables

B. manipulating phonemes

C. counting syllables

D. recognizing rhymes

A

B. manipulating phonemes

Manipulating phonemes is a more complex aspect of phonological awareness and would likely be one of the last skills prior to explicit phonics instruction.

Syllable awareness is a basic component of phonological awareness. The student would still require further instruction on phonological awareness before moving on to explicit phonics instruction.

How well did you know this?
1
Not at all
2
3
4
5
Perfectly
228
Q

Which of the following should be part of post-secondary schooling transition planning for a high school sophomore with autism spectrum disorder (ASD)?

A. Helping the student obtain a driver’s license to travel to and from work.

B. Encouraging the student to apply to as many colleges as possible.

C. Arranging job interviews on the student’s behalf for positions deemed appropriate to his abilities.

D. Establishing whether the student plans to live independently or in a supportive environment.

A

D. Establishing whether the student plans to live independently or in a supportive environment.

An important part of transition planning involves making sure the student and his family are considering options for the future, such as where and with whom the student plans to live. The student’s input and preference are valuable parts of the planning process that can help guide decisions to make adequate preparation for the transition from secondary school to adult life.

How well did you know this?
1
Not at all
2
3
4
5
Perfectly
229
Q

When collaborating with stakeholders about the goals and services for a student who receives special education services, a key aspect of the collaboration is:

A. to focus on state standards.

B. to ensure that the student is successful and learns.

C. to come to a consensus regarding the goals and services.

D. to document the notes from every conversation between school personnel and stakeholders.

A

B. to ensure that the student is successful and learns.

The focus should always be on the success and learning of the student.

How well did you know this?
1
Not at all
2
3
4
5
Perfectly
230
Q

If a student who uses a feeding tube at school experiences nausea or vomiting during or shortly after feeding, which of the following is the most likely cause?

A. insufficient formula

B. improper cleaning of tube

C. not enough water or fiber

D. too much formula or too high a feeding rate

A

D. too much formula or too high a feeding rate

The amount and rate of delivery of formula should be specified in the student’s IEP and closely monitored by caregivers who assist with feeding at school, as changes in either variable can lead to nausea or vomiting.

How well did you know this?
1
Not at all
2
3
4
5
Perfectly
231
Q

The most effective way to ensure that IEP goals are properly written and implemented is to align them to:

A. state standards which are likely to be assessed in upcoming grading periods.

B. state standards in which the student has displayed a strength over multiple assessment opportunities.

C. state standards in which the student has displayed a weakness over multiple assessment opportunities.

D. state standards which are at least two grade levels below the student’s actual grade placement.

A

C. state standards in which the student has displayed a weakness over multiple assessment opportunities.

IEP goals should reflect an academic weakness that would benefit the student if improved upon. Accurate identification of strengths and weaknesses requires multiple data points and, possibly, multiple modes of assessment.

How well did you know this?
1
Not at all
2
3
4
5
Perfectly
232
Q

Mr. Carter is a special education teacher who works with middle school students. He receives a request from a sixth-grade science teacher to help evaluate data for a new student whose primary home language is Spanish. Upon reviewing the student’s data, Mr. Carter finds that the student performs well on lab assignments and other hands-on tasks with visual elements, but that she struggles with tasks involving grade-level academic vocabulary. Which of the following would be the most logical next step for Mr. Carter to take?

A. Refer the science teacher to the campus contact for ESL support.

B. Begin the referral process to have the student tested for special education services.

C. Provide the science teacher with some options for including visual aids when introducing and testing new vocabulary and check back to see if the student’s performance improves.

D. Pull the student for individual instruction in strategies for decoding and using new vocabulary.

A

C. Provide the science teacher with some options for including visual aids when introducing and testing new vocabulary and check back to see if the student’s performance improves.

Since it is likely that the student is struggling with connecting new vocabulary to existing knowledge as a result of her status as an English language learner, the logical first step is to provide visual aids as a support to see if her comprehension and performance improve.

How well did you know this?
1
Not at all
2
3
4
5
Perfectly
233
Q

Bailey, a fourth-grade student with a specific learning disability (SLD) in math problem-solving, struggles to solve word problems that require multiple computations. Which of the following would be an appropriate support for Bailey’s math teacher to implement?

A. Simplify the vocabulary in the word problems so Bailey does not get confused.

B. Give Bailey a calculator, so she can focus on decoding the word problems instead of completing the calculations.

C. Instead of word problems, allow Bailey to complete worksheets with math facts that require the same operations.

D. Highlight the key information and operational signal words within the word problems on Bailey’s worksheets and give her direct instruction on how to find and emphasize this information herself.

A

D. Highlight the key information and operational signal words within the word problems on Bailey’s worksheets and give her direct instruction on how to find and emphasize this information herself.

Emphasizing key information is an effective strategy for students who struggle with multi-step problems. Bailey still gets to practice reading and solving word problems with the support of guidance toward identifying which numbers and operations to use.

How well did you know this?
1
Not at all
2
3
4
5
Perfectly
234
Q

Which of the following assistive technology devices would be most appropriate in supporting a medically fragile student who has lost the ability to speak clearly?

A. A whiteboard for writing down messages and requests.

B. A touchscreen text-to-speech tablet programmed with key words and phrases the student can identify.

C. A flipchart with pictures of common tasks and requests the student can show to communicate his needs.

D. A laptop computer that takes dictation and transcribes speech to text.

A

B. A touchscreen text-to-speech tablet programmed with key words and phrases the student can identify.

Text-to-speech technology is the best analog for a student who is unable to speak himself. Pre-programming key words and phrases and displaying them in a manner that is accessible to the student is also beneficial, since the student may not possess the typing skills necessary to communicate clearly.

How well did you know this?
1
Not at all
2
3
4
5
Perfectly
235
Q

Which of the following is the most prevalent cause of due process hearings in the state of Texas?

A. disputes regarding placement

B. disputes regarding identification

C. disputes regarding evaluation

D. disputes regarding the IEP

A

D. disputes regarding the IEP

According to a 2017 study conducted by Sage Journals, in which the authors analyzed 139 due process hearings in Texas between 2011 and 2015, 62% of all hearings included a dispute over the IEP.

According to a 2017 study conducted by Sage Journals, in which the authors analyzed 139 due process hearings in Texas between 2011 and 2015, 37% of all hearings included a dispute over placement of students with disabilities.

How well did you know this?
1
Not at all
2
3
4
5
Perfectly
236
Q

Which of the following would be most helpful in aiding a high school junior with a mild intellectual disability to prepare for a job placement as a stocker at a retail business?

A. Training her to use a cash register and make change for cash payments.

B. Help her practice categorizing, organizing, and displaying various products that could be sold at the location.

C. Discuss marketing theory relative to how to build and arrange product displays.

D. Practicing the route from home to work, so she can drive independently.

A

B. Help her practice categorizing, organizing, and displaying various products that could be sold at the location.

Sorting and arranging products would be effective and applicable practice for the student’s job as a stock-person at a retail location.

How well did you know this?
1
Not at all
2
3
4
5
Perfectly
237
Q

Hafsa, a first-grade student diagnosed with a developmental delay, has begun to recognize familiar words in the environment or in text. Which of the following literacy skills is she most likely to be working on during small-group guided reading?

A. phonological awareness and manipulating graphemes or phonemes to make new words (eg. sch-ool becomes dr-ool, m-at becomes b-at)

B. practicing reading with fluency and understanding punctuation

C. recognizing sight words and using illustrations and writing patterns to help decode each sentence

D. chunking words to help sound out longer or compound words

A

C. recognizing sight words and using illustrations and writing patterns to help decode each sentence

This would be the most appropriate skill for Hafsa to be working on in a small group considering her reading level. Supported by the teacher in a small group, she would encounter sentences and read books that hinge on learning a sight word. For example, the book may be called “I like” and follow a pattern using sight words. A page might say: “I like to ride my bike” and have a picture of the character riding a bike. The student would know the pattern and see the picture and be able to read the sentence with prompting.

How well did you know this?
1
Not at all
2
3
4
5
Perfectly
238
Q

What is the primary benefit of framing behavior redirection responses in a positive manner (telling the student what they SHOULD do), rather than a negative manner (telling the student what they SHOULDN’T do)?

A. Positive redirection takes less time than negative redirection and allows the teacher to get back to the task at hand more quickly.

B. This practice avoids the risk of hurting the student’s feelings and putting them on the defensive.

C. Positive redirection is less embarrassing for a student to deal with than negative redirection.

D. Modeling or describing the desired behavior creates a more positive interaction and gives the student a positive expectation to try and meet instead of a negative limit to stay within.

A

D. Modeling or describing the desired behavior creates a more positive interaction and gives the student a positive expectation to try and meet instead of a negative limit to stay within.

Behavior research shows that the most effective way to change undesired behavior is to teach the student to replace it with a better option.

How well did you know this?
1
Not at all
2
3
4
5
Perfectly
239
Q

Connie is a fifth-grader with an emotional disturbance who frequently has angry outbursts when she is required to work with others in a group setting. The best way for her teacher to help her is to:

A. make time for a private conversation with Connie about what is upsetting her and how to help her work more cooperatively with others.

B. give Connie a pass to go see the school counselor when she gets upset so she doesn’t have these outbursts in front of the class.

C. step in and verbally reprimand both Connie and her group partners in front of the class so she doesn’t feel singled out.

D. give Connie independent assignments when the rest of the class is doing group work.

A

A. make time for a private conversation with Connie about what is upsetting her and how to help her work more cooperatively with others.

Private discussions about behavior are always preferable to implementing discipline in a group setting, especially for students with emotional disturbance or behavior disorders. If Connie is able to take ownership of the plan for future group work she may feel like she has more control over the situation and may be able to avoid whatever anxiety she has been experiencing in group settings.

How well did you know this?
1
Not at all
2
3
4
5
Perfectly
240
Q

A teacher wants to model for students how to pull the main idea(s) from a nonfiction text. Which activity below would be the most effective way to demonstrate this for the students?

A. Read a text that’s projected for the class and take notes in the margins while reading.

B. While reading a text projected for the class, the teacher highlights transition words and phrases used in each paragraph.

C. While reading aloud to the class, the teacher pauses to define vocabulary terms using context clues.

D. Provide student with a teacher-written summary of a text after the class has read it together.

A

A. Read a text that’s projected for the class and take notes in the margins while reading.

By modeling his thought process while reading, the teacher is able to demonstrate how a strong reader summarizes information during reading. The teacher is also demonstrating an effective tool students can use when they read on their own to find and summarize main ideas.

How well did you know this?
1
Not at all
2
3
4
5
Perfectly
241
Q

Which of the following behaviors would most likely be exhibited by a student with attention-deficit hyperactivity disorder (ADHD)?

A. inability to focus on one task or subject for extended periods and poor impulse control

B. perseverative focus on narrow areas of interest and extreme sensitivity to sensory stimuli

C. average development of general cognitive skills with a pronounced deficit in one area

D. extreme nearsightedness

A

A. inability to focus on one task or subject for extended periods and poor impulse control

Students with ADHD struggle to maintain focus for extended periods and tend to display impulsive behaviors and speech.

How well did you know this?
1
Not at all
2
3
4
5
Perfectly
242
Q

A second-grade student has shown strength in memorizing sight words and reading common, familiar words in books, but she consistently struggles with unfamiliar words even if they are decodable. Based on her specific challenge, in which of the following skills does she require further instruction?

A. prosody

B. structural analysis

C. direct phonics instruction

D. contextual analysis

A

C. direct phonics instruction

Direct phonics instruction teaches students to sound out words. This described student is able to read familiar words, but not new words, so she appears to be depending on her memory to recognize words and not her ability to sound them out.

How well did you know this?
1
Not at all
2
3
4
5
Perfectly
243
Q

A kindergartener has started showing the ability to decode words by breaking them into simple forms. Which of the following best describes this student’s current stage of literacy development?

A. Proficient

B. Beginning Reader

C. Emergent Literacy

D. Early Fluent

A

B. Beginning Reader

Beginning readers are beginning to make sense of the text on the page. They will begin to decode and recognize more words.

The emergent stage of development is when children understand that written language communicates meaning and will recognize some common words like their own name.

How well did you know this?
1
Not at all
2
3
4
5
Perfectly
244
Q

Mrs. Price is a language arts teacher in a sixth-grade inclusion class. She is preparing to have her students compose an expository essay answering the following question: Why is honesty important? Which of the following types of graphic organizer would be most effective in helping Mrs. Price’s students with a specific learning disability (SLD) in written expression compose an essay that aligns with the prompt?

A. a Venn diagram

B. a brainstorming web with space for personal observations related to a specific topic

C. a flow chart

D. a heart map showing interests and dislikes

A

B. a brainstorming web with space for personal observations related to a specific topic

A web encouraging students to note their own ideas related to a specific topic is an effective first step toward organizing their thoughts into a format that fits the style of an expository essay, where they are required to support their thesis with multiple examples.

How well did you know this?
1
Not at all
2
3
4
5
Perfectly
245
Q

A teacher has provided her class with a descriptive writing assignment. The students have chosen a topic and are now filling out a sensory-focused graphic organizer in which they list descriptions that align with each of the five senses. What step in the writing process are the students most likely to be in?

A. editing

B. prewriting

C. drafting

D. revising

A

B. prewriting

Before beginning descriptive writing, students should brainstorm ways in which they might describe the topic. The graphic organizer is created for this purpose.

How well did you know this?
1
Not at all
2
3
4
5
Perfectly
246
Q

Which of the following behaviors are appropriate for learners in the Early or Beginning stage (Stage 2) of literacy development?

A. self-correction when what is read is unclear

B. memorizing texts in order to appear to “read” them

C. using a finger to move through a text, word-by-word

D. recognizing the first letter of their name

A

C. using a finger to move through a text, word-by-word

It is appropriate for learners in the Early or Beginning stage (Stage 2) of literacy development to “finger-point” as they move from word to word within a text.

How well did you know this?
1
Not at all
2
3
4
5
Perfectly
247
Q

Which of the following strategies aimed at reducing disruptive behavior is most likely to effect lasting change?

A. Assign a peer partner to remind disruptive students of class norms when they exhibit negative behavior.

B. Remove disruptive students from the classroom so the rest of the class can focus.

C. Provide praise and reward appropriate behavior while ignoring negative behavior.

D. Calmly but firmly correct the disruptive behavior every time it happens.

A

C. Provide praise and reward appropriate behavior while ignoring negative behavior.

Research-backed behavior theory indicates that rewarding desired behavior is significantly more effective than punishing unwanted behaviors. The goal in behavior modification should always be to REPLACE the unwanted behaviors, not simply eliminate them. Thus, feeding the positive behavior with praise and attention and starving the negative behavior is the path to long-term change.

How well did you know this?
1
Not at all
2
3
4
5
Perfectly
248
Q

Which of the following is the best method for demonstrating the concept of equivalent fractions to a group of fourth-grade students in an inclusion math class?

A. Divide the class into various groups based on individual traits (eye color, hair color, age) and compare the resulting fractions of each group to the others.

B. Provide a picture of two pizzas of equal size cut into different numbers of slices.

C. Show a brief video explaining the process of simplifying fractions to find lowest common denominators for the process of comparing fractions.

D. Using sidewalk chalk, explore how many student-length segments it would take to equal two teacher-length segments.

A

D. Using sidewalk chalk, explore how many student-length segments it would take to equal two teacher-length segments.

This activity provides visual cues to solidify understanding, and it also requires hands-on learning opportunities and cooperation with peers to complete. This is an advantage in inclusion settings as it involves multiple modalities to engage a diverse group of learners.

How well did you know this?
1
Not at all
2
3
4
5
Perfectly
249
Q

Which of the following would be illegal when considering a child for special education services?

A. testing all children in English

B. allowing a committee to make the decision if services are appropriate

C. qualifying a child in high school that has been able to pass previous grades

D. using multiple tests to assess a disability

A

A. testing all children in English

Children must be tested in their native language. It is illegal to require all students to test in English.

How well did you know this?
1
Not at all
2
3
4
5
Perfectly
250
Q

In reviewing the parent questionnaire before an initial IEP meeting for a second-grade student, a diagnostician discloses the following:

The student did not begin talking in complete sentences until his fourth year.

The student prefers soft fabrics and loose, elastic waistbands.

The student tends to melt down and throw tantrums when his hands get dirty.

The student is excessively interested in trains and only wants to discuss this topic, despite repeated attempts to engage his attention in other topics.

Based on this information, the student should be tested for:

A. attention-deficit hyperactivity disorder (ADHD)

B. autism spectrum disorder (ASD)

C. specific learning disability (SLD)

D. emotional disturbance

A

B. autism spectrum disorder (ASD)

The combination of speech delay, sensory issues, and consistent perseveration in a narrow area of interest are typically associated with ASD and further testing is appropriate.

How well did you know this?
1
Not at all
2
3
4
5
Perfectly
251
Q

What is the first step in initiating a referral when the district suspects a student may require special education services?

A. Draft and implement an initial IEP for the student to see if her academic performance improves.

B. Contact the student’s parents or guardians to request that they seek an outside diagnosis of any possible disability.

C. Request consent from the student’s parents or guardian to begin an evaluation for possible disability.

D. Convene an admission, review, and dismissal (ARD) committee meeting to discuss relevant data and possible interventions.

A

C. Request consent from the student’s parents or guardian to begin an evaluation for possible disability.

The first step in any initial evaluation for special education services is to obtain parental consent for evaluation.

How well did you know this?
1
Not at all
2
3
4
5
Perfectly
252
Q

Marisol is a second-grade student with a specific learning disability (SLD) in math calculation. Which of the following supports can best help Marisol learn to complete two-digit subtraction problems?

A. concrete reinforcers, such as counting sticks and blocks

B. flashcards

C. oral administration of the problems

D. a calculator

A

A. concrete reinforcers, such as counting sticks and blocks

Concrete reinforcers, like connecting blocks or tokens, can help a student who struggles with basic calculation to better visualize and understand what a math problem is asking her to do.

How well did you know this?
1
Not at all
2
3
4
5
Perfectly
253
Q

How might a specific learning disability (SLD) in reading comprehension impact a student’s ability to demonstrate mastery in math?

A. Students with reading comprehension deficiencies will likely struggle to decode and solve word problems.

B. Students with reading comprehension deficiencies will likely struggle to solve multi-step equations.

C. Students with reading comprehension deficiencies will likely struggle with spatial reasoning and geometry problems.

D. Students with reading comprehension deficiencies will likely struggle to maintain proper organization when performing calculations.

A

A. Students with reading comprehension deficiencies will likely struggle to decode and solve word problems.

Students with deficiencies in reading comprehension often lack sufficient vocabulary and background knowledge to decode word problems and understand what is being asked of them; therefore, they may struggle to solve these problems.

How well did you know this?
1
Not at all
2
3
4
5
Perfectly
254
Q

Maddie is a first-grade student with a specific learning disability, specifically dyscalculia. During mathematics, Maddie’s class is working on adding two-digit and one-digit numbers. One strategy that Maddie’s special education teacher suggests to support Maddie in learning addition within the general education classroom is:

A. to not worry about Maddie’s progress towards math-related goals since she struggles with dyscalculia.

B. to have Maddie sit next to a peer to double-check her work with them for accuracy.

C. to give Maddie the answer key so that she can see how the problems are solved.

D. to allow Maddie to use graph paper to help her keep track of the placement of the numbers.

A

D. to allow Maddie to use graph paper to help her keep track of the placement of the numbers.

Allowing Maddie to use graph paper is the best strategy to support her in two and one-digit addition in the classroom. Students with dyscalculia often struggle with maintaining appropriate place value when adding or subtracting numbers with multiple digits, so performing these calculations on graph paper can help Maddie stay organized.

How well did you know this?
1
Not at all
2
3
4
5
Perfectly
255
Q

Which of the following is the proper next step for a local education agency (LEA) to take once it has received communication from a parent within its district who believes her preschool-aged child may have a developmental disability?

A. Inform that parent that she can request a full individual evaluation (FIE) for her child once the child is enrolled in school.

B. Investigate the parent’s concerns, and, if necessary, initiate the procedure for completing a full individual evaluation (FIE) of the child.

C. Provide a list of trusted private diagnosticians to the parent so she can schedule a full individual evaluation of the child.

D. Refer the parent to helpful resources to better educate her about the challenges her child may face once she begins school.

A

B. Investigate the parent’s concerns, and, if necessary, initiate the procedure for completing a full individual evaluation (FIE) of the child.

As part of the Individuals with Disabilities Education Act (IDEA), public schools are required to identify, locate, and evaluate any child who may need special education. Schools are not required to agree to every request for evaluation, but the school must consider every request.

How well did you know this?
1
Not at all
2
3
4
5
Perfectly
256
Q

After reading Cloudy With a Chance of Meatballs by Judi and Ron Barrett, the teacher shows a short film clip of the animated feature film by the same title. This least enhances the comprehension skill of:

A. compare and contrast.

B. extended meaning.

C. story recall and retelling.

D. visual complementing.

A

C. story recall and retelling.

Comparing the animated version to the written would not best enhance the student’s ability to recall story points.

How well did you know this?
1
Not at all
2
3
4
5
Perfectly
257
Q

During independent reading time, a teacher walks around and holds mini book conferences with each student. She asks basic comprehension questions like, “What’s happening now?” or “How did the character respond to X?” While conferencing with Jane, the teacher has trouble getting Jane to relate anything that has happened other than the characters’ names. In order to help Jane succeed, what is the first step the teacher should take?

A. Assess Jane’s current reading level and make sure the text is not above her frustrational reading level.

B. Assess Jane’s current reading level and compare the reading level of the text she’s currently reading to texts she has read previously.

C. Assess Jane’s current reading level and make sure the text is not at her instructional reading level.

D. Assess Jane’s current reading level and make sure the text is not above her independent reading level.

A

D. Assess Jane’s current reading level and make sure the text is not above her independent reading level.

Finding Jane’s current reading level and ensuring that her novel selection is within an appropriate range is a good first step for helping Jane read texts that she can comprehend and discuss with less difficulty.

A student’s frustrational reading level is the level at which a student can read with extensive teacher assistance. Because this text is for independent reading time, Jane needs a text at her independent reading level, not her frustrational reading level.

How well did you know this?
1
Not at all
2
3
4
5
Perfectly
258
Q

Who is responsible for understanding and implementing the IEP of a student with a disability?

A. the student’s special education case manager

B. the student’s parents

C. the school counselor

D. any individual with a direct educational interest in the student

A

D. any individual with a direct educational interest in the student

All of the teachers and paraprofessionals who interact with a disabled student during the course of a normal school day should understand and follow relevant portions of the student’s IEP.

How well did you know this?
1
Not at all
2
3
4
5
Perfectly
259
Q

A seventh-grade student with an intellectual disability is working on basic algebra problems. His special education teacher notices the following mistake:

Problem: 6x = 24
/6
x = 24

Which of the following would be the best intervention to try with the student?

A. Provide a quiet alternative space for him to work so that he can focus on the problems and get more work done.

B. Use counters to help illustrate the concept of finding the variable by beginning with the total number, removing the number of counters called for in the problem, and finding X as the remaining number of counters.

C. Write the algebra problem as a word problem that asks the student to do the operation required to find the value of a variable, so that he understands what he is solving for.

D. Use an equal arm balance scale and dried beans to illustrate the concept of adding and removing things to and from both sides to keep the scale even.

A

D. Use an equal arm balance scale and dried beans to illustrate the concept of adding and removing things to and from both sides to keep the scale even.

Because the student only divided 6 from one side of the equation, the teacher can assume that the student is struggling with the central algebraic concept of keeping both sides equal. Using a hands-on intervention that demonstrates why you cannot take something from one side if the equation needs to remain equal would help the student understand and remember that he needs to alter both sides each time he performs an operation.

How well did you know this?
1
Not at all
2
3
4
5
Perfectly
260
Q

Which of the following is NOT an example of augmentative and alternative communication (AAC)?

A. American Sign Language (ASL)

B. picture boards

C. cochlear implants

D. text-to-speech software applications

A

C. cochlear implants

Cochlear implants are electronic devices worn in and behind the ear that stimulate the cochlear nerve, allowing people with hearing loss to hear and interpret sound. They are not considered AAC devices, since they do not aid in the wearer’s ability to express ideas or needs to others.

How well did you know this?
1
Not at all
2
3
4
5
Perfectly
261
Q

Mrs. Gonzalez is a special education teacher assigned to co-teach with the sophomore English language arts team. At the team’s first common planning time meeting, team members are establishing norms and choosing which of the shared responsibilities they will take on. Which of the following roles would be the most logical fit for Mrs. Gonzalez?

A. make recordings of all lessons to post in virtual classrooms for students to review

B. convert all assessments to a digital platform for more flexible administration of tests

C. change the pace of instruction to better fit the needs of students with disabilities

D. make needed accommodations to assignments and tests

A

D. make needed accommodations to assignments and tests

The special education teacher who partners with an instructional team should be the group’s subject-matter expert on how to accommodate or modify assignments and tests.

How well did you know this?
1
Not at all
2
3
4
5
Perfectly
262
Q

A formative assessment is intended to:

A. determine the efficacy of a method of instruction.

B. assess a student’s overall developmental capability.

C. measure learning at various incremental points within a larger unit of study.

D. assess a student’s progress when compared to that of non-disabled peers.

A

C. measure learning at various incremental points within a larger unit of study.

Formative assessments are intended to monitor student learning to provide ongoing feedback as the class progresses through a unit of study.

How well did you know this?
1
Not at all
2
3
4
5
Perfectly
263
Q

Which organization below specifically provides information to parents, communities, educators and the general public on specific disabilities; programs and services for infants, children, and youth; U.S. special education law; and effective educational practices?

A. Council for Exceptional Children (CEC)

B. American Association of People with Disabilities (AAPD)

C. The American Association on Intellectual and Developmental Disabilities (AAIDD)

D. The National Dissemination Center for Children with Disabilities (NICHCY)

A

D. The National Dissemination Center for Children with Disabilities (NICHCY)

The National Dissemination Center for Children with Disabilities (NICHCY) provides information to parents, communities, educators and the general public on specific disabilities; programs and services for infants, children and youth; U.S. special education law; and effective educational practices.

AAPD advocates for full civil rights for the over 60 million Americans with disabilities by promoting equal opportunity, economic power, independent living, and political participation.

How well did you know this?
1
Not at all
2
3
4
5
Perfectly
264
Q

Jacob is a non-verbal fourth-grade student with a severe intellectual disability. Which of the following types of assistive technology would likely be most beneficial for Jacob and his teachers?

A. speech-to-text software

B. text-to-speech software

C. a picture exchange communication system (PECS)

D. an electronic dictionary

A

C. a picture exchange communication system (PECS)

The picture exchange communication system (PECS) is an alternative communication system that uses picture cards to convey wants, needs, questions, or thoughts to another person using a series of six phases. It is an effective way to help nonverbal students to communicate.

265
Q

Which of the following strategies would be most effective in helping a fifth-grade student with dyslexia learn to decode words?

A. skipping difficult words and coming back to them later

B. previewing academic vocabulary before beginning a new unit of study

C. consistently reviewing and referring to the phonic alphabetic code chart

D. writing a summary of text passages after reading them

A

C. consistently reviewing and referring to the phonic alphabetic code chart

The phonic alphabetic code chart is a graphic breakdown of the 44 speech sounds (called phonemes) of the English language. The chart explains which letters combine to create which sounds. A review of this chart before or during reading can help students with dyslexia remember how to break difficult words down into their smallest pieces to decode and better understand them.

266
Q

Which of the following is NOT an important consideration when determining the least restrictive environment for a student with attention-deficit/hyperactivity disorder (ADHD)?

A. the student’s social and emotional skill level

B. the extent that the student’s disability impacts his ability to learn

C. access to the general education curriculum

D. the student’s effect on peers

A

D. the student’s effect on peers

All admission, review, and dismissal (ARD) committee decisions should be made with the focus aligned to providing the student as much access as possible to the general education curriculum with non-disabled peers, regardless of the student’s impact on others. It is the responsibility of the local education agency (LEA) to accommodate and manage the student’s behaviors and abilities while providing a free and public education (FAPE).

267
Q
A
268
Q

A fifth-grade student with a specific learning disability (SLD) in written expression would likely benefit most from which of the following supports when completing an assigned essay?

A. a peer partner to help edit and revise the essay’s first draft

B. darkly lined paper

C. text-to-speech software to read the essay prompt aloud

D. a graphic organizer with sentence stems and transition sentences related to the essay’s topic already printed

A

D. a graphic organizer with sentence stems and transition sentences related to the essay’s topic already printed

Students who struggle with written expression often have trouble organizing their thoughts and translating them to the written word. A graphic organizer with sentence stems and a selection of common transition phrases can help these students see how the essay should be organized and give them support in making their ideas fit the essay prompt.

269
Q

When the parent of a school-aged student expresses concerns about their student’s lack of academic progress, and a disability is suspected, the first step of the process is for the district to obtain:

A. all assessment and evaluation data from any previous district.

B. results from a range of assessments administered by a diagnostician to determine if the student suffers from a disability.

C. signed written consent for evaluation from the parents.

D. formal reports from the student’s teachers detailing observed strengths and weaknesses.

A

C. signed written consent for evaluation from the parents.

The first step in the process for completing an initial evaluation for special education is to get signed consent from the parent(s) giving the local education agency (LEA) permission to evaluate the student.

270
Q

Cody is a sixth-grade student with a moderate intellectual disability who performs below grade level in language arts and math. He struggles with executive function, impulse control, and work completion, but has shown that he can be academically successful with close supervision and redirection. Which of the following placements would be most appropriate for Cody?

A. co-taught classes for language arts and math and general-education classes for all other subjects

B. resource for language arts and math and general-education classes for other core subjects and electives with paraprofessional support in all classes

C. self-contained classes for all core subjects and general-education classes for electives to provide socialization opportunities

D. general-education classes for all subjects with an assigned peer partner for reminders of expectations and redirection to tasks

A

B. resource for language arts and math and general-education classes for other core subjects and electives with paraprofessional support in all classes

Since Cody has demonstrated success under close supervision with redirection, paraprofessional support throughout his school day is appropriate. Additionally, resource placement for language arts and math is appropriate since he performs below grade level in both subjects.

While the support available in co-taught classrooms may be appropriate for Cody, he likely needs more support than can be offered in the general-education setting.

271
Q

Sometimes reading difficulties are known not to be the outcome of lack of opportunities in education or limited intellect. Recent educational research has found that these difficulties are most often the result of what instead?

A. lack of skills in phonological processing

B. background in another language

C. not being read to daily as a young child

D. lack of being motivated

A

A. lack of skills in phonological processing

Research has shown that reading difficulties most often occur due to the lack of skills in phonological processing after other factors have been ruled out including limited intellect or poor educational opportunities.

272
Q

Jackson is a graduating senior with autism spectrum disorder (ASD) who will soon begin a new job shelving books and helping to maintain records at the local library. Which of the following is the best way Jackson’s special education case manager can support him in preparing for this transition from school to work?

A. Review Jackson’s existing IEP accommodations with him to analyze which supports would be feasible and helpful in his new position.

B. Engage in role-play interactions to help Jackson practice for his job interview.

C. Assign Jackson a number of classic novels to read over the next several weeks to make him more knowledgeable in his area of employment.

D. Emphasize to Jackson that he is transitioning away from academics and, therefore, no longer needs the academic supports in his IEP.

A

A. Review Jackson’s existing IEP accommodations with him to analyze which supports would be feasible and helpful in his new position.

It is important to make sure that transitioning students with disabilities understand the supports that can help them succeed in daily life. It is also beneficial to give these students a chance to practice advocating for themselves in the workplace through role-play and direct teaching.

273
Q

Which of the following strategies would be most effective in helping a struggling reader in first grade apply rules of phonemic awareness to decode new words?

A. Have the student read silently to allow them to practice without the pressure of reading aloud in class.

B. Provide the student with picture cards to match to simple words.

C. Group words according to consonant blends or interior vowel sounds, starting with words the student knows and progressing to new words which follow the same patterns.

D. Allow the student to draw a picture representing words she is unable to write or correctly pronounce.

A

C. Group words according to consonant blends or interior vowel sounds, starting with words the student knows and progressing to new words which follow the same patterns.

Starting with a known word and allowing the student to explore new words that follow a similar pattern can help build phonemic awareness and decoding skills.

274
Q

Ms. Hernandez is a second-grade inclusion teacher who will be meeting with her students’ parents for back-to-school night. Ms. Hernandez will have the opportunity to share a brief presentation regarding topics of study this school year. She has asked each family to fill out an index card with key information. Which of the following items would be important for her to ask parents to include?

A. parents/guardians’ supervisor(s) at work

B. parents/guardians’ relation to children

C. student’s preferred name or nickname

D. parents/guardians’ preferred contact number during the school day

A

D. parents/guardians’ preferred contact number during the school day

It is important to have contact number(s) for parents/guardians on file and know which numbers are best to reach them during the school day.

If a student uses another name or nickname, this is a question that the teacher can ask students directly.

275
Q

All of the following topics would be appropriate for discussion and consideration during an admission, review, and dismissal (ARD) committee meeting EXCEPT:

A. the student’s history of performance on various state assessments

B. the student’s relationship with his teachers and other school employees

C. the student’s interactions with specified classmates who have similar IEPs

D. the student’s behavior during transitions between classes and unstructured periods like lunch or recess

A

C. the student’s interactions with specified classmates who have similar IEPs

Discussion of other students, disabled or otherwise, is not allowed during admission, review, and dismissal (ARD) committee meetings.

276
Q

Ms. Nakaroti wants to teach her students about properties of points, lines, planes, and angles. Which of the following should she include in her planning for the unit?

A. Make sure students write an essay that uses all of the key terms as their summative assessment.

B. Ensure that students are engaged in group work every single instructional day.

C. Analyze the standards to determine learning objectives before she starts writing lesson plans.

D. Use google to find worksheets about the topic.

A

C. Analyze the standards to determine learning objectives before she starts writing lesson plans.

The standards should always be consulted before beginning lesson planning.

277
Q

Matthew is a seventh-grade student who is suffering from an emotional disturbance following a recent traumatic experience that resulted in several physical injuries. His initial IEP meeting is scheduled for later this month, and the admission, review, and dismissal (ARD) committee is requesting behavior data from Matthew’s general education teachers. Which of the following behaviors is likely related to Matthew’s disability?

A. avoidance of physical contact and a strong desire to control all interactions with others

B. repetitive physical movements and perseverated focus on one topic or area of interest

C. lack of fine motor control resulting in illegible printing and writing

D. lack of emotional affect when reading aloud in class

A

A. avoidance of physical contact and a strong desire to control all interactions with others

Students who have recently experienced trauma with physical injuries may find physical contact with others uncomfortable, especially when it is unexpected. Additionally, students in states of high anxiety often seek stasis by seeking to control all aspects of their interactions with others.

278
Q

Direct instruction in the meaning and usage of various prefixes and suffixes would most likely help a student who is struggling with which of the following skills?

A. summarizing expository text

B. diagramming a narrative plot

C. understanding figurative language

D. decoding and recognizing words

A

D. decoding and recognizing words

Intensive study of prefixes and suffixes is most helpful for students who struggle to decode and understand new words in context.

279
Q

A third-grade student is struggling with rounding numbers to the nearest hundred. Which of the following is the best first step for his teacher to take?

A. teach the student a poem about the steps for rounding that they can use as a memory aid

B. having the student plot numbers on a number line and identify the closest hundred value

C. contact the student’s parents and send additional practice home to prevent the student from falling behind

D. temporarily stop instruction on rounding and focus on topics that the student is successful at in order to improve their confidence level

A

B. having the student plot numbers on a number line and identify the closest hundred value

This is an effective strategy to use because it will provide the student with a visual representation of rounding.

280
Q

Which of the following supports would be most beneficial when introducing new vocabulary to a student with a mild intellectual disability?

A. Provide an image to help explain each new vocabulary word.

B. Have the student locate the words in a dictionary and copy down the appropriate definition for each.

C. Have the student copy the words and definitions three times each.

D. Provide a synonym and antonym for each new vocabulary word.

A

A. Provide an image to help explain each new vocabulary word.

Students with intellectual disabilities often need concrete examples to help them absorb and understand new concepts and ideas. Providing images that help demonstrate the meaning of new vocabulary will help connect the words with their proper meanings and give students a better chance of recalling and using the word correctly in the future.

281
Q

The Texas Education Code (TEC) sets legal standards for all schools in the state of Texas. What schools are required to abide by these laws?

A. any public or private school that provides instructional services for students with disabilities

B. any public school that receives taxpayer funds

C. any public school that has students receiving instructional services for students with disabilities

D. any public or private school in Texas

A

B. any public school that receives taxpayer funds

All public schools receive taxpayer funds and must abide by the TEC.

Only public schools are required to abide by the TEC.

282
Q

A fourth-grade teacher wants to provide realistic experiences in the understanding and development of graphic representations. After she models the development of a class birthday month bar chart, the teacher assigns her students to choose a question to investigate in order to develop their own bar chart. Which of the following questions would be least appropriate for students to investigate for the purpose of developing a graph?

A. What color of cars do your parents drive?

B. What do you like best about school?

C. What number of students play basketball vs. soccer?

D. In which zip code do you live?

A

B. What do you like best about school?

The answers to this question would not translate into concrete data that would fit into a bar chart.

283
Q

Mrs. Latham is a tenth-grade English teacher with multiple inclusion classes. Which of the following reading strategies Mrs. Latham uses in class is most likely to be beneficial to students in other content areas?

A. annotating text to identify key information and concepts

B. decoding figurative language to help analyze poetry

C. diagramming the plot of a narrative passage

D. evaluating stage directions in a drama to understand the tone or mood the playwright intended

A

A. annotating text to identify key information and concepts

Text annotation is a valuable and flexible strategy for quickly identifying main ideas in any text across all academic disciplines.

284
Q

Mrs. Latham is a tenth-grade English teacher with multiple inclusion classes. Which of the following reading strategies Mrs. Latham uses in class is most likely to be beneficial to students in other content areas?

A. annotating text to identify key information and concepts

B. decoding figurative language to help analyze poetry

C. diagramming the plot of a narrative passage

D. evaluating stage directions in a drama to understand the tone or mood the playwright intended

A

A. annotating text to identify key information and concepts

Text annotation is a valuable and flexible strategy for quickly identifying main ideas in any text across all academic disciplines.

285
Q

Which of the following would NOT be considered assistive technology for a student with a visual impairment?

A. magnifying prism

B. calculation aids

C. text-to-speech software

D. large print books and assignments

A

B. calculation aids

A student with a visual impairment would not benefit from the use of a calculator unless that student also has a qualifying disability related to math calculation.

286
Q

Kayla is a fifth-grade student who is in the evaluation phase of an initial referral for special education services. Her parents have expressed frustration with her inability to follow instructions when completing chores at home. Her teachers report that she is easily distracted and exhibits impulsive actions in class, although she is respectful and easily redirected. Kayla is undergoing diagnostic testing with a district representative. In the meantime, which of the following actions by the special education teacher assigned to the admission, review, and dismissal (ARD) committee would be most helpful?

A. Provide Kayla’s parents and teachers with some strategies for assisting children with autism spectrum disorder (ASD) in learning and applying appropriate social behaviors.

B. Provide Kayla’s parents and teachers with some strategies for helping children with a specific learning disability (SLD) in reading comprehension to better understand written and verbal instructions.

C. Provide Kayla’s parents and teachers with some strategies for improving focus and impulse control in children with attention-deficit hyperactivity disorder (ADHD).

D. Provide Kayla’s parents and teachers with some strategies for helping children with emotional disturbance control and manage anxiety and frustration.

A

C. Provide Kayla’s parents and teachers with some strategies for improving focus and impulse control in children with attention-deficit hyperactivity disorder (ADHD).

Based on the information available, it would be appropriate to give Kayla’s caregivers and instructors information that could help manage some of the behaviors she exhibits, even if the official diagnosis has not been completed. It is important to note that this is not the same as implementing academic accommodations or modifications or implementing a behavior intervention plan (BIP), neither of which should be done until after the IEP meeting. Additionally, staff should avoid explicitly telling Kayla’s parents that they suspect she has ADHD until testing is complete and a diagnosis is available.

287
Q

A second-grade reading teacher plans to regularly incorporate all of the skills that support reading comprehension. For an upcoming reading activity, the teacher has marked several locations in the text. When the students reach each of the pre-marked locations, they will stop and turn to their partner to make a comment, ask a question, make a connection, or make a prediction. Which reading comprehension skill is the teacher supporting with this activity?

A. self-monitoring or metacognition

B. recognizing characteristics of specific genres

C. building background knowledge

D. reading fluently

A

A. self-monitoring or metacognition

This activity requires students to check in periodically and think about what they are reading. This will help them self-monitor their comprehension.

288
Q

Which of the following would be most helpful for a first-grade student who is having difficulty understanding how letters combine to form words that represent real objects?

A. practicing consonant blends to increase fluency

B. using cards that pair three-letter words with pictures of the things or concepts the words represent

C. copying new vocabulary words three times each

D. reading aloud to the student

A

B. using cards that pair three-letter words with pictures of the things or concepts the words represent

One of the foundational steps for developing readers is realizing that letters form words that represent real-world objects and actions. Providing a graphic representation of these words can help solidify the connection between written and spoken language.

289
Q

When preparing for a student’s annual IEP meeting, the special education teacher should be sure to solicit input from:

A. only general education teachers in state-tested subjects.

B. all general education teachers who work with the student, regardless of subject taught.

C. only elective teachers, since assessment data is available for core subjects.

D. only general education teachers in whose subject the student has IEP goals.

A

B. all general education teachers who work with the student, regardless of subject taught.

When preparing for an IEP meeting, it is important to gather data and reports from all teachers who have a part in implementing the student’s IEP. The student may behave and perform differently in different classes and situations, and each teacher may be able to provide valuable insight to the admission, review, and dismissal (ARD) committee.

290
Q

Which of the following is an important component of choral reading?

Select all answers that apply.

A. participating in a whole-class discussion after reading the text

B. hearing the teacher model oral reading

C. marking confusing words while listening to the text

D. reading aloud

A

B. hearing the teacher model oral reading

Hearing a story read fluently by the teacher is a component of choral reading. Hearing the story or sentences read by the teacher first, before a student reads it aloud independently, can improve reading fluency.

D. reading aloud

Reading aloud is a component of choral reading, and it is important to help build reading fluency.

291
Q

Kate, an eighth-grade student with attention-deficit hyperactivity disorder (ADHD), struggles with organization and planning in her life outside of school. She is able to complete and submit most in-class assignments with little trouble, but when it comes to longer projects, she often misses deadlines or forgets to complete her work. Kate’s district provides all students with tablets, which can be utilized off-campus. Which of the following supports discussed in her annual IEP meeting would be most helpful for Kate as she transitions to high school?

A. a peer partner to help Kate keep track of assignments and projects

B. a binder with dividers for each of her classes

C. a printed planner with daily, weekly, and monthly calendars

D. An electronic calendar that coordinates with the district’s learning management system (LMS)

A

D. An electronic calendar that coordinates with the district’s learning management system (LMS)

A calendar that syncs with assignments posted in the district’s LMS would give Kate one location to check on and keep track of which work is due in which class, without requiring extra steps on her part.

292
Q

A fourth-grade teacher in a language arts class uses the following assessments in class:

An early, mid-year, and final measure of the number of words students can read correctly in 60 seconds.

A rating scale for emotional affect displayed when students read aloud during any activity.

Charting student attention to punctuation while reading aloud, both before and after direct instruction on the topic.

The data collected from these assessments would be most appropriate in measuring progress toward an IEP goal in which of the following areas?

A. understanding and using newly acquired vocabulary

B. reading fluency

C. reading comprehension

D. phonemic awareness

A

B. reading fluency

Reading fluency is a measure of a student’s reading rate combined with their awareness of punctuation and emotional affect when reading aloud. Reading with grade-level fluency is a fundamental step toward developing stronger reading comprehension.

293
Q

Which of these is a primary benefit of placement in a resource classroom?

A. The classes tend to be smaller, reducing the chance of distraction and behavior issues.

B. All instruction is tailored to students with special needs.

C. There are no non-disabled peers, so students in a resource class can be themselves.

D. The teacher in a resource room is not required to align instruction to state curriculum standards.

A

B. All instruction is tailored to students with special needs.

A resource class is typically comprised of a small number of students with special needs who function at similar levels of academic achievement. This allows the instructor to focus more narrowly on specific skill development unique to special education populations.

294
Q

Jeremy is a third-grade student with autism spectrum disorder (ASD). Jeremy’s teachers report that he has begun disrupting class by loudly repeating the things that others say, until he is sent to a behavior redirection classroom, where he is allowed to work on a jigsaw puzzle to calm down. This behavior began shortly after Jeremy’s most recent IEP meeting, where the redirection room was added to his IEP as a behavior support. The most likely explanation for the change in Jeremy’s behavior is that:

A. he uses oral repetition as a strategy to help him better understand what others are saying.

B. he has learned that exhibiting disruptive behaviors in class will lead to the opportunity for a preferred activity in a relaxing environment.

C. he is bored by the level of instruction in the classroom.

D. he is frustrated by an inability to complete his assigned tasks.

A

B. he has learned that exhibiting disruptive behaviors in class will lead to the opportunity for a preferred activity in a relaxing environment.

It is important to remember that all behavior serves a purpose. If Jeremy prefers working on a puzzle to participating in class, he will likely continue to do the things that lead to the outcome he wants. A brief admission, review, and dismissal (ARD) committee meeting or a conference with Jeremy’s case manager may be useful in readjusting how the redirection room will be implemented.

295
Q

A third-grade teacher notices that some of her students are struggling with putting fractions in order from least to greatest. What would be the most appropriate manipulative to help students with this concept?

A. pattern blocks

B. base ten blocks

C. unifix cubes

D. fraction strips

A

D. fraction strips

Fraction strips provide students with a concrete representation of what each fraction looks like. By using these to model different fractions, students can see that certain fractions are larger or smaller than others.

296
Q

Which student profiled below would more than likely qualify for Section 504 of the Rehabilitation Act?

A. a student who needs a modified curriculum

B. a student who needs related services

C. a student who needs a behavior contract

D. a student who is not making academic progress with their current in-class support schedule

A

B. a student who needs related services

Section 504 of the Rehabilitation Act offers related services.

297
Q

Which of the following mathematics lessons would be most helpful in preparing a high school resource class for independent living?

A. a lesson on how to calculate the volume of a cylinder

B. a lesson on managing a personal checking account and paying bills

C. a lesson on how to graph the slope of an equation

D. a lesson on how to calculate a vehicle’s gas mileage

A

B. a lesson on managing a personal checking account and paying bills

Managing money and paying bills are important skills of daily living for students who want to live independently.

298
Q

Aryanna is a fourth-grade student with a learning disability. Recent assessment data shows that Aryanna is able to find the perimeter of rectangles but struggles to find the area. When asked to calculate area, she typically adds the length and width rather than multiplying. Which of the following interventions would best help Aryanna?

A. teaching Aryanna a mnemonic device to remember the formula for the area of a rectangle

B. providing small-group instruction in which square tiles are used to cover the surface of a rectangle

C. providing Aryanna with a formula sheet to use on assignments and tests

D. asking the campus math interventionist to provide pull-out support for Aryanna to ensure that she does not fall behind

A

B. providing small-group instruction in which square tiles are used to cover the surface of a rectangle

Based on the information provided in the question, it is likely that Aryanna is lacking a concrete understanding of the concept of area. Using tiles to cover the surface of a rectangle will help reinforce this concept.

Aryanna appears to have a misunderstanding of the concept of area. While a mnemonic device may help her remember the formula, it will not improve Aryanna’s conceptual understanding of area.

299
Q

Madison is a second-grade student who was recently diagnosed with a mild intellectual disability. Even though she has been educated in the general education classroom for the last two years with no issues aside from occasionally needing more academic support, during her initial IEP meeting, her parents express concern that she won’t receive the instruction she needs unless she is moved to a self-contained classroom. How should the admission, review, and dismissal (ARD) committee respond to Madison’s parents’ concerns?

A. Compromise by moving Madison to a resource class for math and reading with general education minutes scheduled in other core classes and electives for socialization with non-disabled peers.

B. Assign a paraprofessional to provide one-to-one support for Madison during the entire school day.

C. Emphasize the benefits of the general education classroom for students with disabilities and highlight the flexibility offered by the co-taught model of instruction.

D. Change Madison’s placement to a self-contained classroom for one grading period to see if she continues to make academic progress at her current rate or regresses.

A

C. Emphasize the benefits of the general education classroom for students with disabilities and highlight the flexibility offered by the co-taught model of instruction.

Parent concerns should always be treated with respect, but the admission, review, and dismissal (ARD) committee also has a responsibility to provide accurate information regarding the least restrictive environment. Based on the evidence presented, Madison is able to function in the general education classroom and would benefit from maintaining that placement with a co-teacher added.

300
Q

Which of the following skills are likely to be improved by regular work with an occupational therapist?

A. running long distances under time restraints

B. developing a comfortable and functional pencil grip

C. correcting tongue placement for specific vocalizations

D. learning to initiate and maintain a conversation with a peer

A

B. developing a comfortable and functional pencil grip

Occupational therapists work with individuals who need to develop, improve, or maintain skills needed for daily living, such as developing a comfortable, functional pencil grip.

301
Q

A teacher is reading a unique, large print text to her class. As she turns the pages, she points to the words and identifies places in which the text is larger or smaller and other pages that have the text oriented vertically and upside down. By pointing out these irregularities, the teacher is building student’s understanding of:

A. print concepts.

B. word recognition skills.

C. the alphabetic principle.

D. prosody.

A

A. print concepts.

Print concepts are the ideas that text runs left to right, top to bottom, and is made of numbers, letters, and punctuation. By pointing out how this book plays with print concepts, the teacher can discuss what is normal and what is unique in this text.

302
Q

Mr. Etsel is a new teacher in seventh grade, and he received a questionnaire about Carson, who was recently diagnosed with Attention Deficit Hyperactivity Disorder (ADHD) by his pediatrician. The special education team is doing a Full and Individual Evaluation (FIE), and part of the evaluation includes gathering data from Carson’s teachers. Mr. Etsel does not understand why additional information is needed. What can the special education teacher tell him?

A. ADHD is not a qualifying disability under federal law unless diagnosed by school personnel.

B. The FIE is just a formality, so Mr. Etzel can just answer a few of the questions and turn it into the special education teacher.

C. The reasons for conducting the FIE are confidential and Mr. Etzel must comply, but he cannot know anything further.

D. A formal evaluation is required by law, and it will be used to determine eligibility for special education services.

A

D. A formal evaluation is required by law, and it will be used to determine eligibility for special education services.

ADHD is a type of health impairment. If a health impairment substantially limits one or more major life activities and has a significant educational impact, a student may be eligible for special education services. Students must undergo an FIE to determine if their disability causes a significant educational impact which requires special education and/or related services.

303
Q

A third-grade student who is emotionally disturbed is in an inclusion classroom. The child has mastered the goals set forth in the Behavioral Intervention Plan (BIP), however, other negative behaviors have developed. What should be the first step the special education teacher will take?

A. Call the parent to see if there have been changes at home.

B. Enforce new consequences to see if the new behaviors can be fixed in the classroom.

C. Request an Admission, Review, and Dismissal (ARD) meeting to set new behavior goals.

D. Request a functional behavior assessment (FBA) to address the causes of the new behaviors.

A

D. Request a functional behavior assessment (FBA) to address the causes of the new behaviors.

This will be the basis for moving forward because once the committee knows why the behavior occurs, they can help correct it.

The teacher has likely tried to redirect and treats this student like any other. New consequences will not be the answer until the motivation for the behaviors is discovered.

304
Q

A first-grade teacher receives a new student who has a diagnosis of autism. He is supported by special education services but spends most of his time in the general education classroom. His current reading level is unknown and he often loses interest in independent work, choosing to draw instead. He also frequently sits by himself at lunch and recess and does not participate in group activities.

Which of the following would be an appropriate step for the general education teacher to improve the student’s interactions with his classmates?

A. providing a lunch-time social skills group with the special education teacher

B. speak to the student’s parents about his isolation during group activities

C. re-arranging the desks in the classroom often to encourage new collaborations

D. encouraging the student to talk to his classmates

A

A. providing a lunch-time social skills group with the special education teacher

By providing an opportunity to directly teach social skills curriculum, the teacher is providing an evidence-based intervention to improve the student’s social interactions.

While classroom arrangement can have an effect on student collaboration, direct teaching in social skills curriculum would be a more effective means for the student in question.

305
Q

A first-grade teacher receives a new student who has a diagnosis of autism. He is supported by special education services but spends most of his time in the general education classroom. His current reading level is unknown and he often loses interest in independent work, choosing to draw instead. He also frequently sits by himself at lunch and recess and does not participate in group activities.

Which classroom accommodations would be most appropriate to assist this student?

A. placing the student next to the teacher’s desk

B. allowing the student to draw during independent work

C. creating a separate area of the classroom for the student to work in

D. providing an incentive system for completing independent work

A

D. providing an incentive system for completing independent work

Providing a reinforcement system for independent work is likely to increase the student’s likelihood of completing the work is an appropriate accommodation.

306
Q

A first-grade teacher receives a new student who has a diagnosis of autism. He is supported by special education services but spends most of his time in the general education classroom. His current reading level is unknown and he often loses interest in independent work, choosing to draw instead. He also frequently sits by himself at lunch and recess and does not participate in group activities.

Which of the following would be an important first step in determining the student’s reading level?

A. start the student at level 1 of the reading intervention program

B. search the internet for autism-specific reading assessments

C. collaborating with the special education teacher to administer a reading assessment

D. observe the student in the library to see what he chooses to read

A

C. collaborating with the special education teacher to administer a reading assessment

By assessing the student’s current reading level in collaboration with the special education teacher, the teachers can build an appropriate intervention plan for the student across classrooms.

307
Q

A special education teacher is having difficulty with a sixth-grade math group. The students are frequently off-task and often do not complete their work correctly. One strategy the teacher can implement to improve student behavior is:

A. send the students back to their general education classroom when they engage in off-task behavior

B. have an aide prepare extra work for the students who do not complete their in-class tasks

C. write a note home to each of the student’s guardians informing them of their behavior

D. create a tally system for focus and correct work that can be exchanged for in-class privileges

A

D. create a tally system for focus and correct work that can be exchanged for in-class privileges

Group reinforcement contingencies have been shown to improve on-task behaviors in individuals with disabilities. By creating an incentive system, the classroom teacher can build a history of reinforcement with the students and increase the likelihood for them completing their tasks in her group.

308
Q

A portable device that is programmed to verbalize words or short phrases for the user is an example of a(an):

A. augmentative communication device.

B. blissymbols.

C. personal amplifier.

D. portable word processor.

A

A. augmentative communication device.

An augmentative communication device may be used by an individual with limited language skills to communicate his/her wants and needs. Some devices can be customized for specific activities (such as lunch) or specific content areas.

309
Q

Which of these behaviors would be concerning when evaluating a 3-year-old’s speech/language development?

A. speaking with a lisp

B. using 2-3 word phrases to answer questions

C. substituting “r” sounds with “w” sounds

D. identifying 5 out of 10 common objects correctly

A

D. identifying 5 out of 10 common objects correctly

At age three, children should be able to identify 10 out 10 common objects. This may indicate a delay in expressive language development.

A lisp is common at this age and is not an indication of articulation concerns.

310
Q

Jackie is a high school student with severe anxiety. Her intelligence falls into the high average range, and she excels in language arts. Her anxiety has begun impacting her ability to complete assignments, as she fears that she has misunderstood directions. Which of these accommodations could help to ease her anxieties in the general classroom?

A. providing preferential seating near the front of the room

B. providing large print directions on a separate page from the assignment

C. providing written and oral directions for assignments and checking for understanding of the directions

D. providing rewards for completed assignments

A

C. providing written and oral directions for assignments and checking for understanding of the directions

These tools allow her to feel confident that she is completing assignments correctly and provides her with an opportunity to ask clarifying questions as needed.

311
Q

Which of the following tasks would provide the truest assessment of student learning after introducing a new mathematics concept?

A. Have students write their own word problem related to the concept and require them to provide an answer key with modeling of step-by-step processes to solve the problem.

B. Have students complete a three-question multiple-choice quiz related to the concept.

C. Have students come to the board one at a time to solve problems related to the concept.

D. Have students discuss the concept with peer partners and verbally quiz one another over the key elements.

A

A. Have students write their own word probrelem related to the concept and require them to provide an answer key with modeling of step-by-step processes to solve the problem.

The ability to synthesize a problem situation with a detailed solution is a strong indicator of student mastery of a concept, with the added benefit of being able to use the best submissions to create assignments for the class to practice later on.

312
Q

Which of the following would be an appropriate summative assessment for fourth grade students who are learning about parts of speech?

A. asking students to submit an exit ticket about parts of speech to guide the next day’s instruction

B. guiding the students through labeling a different part of speech on a sentence at the beginning of each class

C. having students independently label each part of speech for words in a given sentence

D. inviting students to come to the board to identify parts of speech together

A

C. having students independently label each part of speech for words in a given sentence

A summative assessment individually tests students’ knowledge of a subject. Asking students to label parts of speech independently is a summative assessment which can be used to determine whether students grasp the concept of parts of speech.

313
Q

A high school student with intellectual disabilities has expressed an interest in working at a clothing store. He would like to stock shelves and help with displays in the store. Which of these transition activities would provide the best training for him to prepare for employment at a store?

A, working alongside typical students in the school’s spirit store, selling hats and shirts

B. developing a visual checklist for different tasks he may need to complete each week

C. helping him find a job at a local retail store where the teacher can check on him weekly

D. taking a fashion design class through the career center in his school district

A

A, working alongside typical students in the school’s spirit store, selling hats and shirts

This provides an opportunity to work and learn from others in a retail setting. The teacher will have the opportunity to supervise and collect data to identify additional interests or needs.

314
Q

A special education teacher has three students in a fourth-grade reading group who are frequently distracted during tests. The students often bubble incorrectly during testing and do not finish in the allotted time. The special education teacher should:

A. explain to the students that they must finish their test in time.

B. provide accommodations to the student on the next exam.

C. consider providing testing accommodations for the students at their next IEP meeting.

D. require the students to test in a one-on-one environment.

A

C. consider providing testing accommodations for the students at their next IEP meeting.

Providing testing accommodations for students can assist with issues in test completion. However, testing accommodations have to be agreed upon in an IEP meeting before being implemented.

315
Q

An eighth-grade student with a specific learning disability in reading has 45 minutes of reading instruction in the general education classroom without in-class support. What strategies can the general education teacher implement to promote the student’s active participation in the reading curriculum?

A. Assign independent reading tasks to the student during reading instruction.

B. Ask the special education teacher for more in-class support.

C. Allow the student to only read preferred books in the general education class.

D. Provide a guided reading group with teacher support during independent reading time.

A

D. Provide a guided reading group with teacher support during independent reading time.

A small group environment with direct instruction is an effective way to provide support for this student. Allowing the group to contain similar-level peers or peer models can build confidence and self-advocacy with this student.

316
Q

Due to reading comprehension deficits, Joel has been in a resource classroom for language arts since first grade. He is beginning fifth grade this fall, and his teachers feel that he is ready to move into the general education setting with support. What is the best method to ensure his success?

A. Provide a special education co-teacher in language arts who provides the same accommodations that he received in the resource room.

B. Provide a paraprofessional to sit at a desk with him and keep him on task.

C. Provide Joel with the opportunity to leave the general ed classroom to visit the resource room when anxious or needing help.

D. Provide a peer tutor that can assist when Joel is feeling frustrated or behind.

A

A. Provide a special education co-teacher in language arts who provides the same accommodations that he received in the resource room.

This will allow for a smooth transition, as the co-teacher will continue to implement Joel’s IEP with familiar and effective supports.

317
Q

A special education teacher is providing in-class support in a second-grade writing classroom. Four students require special education services in writing, while the rest of the class does not. What strategies should the special education teacher use to implement instruction in this setting?

A. Allow the general education teacher to teach the lesson and observe.

B. Coordinate with the general education teacher to adapt the lesson plans to the student’s individual needs.

C. Simultaneously teach a writing lesson to the four students in another area of the classroom.

D. Teach the remaining students the writing lesson while the general education teacher works with the four students.

A

B. Coordinate with the general education teacher to adapt the lesson plans to the student’s individual needs.

Collaboration with the general education teacher allows for the writing curriculum to be effectively adapted to the student’s needs in their least restrictive environment.

318
Q

Raquel has an orthopedic impairment that causes fine motor deficits. Raquel’s testing accommodations include a human scribe. Which of the following is a reason for this accommodation?

A. A human scribe can give Raquel opportunities to take breaks.

B. A human scribe allows Raquel to finish the test without needing extended time.

C. A human scribe can provide prompts to help Raquel develop her response.

D. A human scribe can record Raquel’s responses legibly.

A

D. A human scribe can record Raquel’s responses legibly.

A student with an orthopedic impairment and fine motor deficits may have limited use of her hands and may struggle to write legibly. A human scribe records the student’s responses legibly so that written responses can be scored accurately.

319
Q

Mr. Ortiz is a paraprofessional that provides in-class support for second-grade students with emotional disabilities. He has two students with Behavior Intervention Plans (BIPs) that center around work completion. One component of the plan is that the students will receive 5 minutes of free choice time if they complete work independently. Mr. Ortiz often prompts the students to begin working and reminds them of their free choice activities, but he doesn’t consistently follow through with providing free choice options before moving onto the next activity.

How can his supervisor help with the implementation of the BIPs?

A. Keep data that shows the number of times Mr. Ortiz offers free choice but moves onto the next assignment instead.

B. Reassign BIP implementation to the second-grade teacher because Mr. Ortiz should not be responsible for the implementation of IEP and BIP goals.

C. Explain the effects of assigning consequences and not following through on them, especially with students who have behavior concerns.

D. Threaten Mr. Ortiz with disciplinary action as he is not following the BIP, which is a legal document.

A

C. Explain the effects of assigning consequences and not following through on them, especially with students who have behavior concerns.

This should help Mr. Ortiz understand that the students’ BIPs include natural consequences (students must finish work before moving onto the preferred activity). If Mr. Ortiz does not implement the plan, he is not encouraging work completion but offering the free choice activity.

320
Q

A special education teacher is working with a group of upper elementary students to help them with a lesson on adding and subtracting with decimals. Which of the following activities would be most helpful to teach this lesson to students with disabilities?

A. using bear-shaped counters to help illustrate the concept of subtraction and addition

B. role-playing working in a store and making change for items using play money

C. using place value blocks to illustrate the value of each decimal place

D. creating a worksheet of funny word problems for them to complete with a partner

A

B. role-playing working in a store and making change for items using play money

This activity would be hands-on and show the real-world application of the skill. Additionally, by this age, students would be more familiar with money than with decimal numbers, so it would be a relatable way to illustrate the concept of decimals.

Place value blocks are a helpful way to illustrate place value with whole numbers, but they do not work for decimal values as they cannot be broken down into pieces smaller than one.

321
Q

A parent expresses concerns to a student’s special education teacher that their child is not learning the reading content in his general education reading classroom. In this situation the special education teacher should:

A. provide the parent with information on special education services for reading.

B. schedule a meeting with the parent and general education teacher to outline a plan to support the student in the reading classroom.

C. refer the guardian to the administrator to answer their questions.

D. encourage the general education teacher to focus more on the special education students.

A

B. schedule a meeting with the parent and general education teacher to outline a plan to support the student in the reading classroom.

Scheduling a meeting with both parties can allow the special education teacher to provide resources for the parent and general education teacher. By working collaboratively, the team can provide appropriate support for the student’s individual needs.

322
Q

Mrs. Shin is writing IEP objectives for a second grade student with math deficits. The Texas Essential Knowledge and Skills for math state: “Students develop an understanding of the base-10 place value system and place value concepts. The students’ understanding of base-10 place value includes ideas of counting in units and multiples of thousands, hundreds, tens, and ones and a grasp of number relationships, which students demonstrate in a variety of ways.”

Which of these would be an appropriate goal?

A. The student will read any number less than 1,000.

B. The student will understand place value for hundreds, tens, and ones.

C. The student will know 4 coins, their value, and how to make a dollar with coins.

D. The student will skip count by 10 and 100.

A

B. The student will understand place value for hundreds, tens, and ones.

This modifies what the student is learning, but it is closely aligned with the TEKS. The student will learn similar concepts as other students in second grade, but at a simplified level.

323
Q

Which of the following activities would be the best measure of whether a first-grade student understands the relationship between numbers?

A. ability to rank stacks of counting blocks of varying lengths from smallest to largest

B. ability to add and subtract single-digit numbers

C. ability to place multiple numbers in the appropriate spots on a blank number line

D. ability to count to 100 in multiple formats (ones, fives, tens, etc.)

A

C. ability to place multiple numbers in the appropriate spots on a blank number line

Requiring a student to rank numbers on a blank number line is a strong indicator of whether he understands the relationship between numbers. It is a visual representation of the numbers’ symbolic value and indicates that the student understands what each number represents.

324
Q

A pre-K student engages in repetitive behavior (lining up toys, hand flapping, etc.) and has difficulty when his class schedule changes unexpectedly. This student’s behavior is characteristic of which disability?

A. Intellectual Disability

B. Autism Spectrum Disorder

C. Emotional Behavior Disorder

D. Specific Learning Disability

A

B. Autism Spectrum Disorder

According to the DSM-IV criteria, stereotyped or repetitive behavior is one of the characteristics for an individual diagnosed with autism. Also, individuals with autism often have rigorous routines that also affect their ability to cope with unexpected changes.

325
Q

Which of the following annual goals is most likely to be included in an Individualized Education Program (IEP) for an eleventh grade student who has autism?

A. keeping hands to self while seated

B. reading grade level text at 50 words per minute (wpm)

C. identifying feelings using picture cues

D. appropriately interpreting body language during conversations with peers and adults

A

D. appropriately interpreting body language during conversations with peers and adults

Students with autism often struggle with social interactions. Interpreting body language will help students to interpret and respond to social cues.

326
Q

A special education teacher is finishing up documentation for an upcoming IEP meeting. The IEP will begin shortly before the student turns 14. The special education teacher should:

A. schedule a time to celebrate the student’s upcoming birthday.

B. contact the parents to provide information on upcoming services.

C. provide additional goal support for the student for later grades.

D. begin a primary transition plan for the student using the district and state criteria.

A

D. begin a primary transition plan for the student using the district and state criteria.

Primary transition plans should begin for students prior to their 14th birthday. This involves parent and student interviews to set up plans for future transition.

327
Q

Simplify the expression: 3(10)^2

A. 900

B. 30

C. 300

D. 3

A

C. 300

The 10 must be squared first (order of operations) and then multiplied by 3.

3(10)2 = 3(100) = 300

328
Q

Caroline is a student in first grade who was homeschooled for preschool and kindergarten. Caroline experienced a traumatic brain injury when she was four years old. When she attends recess with her class outside in the first three months of school, Caroline has a difficult time keeping her eyes open. She also struggled to see pictures in a book during small group time in the classroom.

What condition might Caroline have?

A. pink eye

B. photophobia

C. astigmatism

D. glaucoma

A

B. photophobia

Photophobia is an extreme sensitivity to light and can be a symptom of a traumatic brain injury.

329
Q

A second-grade teacher is planning to use an informational text about the accomplishments of Amelia Earhart as an opportunity for her students to practice their summary skills. Prior to independently reading the text, the teacher will show the class a photograph of Earhart in the cockpit of a plane, and the class will watch a short video that overviews the history of aviation. The teacher’s lesson plans demonstrate her understanding of the skills that support reading comprehension. Which of the following best describes her goal for the pre-reading activity?

A. Schema development supports comprehension.

B. Pre-assessing students’ related knowledge allows the teacher to anticipate those that will need scaffolding and support on the summary activity.

C. Exposure to related vocabulary supports fluency and comprehension.

D. The ability to create a mental picture improves the likelihood that students will be engaged readers.

A

A. Schema development supports comprehension.

The teacher is supporting schema development prior to reading the text. Building background knowledge and activating prior knowledge will improve comprehension.

330
Q

In a co-taught, seventh-grade math class, the special education teacher notices that two of her students with mild intellectual disabilities are struggling to multiply fractions. She sees them counting on their fingers as they try to do math in their heads and writing additional problems in which they are adding the same number multiple times. Which of the following would be an appropriate aid to provide to these students to help them complete the assignment?

A. a calculator

B. a hundreds chart

C. a multiplication chart

D. fraction manipulatives

A

C. a multiplication chart

Based on the behavior seen by the teacher, the problem appears to be that these students do not have their multiplication facts memorized. They are working to count up or add for each step of the multiplication. Providing a multiplication chart will help them work faster and reinforce their memory of these basic facts, without removing them from what they are actually doing as a calculator might.

331
Q

A seventh-grade science class is beginning a unit on experimentation. One student in the class has cerebral palsy and cannot effectively use some of the equipment in the experiments. What can the teacher do to incorporate this student into the activities?

A. Allow the student to complete the experiment to the best of his abilities.

B. Have the experiment be completed in groups with specific roles, most of which the student can effectively complete.

C. Provide an alternate assignment to the student.

D. Allow the student to observe the experiments only due to safety concerns.

A

B. Have the experiment be completed in groups with specific roles, most of which the student can effectively complete.

By assigning specific roles, this allows the student to choose a role he can successfully complete alongside his peers. This method provides the student appropriate access to the general education curriculum.

332
Q

Which of the following is not a required member of an Admission Review Dismissal (ARD) meeting?

A. a school district administrator

B. the special education teacher

C. a general education representative

D. the parent’s legal counsel

A

D. the parent’s legal counsel

Parents are able to bring legal counsel to an ARD meeting with prior notice to the school district, but presence of legal counsel is not required.

333
Q

Mrs. Mathers is reading aloud with her class. As she reads she periodically stops and asks students questions about the text to clarify important plot developments and to explain the main ideas in the text. Which of the following is Mrs. Mathers most likely trying to accomplish?

A. encouraging students to imagine themselves in the story

B. monitoring students’ comprehension of the text

C. scaffolding reading instruction with previously read texts

D. engaging students’ interest in reading

A

B. monitoring students’ comprehension of the text

Mrs. Mathers’ questions are gauging the students’ comprehension of the text. Her questions directly relate to and assess how well students comprehend the concepts of the text.

334
Q

An elementary teacher should teach students to determine the functions and purposes of media. These include which of the following?

A. literacy and fluency

B. advertising and capitalism

C. education, information, and persuasion

D. reality and entertainment

A

C. education, information, and persuasion

This is the correct answer. Other purposes of using media appropriately are for information and entertainment.

335
Q

Which of the following standards would a third-grader most need to master before learning ordinality of large numbers?

A. value of a digit

B. rounding large numbers

C. addition of three-digit numbers

D. cardinality

A

A. value of a digit

Ordinality relates to ordering objects (eg. determining which object or number is bigger). In order to understand which six-digit numbers are larger than others, a student must first understand how place value affects the value of a digit. Without this foundational knowledge, a child might believe that 77,000 is larger than 111,000 because 7 is a bigger number than 1.

336
Q

Which strategy would best assist a transitioning student with a learning disability in the workplace?

A. providing the student’s guardian with a pamphlet about potential employment opportunities

B. creating vocational goals within the IEP to target specific job-related skills

C. asking the student to research potential job opportunities

D. conducting mock interviews with the student after school

A

B. creating vocational goals within the IEP to target specific job-related skills

By providing goals that the student can work on in the IEP, the teacher is instructing the student in specific skills that can generalize into a workplace environment.

337
Q

Before reading a literary piece, an elementary teacher will ask the class, “What do you know about…?” Which of the following best describes the purpose of this question?

A. initiate interest

B. set the topic for the students

C. connect to the student’s schema

D. allow the students to be involved in a discussion

A

C. connect to the student’s schema

This is the correct answer because students need to have background knowledge about what they are going to read. Students learn best by placing new knowledge in the context of already known knowledge.

338
Q

Which of the following is the main purpose of instructing students with significant cognitive disabilities using a functional curriculum?

A. help students to participate in adaptive physical education

B. help students to complete state assessment requirements for graduation

C. help students to develop daily living skills needed for adulthood

D. help students to prepare for college or technical schools

A

C. help students to develop daily living skills needed for adulthood

Functional curriculums focus on daily living skills which may include self care, independent living, behavior management, and functional academics in preparation for exit from high school.

339
Q

Some gifted students are labeled as having dual exceptionalities. What is the primary reason dually exceptional children are not identified through special education?

A. If they are able to be successful, they do not need services through special education.

B. Being in the gifted program automatically disqualifies students for special education programming, so they are not allowed to have an evaluation by the ARD committee.

C. Only children with severe disabilities are identified early in their academic careers.

D. In the early elementary years, they may be able to compensate for their deficits due to being gifted, so it makes them appear as average students with talents in a few academic areas.

A

D. In the early elementary years, they may be able to compensate for their deficits due to being gifted, so it makes them appear as average students with talents in a few academic areas.

Dually exceptional children are often not identified as needing special education because their giftedness masks their disabilities. For example, a gifted child with dyslexia can read enough words in a passage and listen to the class discussion to have a strong understanding of what is being taught.

340
Q

Hazel is a fifth-grade student with spina bifida who uses a wheelchair and does not have any intellectual impairments. She has recently moved to a new school district and is struggling to build relationships with the students in her grade at school. What can the special education teacher suggest?

A. Hazel should play online sports video games after school to make new friends.

B. Hazel should sit at different seats each day during lunch to make friends.

C. Hazel should try new hobbies that are conducive to wheelchair users.

D. Hazel should join the Girl Scout troop that has other girls in 5th grade.

A

D. Hazel should join the Girl Scout troop that has other girls in 5th grade.

This allows Hazel the opportunity to make friends in the school and creates situations in which friendships can form.

341
Q

A parent is upset about a program that is being implemented as part of their child’s special education services. What can the parent do to adjust these services?

A. remove the student from the school and enroll them elsewhere

B. ask the special education teacher to remove the student from the program

C. request an ARD meeting to discuss the student’s services

D. plan to discuss the services at the next annual ARD meeting

A

C. request an ARD meeting to discuss the student’s services

Parents and guardians can request an ARD meeting at any time to discuss their student’s special education services.

342
Q

Free Appropriate Public Education (FAPE) is outlined within the Individuals with Disabilities Education Act (IDEA) to ensure that:

A. school districts provided appropriate services to students with disabilities.

B. private schools could not educate students with disabilities.

C. individual with disabilities could afford tuition to public universities.

D. lunch would be free if students with disabilities could not pay.

A

A. school districts provided appropriate services to students with disabilities.

FAPE outlines that public schools must provide appropriate education services, including potential outside service providers, at no cost to parents or guardians.

343
Q

A third-grade student with autism and a severe speech impediment is provided an alternative communication device in their IEP meeting. One effective way that the teacher can promote the use of this AAC device is to:

A. reserve the device for use only during speech language therapy.

B. let the student use the device on for games that they enjoy playing.

C. only use the device when the guardian specifically requests it.

D. ensure the device is readily available for use and within reach.

A

D. ensure the device is readily available for use and within reach.

Allowing the device to be readily available and within reach will increase the number of opportunities the child has to communicate using the device. This can allow the teacher to actively prompt the student to use the device during instruction.

344
Q

While conducting research for an informational essay on an ancient civilization, a seventh-grade student is using an encyclopedia as a reference. The student is flipping through the pages trying to find the right entry. In order to best help the student locate the information quickly, the teacher should give the student instruction on:

A. choosing appropriate research material.

B. using subheadings as a search tool.

C. how to locate and use the glossary.

D. how to locate and use the index.

A

D. how to locate and use the index.

The student would benefit from a quick reminder about how indexes work and how to use the index to quickly locate the correct information.

345
Q

During the writing process for an informative text, a fifth-grade student with a specific learning disability needs to be able to organize information and build on ideas by choosing an appropriate organizational strategy. Which of the following is the best way for the inclusion teacher to support the student in this process?

A. Allow the student to choose their own writing topic.

B. Teach a mini-lesson on transitions in a small group format.

C. Plan for a writing conference with the student between the rough and final draft of the essay.

D. Provide the student with a graphic organizer applicable to the topic of his essay.

A

D. Provide the student with a graphic organizer applicable to the topic of his essay.

Graphic organizers are important tools in the pre-writing process, and in this case it can be used to help the student organize his ideas in a way that will support the informative purpose.

346
Q

A fifth-grade special education teacher administers a pre-assessment to her small group before beginning a new math unit. What knowledge can the special education teacher gain by administering this assessment?

A. The teacher can determine if they are able to appropriately take an exam.

B. The students can learn how the teacher prepares quizzes and how they should select answers.

C. The teacher can determine how much time must be spent on each of the concepts by how the students score.

D. Nothing can be determined until a post-assessment is conducted.

A

C. The teacher can determine how much time must be spent on each of the concepts by how the students score.

By administering a pre-assessment, the teacher can tailor the unit to the individuals in the classroom. Results from this assessment can shape instruction and activities and allow the teacher to meet the student’s individual needs.

347
Q

The case Endrew F. v. Douglas County School District (2017) influenced which of the following aspects of special education?

A. development of goals

B. determination of ESY

C. documentation of accommodations

D. provision of FAPE

A

D. provision of FAPE

The Endrew F. decision raised the standard for meaningful educational progress. The judge ruled that “every child should have the chance to meet challenging objectives.”

348
Q

Which of the following strategies involves collecting data at a series of intervals to determine students’ current performance levels?

A. visualization

B. reciprocal teaching

C. elaborated feedback

D. progress monitoring

A

D. progress monitoring

Progress monitoring involves collecting data to determine growth or regression. This may be used for academic and non-academic areas of need.

349
Q

A fifth-grade student with dyslexia continues to struggle with reading even though she has received special education services for over a year. The teacher would like to use assistive technology (AT) to help her. What would be the first low-tech type of assistive technology that may be considered for addition into her Individualized Education Plan (IEP) and why?

A. word prediction software, so that she can type written responses more quickly

B. a text to speech app that can be used on any assignment, because it makes all assignments accessible

C. audio files with headphones, because they are not disruptive and enhance comprehension

D. colored overlays, because they are low cost and do not disrupt the classroom setting

A

D. colored overlays, because they are low cost and do not disrupt the classroom setting

Colored overlays are helpful for students with dyslexia as it reduces eye strain and allows for easier reading. This is low cost and can be used on any reading passage.

350
Q

A 5th-grade student with a learning disability generally comprehends engaging, narrative texts, but is struggling with content-specific informative texts. The teacher regularly implements pre-reading activities for the class as a whole. Which of the following strategies should the inclusion teacher implement to help this student when reading informative texts?

A. take turns reading aloud with a neighbor

B. use a dictionary to define every unfamiliar word

C. decrease reading rate

D. paraphrase the text sentence by sentence

A

C. decrease reading rate

When reading to learn and reading something outside of your comfort zone, it is often helpful to read more slowly than usual.

351
Q

The teaching of which of the following geometry skills to students with disabilities would be most aided by the use of technology such as a touch screen?

A. Finding lengths of segments on a number line

B. identifying congruent polygons and classifying congruence transformations

C. finding the distance between two points using the distance formula

D. using the Pythagorean theorem

A

B. identifying congruent polygons and classifying congruence transformations

This is the only one of the answer options that requires manipulating objects. Being able to lay shapes on top of each other or rotate them could be very helpful to teach the concept of congruence and transformations such as rotations or reflections.

352
Q

Research has analyzed many predictors of students’ success in learning to read in early grades. This research has found that which of the following abilities are the best predictors of good readers?

A. knowledge of common phonics patterns

B. ability to infer information from a written text

C. ability to produce words following a pattern of alliteration

D. knowledge of letters and phonemic awareness skills.

A

D. knowledge of letters and phonemic awareness skills

Research has found that the knowledge of letters and acquisition of phonemic awareness skills are the best predictors of students’ success in learning to read.

353
Q

A special education teacher has finished printing out an IEP for an upcoming ARD meeting. Another teacher, who is not the student’s general education teacher, asks if she can see the IEP paperwork as she has the student’s sibling in her classroom. The special education teacher should:

A. provide the teacher with a synopsis of the student’s IEP.

B. not allow the other teacher to view the IEP paperwork.

C. invite the teacher to the student’s upcoming ARD meeting to review the IEP.

D. allow the teacher to view the paperwork but not copy it.

A

B. not allow the other teacher to view the IEP paperwork.

The teacher should not allow an individual that is not implementing the IEP to view it unless consent is given from the parent or guardian.

354
Q

Having two teachers plan, instruct, and assess a class of students with and without disabilities is an example of

A. team teaching

B. consultative support

C. cooperative learning

D. peer tutoring

A

A. team teaching

Team teaching may also be referred to as co-teaching. This occurs when two teachers plan, instruct, and assess a class of students.

355
Q

Roman has recently started on medication for ADHD. His mother sent an email to the teacher to let her know that his behavior and attentiveness should improve. What negative side effects may the parent ask the teacher to monitor?

A. lethargy or sleepiness in class

B. shakiness in the hands

C. lack of hunger or excessive thirst

D. twitching facial muscles

A

C. lack of hunger or excessive thirst

This is a common side effect of ADHD (Attention Deficit Hyperactivity Disorder) medication. The parent should monitor this and share concerns with the doctor as this may affect the child’s growth and development.

356
Q

A teacher is reading aloud to her class. At the end of a paragraph, she stops to ask the students a question like, “Where do you think the tiger is headed now?” In asking a question like this, the teacher is trying to:

A. encourage students to make connections between texts.

B. encourage students to connect the story to their own lives and experiences.

C. encourage students to make predictions based on textual evidence.

D. encourage students to share their point of view.

A

C. encourage students to make predictions based on textual evidence.

By asking this question, the teacher is hoping students will use evidence from the reading to make a prediction of the tiger’s destination.

357
Q

Miss Jenkins is a paraprofessional at Spring Oaks Elementary, and part of her daily schedule includes helping with cafeteria duty. She approaches the special education teacher to discuss the lunchroom behavior of Juma, a special education student who is new. Miss Jenkins is bothered that Juma never uses a fork and only eats with her hands, even when having messy foods such as spaghetti and sauce.

When considering this behavior, the adults should be aware of:

A. how the other students respond and correct her if it is bothersome to them.

B. her family’s socio-economic status as they may not be able to afford utensils.

C. her fine motor skills and assess if using utensils is too difficult.

D. the culture that Juma is from and if they eat with their hands instead of utensils.

A

D. the culture that Juma is from and if they eat with their hands instead of utensils.

Some cultures use their hands for eating, so Juma may not have experience using utensils.

358
Q

Marybelle, John, and Daphne are students in a resource English I class. Their teacher taught them how to record annotations in the last book they read. This week, their homework has been to read and annotate Chapter 1 in their new book. Today, the students had an open-book quiz and the average score was 68%. What should the teacher do?

A. Simplify the quiz for Chapter 2.

B. Review the students’ annotations and reteach the missed concepts.

C. Send home instructions on annotating so parents can help them.

D. Annotate the book in class to prevent them from missing any information.

A

B. Review the students’ annotations and reteach the missed concepts.

This method allows the teacher to determine whether students were not effectively annotating the book or if errors were related to specific concepts on the quiz.

359
Q

A teacher has decided that he wants the students to become more involved in the assessment process at the end of a teaching unit. Which way would be the best incorporation of students into the assessment process?

A. The students should grade their tests and report the grades to the teacher.

B. The teacher should send the test home to have the families support the students while they take the test as homework.

C. The students should create an assessment and establish a rubric.

D. The teacher should post the grades of the students to allow them to organize the scores to find the mean, median, and mode.

A

C. The students should create an assessment and establish a rubric.

Student-created assessments and student-created rubrics are both ways to incorporate students into the assessment process.

360
Q

Mrs. Janes begins each day with a warm-up question that reviews material taught the day before. Students write their responses individually and then they discuss as a class. While they are writing, she moves around the room reading each response. What type of assessment is this?

A. review

B. summative

C. formative

D. formal

A

C. formative

This formative assessment allows the teacher to determine whether students are progressing through the current content. If students are struggling with the answer, she can immediately reteach it before moving on to the next concept.

361
Q

A kindergarten student with autism and a speech impairment is engaging in minor self-injurious behavior when presented with work demands. Before beginning an intensive intervention, what steps should the teacher take to ensure compliance and safety?

A. Provide a communication method for the student when he engages in this behavior.

B. Hold an ARD meeting to ask the parents for consent to conduct an FBA.

C. Begin taking data on behavior and developing a hypothesis.

D. Contact the administrator every time the student engages in this behavior.

A

B. Hold an ARD meeting to ask the parents for consent to conduct an FBA.

Prior to conducting any form of evaluation, such as a functional behavior assessment (FBA), an ARD meeting must be conducted to obtain parental consent.

362
Q

Which of the following prohibits discrimination of individuals based on their disabilities?

A. Every Student Succeeds Act (ESSA)

B. Individuals with Disabilities Education Act (IDEA)

C. Free Appropriate Public Education (FAPE)

D. Section 504 of the Rehabilitation Act

A

D. Section 504 of the Rehabilitation Act

Section 504 is a anti-discrimination statute which prohibits discrimination due to disabilities.

363
Q

Marcus is a first-grade student working on place value task cards at a math center. Each task card shows an image of base ten manipulatives, and students are to write the corresponding number in standard form on their recording sheet. Marcus selects a card that shows 1 hundreds block and 3 ones cubes. On his recording sheet, he writes that the number shown on the card is 13.

Based on this scenario, which of the following tools would be the most effective to provide to Marcus?

A. a set of unifix cubes

B. a hundreds chart

C. a set of base ten blocks

D. a dry-erase place value chart

A

D. a dry-erase place value chart

Marcus is struggling to recognize that each place needs to be filled by a digit, even if it is zero. Therefore, a place value chart would be an effective tool for Marcus to use. With the task card described in the question, a place value chart would help Marcus recognize that there are 3 ones, 0 tens, and 1 hundred, making the number 103.

While concrete manipulatives are crucial when teaching place value, they would likely not help Marcus in this specific scenario. The task cards already show an image of base ten blocks, so Marcus would likely use the manipulatives to recreate the image and continue to write the number as 13.

364
Q

Ms. Stevens’ third-grade class is completing an assignment in which they circle the larger of two fractions. While observing students as they work, she notices that one student, Ava, is consistently circling the smaller fraction on each pair. When Ms. Stevens asks Ava to explain her thought process on one of the problems, Ava states that “⅛ is greater than ¼, because 8 is bigger than 4.”

Based on this comment, which of the following would be the best way for Ms. Stevens to support Ava?

A. asking Ava if she would rather have ¼ of a pizza or ⅛ of a pizza

B. asking the campus math interventionist to pull Ava out of class for additional one-on-one support

C. encouraging Ava to draw a picture representation of each fraction before deciding which one is larger

D. providing Ava with fraction bar manipulatives to model two fractions before identifying the larger fraction

A

D. providing Ava with fraction bar manipulatives to model two fractions before identifying the larger fraction

Providing Ava with fraction manipulatives will allow her to have a concrete visual representation of the size of each fraction, improving her ability to compare fractions.

365
Q

A special education teacher is planning phonological awareness instructional activities to support a group of students struggling with these skills. To properly scaffold instruction in phonological awareness, a teacher should present skills in which order?

A. Rhyming, Onset Identification, Phoneme Blending, Phoneme Manipulation

B. Onset Identification, Phoneme Blending, Rhyming, Phoneme Manipulation

C. Onset Identification, Phoneme Blending, Rhyming, Phoneme Manipulation

D. Phoneme Blending, Phoneme Manipulation, Onset Identification, Rhyming

A

A. Rhyming, Onset Identification, Phoneme Blending, Phoneme Manipulation

This option has the skills correctly listed from easiest to most difficult and would be the best order to follow to properly scaffold student instruction.

366
Q

A ninth-grade student with a specific learning disability in reading is frequently unable to complete his assigned work in the general education classroom. His general education teacher states that the student has not turned in multiple assignments. He completes all of his reading assignments efficiently in his special education reading small group. At the upcoming IEP meeting, the special education teacher should:

A. ask the student to try harder to complete his assignments in reading.

B. increase the amount of pull-out reading support the student is receiving.

C. suggest adding in-class support for the student during reading.

D. discuss implementing a Behavior Intervention Plan (BIP).

A

C. suggest adding in-class support for the student during reading.

If the student is able to complete the necessary skills with support outside of the general education classroom, providing in-class support would promote the generalization of these skills. This provides the appropriate support in the least restrictive environment (LRE).

367
Q

In a middle-school language arts classroom, a successful reading program should:

A. focus on teacher-selected texts.

B. use classic literature selected by students or teachers.

C. allow for student-selected texts.

D. use leveled readers exclusively.

A

C. allow for student-selected texts.

By middle school, students are developing more autonomy and allowing them to choose books should encourage engagement in reading.

368
Q

All of the following are ways to determine a student’s reading level except:

A. asking students to separate multisyllabic words into individual syllables or word parts.

B. asking comprehension questions like, “What are the four stages of the water cycle?”

C. using district-provided reading level assessment software.

D. conducting a miscue analysis and oral reading assessment.

A

A. asking students to separate multisyllabic words into individual syllables or word parts.

While the ability to read multisyllabic words fluently contributes to a student’s reading level, this activity would assess a student’s understanding of syllabication exclusively, not the current reading level.

369
Q

During science labs, Mr. Able notices that some students do most of the work, and others are excluded. In an effort to have all students actively participate, he has them collect data on their own lab sheets, but he notices that this causes some of his struggling students to spend the entire lab recording data. This prevents these students from preparing materials or completing lab procedures. What is another approach he could try?

A. Provide two class periods to complete labs so that students can move at a slower pace. Students who finish early can use the free time as a study period.

B. Provide each group with one paper to collect data. Assign designated roles to each group member.

C. Provide all students with a copy of the paper to collect data, but only require students to gather written data if they are in the gifted program.

D. Encourage students to be more inclusive and help each other by giving bonus points for writing for students who are behind.

A

B. Provide each group with one paper to collect data. Assign designated roles to each group member.

This helps struggling students succeed, as they have a set of expectations to meet without the stress of needing to copy data from the lab. By assigning designating roles, Mr. Able can build upon students’ individual strengths while encouraging collaboration.

370
Q

Which of the following strategies would help a group of students develop an understanding of text directionality?

A. The teacher provides a series of images representing events in a familiar story and asks the students to put them in the correct order.

B. The teacher takes pictures of each page of a book then creates a slideshow to project the images large enough for all students to see.

C. The teacher reads to the class daily from oversized books, discussing the front and back covers before beginning, and pointing to the words as she reads them.

D. The teacher reads the class a story, then asks them to draw a new cover for the book that represents the story.

A

C. The teacher reads to the class daily from oversized books, discussing the front and back covers before beginning, and pointing to the words as she reads them.

This is an appropriate strategy to help students familiarize themselves with the different parts of a book and begin to see that text moves from left to right and top to bottom in a book.

371
Q

Spina bifida is a developmental birth defect characterized by:

A. an overactive spinal column.

B. an abnormal prenatal ultrasound.

C. a developmental delay present at birth.

D. an incomplete closure of the neural tube.

A

D. an incomplete closure of the neural tube.

Spina bifida is a developmental birth defect characterized by an incomplete closure of the neural tube.

372
Q

A third-grade student with attention-deficit hyperactivity disorder (ADHD) is engaging in severe property destruction. The general education teacher mentions she had a student in a previous year who engaged in this behavior. She suggests implementing the behavior intervention plan for her previous student with the current student. The special education teacher should:

A. go along with the behavior plan as long as it is effective.

B. advise against this and invite the teacher to the ARD meeting to request an FBA.

C. pull the student from the general education classroom until their behavior is under control.

D. provide the teacher with a copy of the procedural safeguards.

A

B. advise against this and invite the teacher to the ARD meeting to request an FBA.

At times, it can be difficult to contradict a colleague on a given student. However, it is the special education teacher’s job to ensure that all laws and procedural safeguards are being followed.

373
Q

A kindergarten student has been identified by a general education teacher as having repetitive behaviors and language delays. After evaluation, the Licensed Specialist in School Psychology (LSSP) determines the student is on the autism spectrum. What steps are required for the student to access special education services?

A. An ARD meeting must be held to review the evaluation and offer services

B. the parent must verbally consent to special education services

C. a special education teacher must observe the student to determine if services are needed

D. the principal must find placement for the student in special education

A

A. An ARD meeting must be held to review the evaluation and offer services

Once evaluation is complete, an ARD meeting must be held to review evaluation data and determine if the parent agrees to the student entering special education services.

374
Q

A second-grade student with a gastronomy tube requires assistance during lunch. Which staff member should be responsible for the g-tube feeding?

A. a health professional or trained special education staff member

B. only the health professional

C. a special education staff member

D. the parent or guardian

A

A. a health professional or trained special education staff member

A nurse can administer a g-tube feeding, but can also train special education staff members in how to administer g-tube feedings.

375
Q

All of the following are effective supports for helping students with disabilities to understand and solve multi-step mathematics problems EXCEPT:

A. providing a graphic organizer with pre-filled stems to trigger connection to steps in the process

B. connecting the problems to real-world or concrete examples

C. listing the required steps in their simplest terms

D. providing a calculator

A

D. providing a calculator

Calculation aids are an effective support for students who struggle with math calculation but are not the most effective aid for students who struggle with the problem-solving process because they do not help understand what to calculate or in what order calculations should be completed.

376
Q

A student with a specific learning disability (SLD) in the area of written expression is reluctant to share answers during class discussions due to fear of being incorrect. Which of the following instructional strategies to be used while reading a short story in language arts class will help the student to gather his thoughts and engage in class discussion?

A. asking the student to practice his answer with a peer before sharing

B. offering an audiobook instead of reading the story independently

C. providing a graphic organizer with prompts to complete while reading

D. providing a laptop to type responses

A

C. providing a graphic organizer with prompts to complete while reading

The graphic organizer will allow the student to gather his thoughts and ideas while reading which will encourage active participation.

377
Q

A second-grade teacher is planning a field trip to a museum for the entire grade. Some students in this grade have varying disabilities, including two students in wheel-chairs. Before scheduling the field trip, the teacher should:

A. arrange lunches to be provided for all the students prior to the field trip.

B. let the special education case manager know that the grade is taking a field trip.

C. request special transportation and ensure the museum is wheel-chair accessible.

D. ask the parents if the students can stay at school during the field trip.

A

C. request special transportation and ensure the museum is wheel-chair accessible.

Planning for community outings is vital to maintaining a safe and respectful environment for students with disabilities. By arranging for special transportation and accessibility, this ensures that students of all ability levels can appropriately access the field trip.

378
Q

A special education teacher is teaching a high-school vocational class for individuals with intellectual disabilities. The current unit is on scheduling vocational activities. The special education teacher wants to create a relevant activity in a community setting. An appropriate lesson for this unit would be:

A. having the students research and schedule public transportation to various places in the city.

B. providing a list of various tasks to be completed around the school.

C. providing the students with a map of the area in which they live.

D. creating a cooking activity that requires multiple steps.

A

A. having the students research and schedule public transportation to various places in the city.

Scheduling transportation is a vital vocational skill for transition-age students with disabilities. This lesson provides a community-based means to practice these skills.

379
Q

According to the guidelines, the IEP committee must address all of the following when planning for students with autism except:

A. typical grade-level peers

B. in-home training

C. behavioral and social strategies

D. community-based training

A

A. typical grade-level peers

While curriculum influences a student’s goals, an IEP should be focused on that student as an individual rather than comparing them to grade level peers.

380
Q

A special education teacher is working to improve the reading fluency of struggling readers in her 4th-grade class. Which of the following strategies is the teacher most likely to use for this purpose?

A. independent reading with grade-level passages

B. require annotations while reading

C. use a graphic organizer that matches the organization of the text

D. choral reading activities

A

D. choral reading activities

Choral reading pairs modeling of fluent reading with oral reading to support fluency development.

381
Q

Which of the following would be the most beneficial activity for kindergarten students who are practicing subitizing skills?

A. a math station where students recreate a given pattern using different colored blocks

B. a small group lesson in which students count a set of objects, touching each object once as they count

C. a partner activity in which both students turn over cards and determines which number is larger

D. a math station where students repeatedly roll a die, recording the number that they roll each time

A

D. a math station where students repeatedly roll a die, recording the number that they roll each time

Subitizing is the ability to quickly recognize the amount of objects in a set without counting each object. By rolling a die and recording the value, students are reinforcing this skill as they gradually learn to recognize the value they rolled without having to count each dot.

382
Q

Victoria is a second-grade student who has been evaluated due to concerns that she has significantly lower expressive communication skills than a typical child of her age. In order to qualify for special education or related services, what else must be established?

A. She also demonstrates a deficit in receptive language skills.

B. She also demonstrates an inability to effectively communicate, which has a significant educational impact.

C. She also demonstrates articulation errors which are not developmentally appropriate for her age and cause her expressive communication to be delayed.

D. She also has hearing loss which impacts her communication abilities

A

B. She also demonstrates an inability to effectively communicate, which has a significant educational impact.

In order for a child to qualify for services, his/her disability must have a significant educational impact.

383
Q

An IEP meeting for a 13-year-old with Attention Deficit Hyperactivity Disorder (ADHD) and emotional disturbance is next week. A general education teacher suggests a goal for the student to remain on-task for an entire 120-minute reading block. What steps should the special education teacher take following this suggestion?

A. Thank the teacher for the suggestion but not include the goal in the IEP.

B. Complete a survey of general education teacher’s reading block assignments.

C. Determine how long students without disabilities are on task during their reading block.

D. Immediately write the goal into the IEP.

A

C. Determine how long students without disabilities are on task during their reading block.

On-task behavior for individuals with disabilities is important to ensure they are receiving adequate access to the curriculum. However, intervention should be directed toward the goal of generalization. By gathering information on peer behavior, a realistic and appropriate target can be set.

384
Q

An eighth-grade student has just moved to the United States from Syria with his family. They are considered refugees. He uses a wheelchair, and his mother has requested special education services because she believes he has an orthopedic impairment. The school has conducted a full and individual evaluation (FIE) in his native language, which is also the primary language spoken at home.

Who are the members of the Admission, Review, and Dismissal (ARD) committee that must be there to determine if he needs services?

A. parent, student, special education teacher, general education teacher, an administrator, a refugee mentor, and an interpreter

B. parent, student, special education teacher, general education teacher, an administrator, an evaluator(s), and an interpreter

C. parent, special education teacher, general education teacher, an administrator, an evaluator(s), and an interpreter

D. parent, special education teacher, general education teacher, and an administrator

A

C. parent, special education teacher, general education teacher, an administrator, an evaluator(s), and an interpreter

IDEA requires that an IEP team include the special education teacher, general education teacher, an administrator (LEA designee), and someone who is qualified to conduct/interpret evaluation results when considering eligibility. Each of these individuals must be present, and the school must offer to provide an interpreter for the parent. The child may attend if he wishes to and the parent agrees; however, he is not required to attend.

385
Q

A student with a moderate physical disability is beginning transition services. What strategies can the teacher use to maximize his ability to gain effective employment?

A. contact the parent to assist with possible employment opportunities

B. allow the student to interview for potential positions and provide feedback

C. give the student a list of potential employment opportunities

D. provide appropriate adaptations to generalize his current skills to employment

A

D. provide appropriate adaptations to generalize his current skills to employment

By providing appropriate adaptations, the teacher is linking current skills that can be transmissible to the workplace. Workplace adaptations can be provided by employers if they are required for an individual with a disability to access their environment.

386
Q

A first-grade student has recently been diagnosed with autism and his parents are attending their first ARD meeting. The student’s mother admits to the special education teacher that she is nervous and does not understand some of the process. How can the special education teacher assist the parent in the ARD meeting?

A. allow the parent to guide the meeting and debrief with them after

B. provide the parent with a copy of the procedural safeguards

C. tell the parent there is nothing to worry about

D. explain common acronyms that occur during the meeting such as IEP, ASD, FIE, etc.

A

D. explain common acronyms that occur during the meeting such as IEP, ASD, FIE, etc.

By explaining jargon that is commonly used in ARD meetings, the parent can better follow the process of the meeting. While the many acronyms used in special education can be helpful, they can be intimidating to parents who are new to the process.

387
Q

Kinsley is a ninth-grade student who has a hearing impairment. She has a cochlear impairment and is also provided with American Sign Language (ASL) interpretation services. Kinsley would like to sign up for the track team, but the team uses a horn as a sound to signal the beginning of each race. Which of the following is most appropriate for Kinsley’s participation on the track team?

A. Provide both the sound of the horn and a visual cue to signal the beginning of the race.

B. Allow Kinsley to practice with the team, but ask her to only attend track meets as a spectator.

C. Ask one of Kinsley’s teammates to tap her on the shoulder whenever it is time to begin the race.

D. Encourage Kinsley to sign up for another extracurricular activity where her hearing impairment will not have an impact.

A

A. Provide both the sound of the horn and a visual cue to signal the beginning of the race.

Providing a visual cue allows Kinsley to participate with her peers without distracting the other runners.

388
Q

What is the primary reason for giving students standardized assessments each year?

A. It provides data for federal funding for schools.

B. It compares them to age- and grade-level-equivalent peers to evaluate progress.

C. It shows whether or not a student continues to need special education services.

D. It shows mastery of IEP goals.

A

B. It compares them to age- and grade-level-equivalent peers to evaluate progress.

Standardized assessments show where a student of the same age and grade level performs in relation to same age- and grade-level peers. It may be used as a data point to show whether a student who is receiving special education services is making consistent progress or if they are accelerating or declining.

389
Q

The parent of a seventh-grade student with a specific learning disability in reading and Attention Deficit Hyperactivity Disorder (ADHD) informs the special education teacher that he has begun a new medication for ADHD. The student begins falling asleep regularly in class and often sleeps through lunch. What steps should the special education teacher take to address the student’s sleeping?

A. collect data on how often they are sleeping and notify the parent about the change

B. allow the student a rest time during his time in the special education classroom

C. notify the student’s pediatrician about the change in behavior

D. call an IEP meeting to address the medication change

A

A. collect data on how often they are sleeping and notify the parent about the change

Medication can often cause a drastic change in behavior in students with disabilities. Informing the guardian with proper evidence of the frequency of sleeping will allow them to address the change in behavior with their pediatrician.

390
Q

Brady is a 16-year-old student with an emotional disability. He participates in a therapeutic program at his school that includes individual counseling and social skills. He plans to pursue a career in computer science after graduation, and he has just accepted a part-time job at a local computer repair store. Brady is able to walk to the store from his house, so he will work on evenings and weekends. What is the best way that his special education teacher can incorporate transition activities to help him to prepare for his job?

A. Provide Brady with a list of local colleges that offer computer science programs so that he can begin registering for college tours.

B. Role play challenging work scenarios such as dealing with a difficult customer, asking a coworker for help, and participating in a performance review with a supervisor so Brady can identify social skills and coping strategies to use at work.

C. Register Brady for driver’s education classes on weekends so that he can get his license and drive to work rather than walk.

D. Notify Brady’s supervisor that he has a disability and request that the supervisor tells the school of any incidents at work.

A

B. Role play challenging work scenarios such as dealing with a difficult customer, asking a coworker for help, and participating in a performance review with a supervisor so Brady can identify social skills and coping strategies to use at work.

Due to Brady’s emotional disability, he may struggle to manage or express his emotions. Role playing scenarios that Brady may encounter at work will help him to prepare for what to expect and identify appropriate ways to respond. Practicing the use of coping strategies at school, reinforces the importance of these strategies and will encourage Brady to transfer these skills outside of school.

391
Q

Which of the following best describes the primary role of mentor texts in writing instruction?

A. Demonstrate the process of turning an idea into a complete piece of writing.

B. Provide a comparison by which students can evaluate the quality of their own writing.

C. Outline the teacher’s expectations and requirements for a student’s own writing assignment.

D. Provide a clear example of a genre that students can use to guide stylistic and/or structural choices.

A

D. Provide a clear example of a genre that students can use to guide stylistic and/or structural choices.

Mentor texts are finished examples of the genre of writing students are being asked to create. These texts are often published pieces of writing that are not meant to be a specific goal for the students to meet, but a stylistic and/or structural guide.

392
Q

Which of these is not a specific learning disability?

A. intellectual disability

B. dysgraphia

C. dyslexia

D. disability in reading comprehension

A

A. intellectual disability

Intellectual disabilities are in their own category of eligibility. The impact is more global and typically affects a student’s academic, cognitive, and adaptive skills.

393
Q

Which of the following would be an appropriate use of group work in a middle-elementary special education math class?

A. reinforcing math facts by playing “around the world,” a speed game in which students compete against each other to quickly answer problems held up by the teacher on large flashcards

B. reviewing the steps for long division by having each student in the order of seating come up to the board and complete the next step of a problem to be solved collectively.

C. reinforcing learning by having students solve problems independently and then explain their reasoning to their partner to check their work

D. providing students with an answer sheet to check their work as they complete a worksheet together

A

C

394
Q

An elementary teacher is planning an instructional unit on some of the most common communication conventions (i.e., punctuation, spelling, and grammar) used in digital media. Which of the following activities would best help support the teacher’s goal?

A. Have students choose one specific communication convention out of the major categories and research its characteristics.

B. Identify how articles are tagged on a blogging system.

C. Create a class website about the rules of standard grammar.

D. Separate the characteristics of the most common communication conventions using a Venn diagram.

A

D. Separate the characteristics of the most common communication conventions using a Venn diagram.

This is the best answer option, as the students are able to see how different communication conventions overlap in function and also how they are different in helping deliver the information.

395
Q

A special education teacher is notified of an incoming student with a physical disability. The student is in a wheelchair and requires transferring throughout the day. What first steps should the special education teacher take to ensure that the transfers are safe for the student and staff?

A. Schedule and plan to have another staff member assist with the transfers.

B. Attempt the first transfer and determine if additional assistance will be required to complete further transfers.

C. Consult with the school physical therapist or guardian to determine a plan for how to transfer the student.

D. Prepare the classroom to accommodate a wheelchair.

A

C. Consult with the school physical therapist or guardian to determine a plan for how to transfer the student.

Due to the variety of physical disabilities, consulting a school specialist or guardian to determine a transfer plan prior to beginning is necessary. This allows for transfers of the student to be safe and respectful for all parties involved.

396
Q

A special education teacher is approached by a general education teacher regarding a student on her caseload. She is requesting support on how to appropriately implement the student’s behavior intervention plan (BIP) in her classroom. The special education teacher should:

A. provide the teacher with a copy of the BIP.

B. inform the general education teacher that the BIP is used only in special education.

C. speak with the student about having better behavior in their general education class.

D. provide an overview and tips to help support the student’s behavior in general education.

A

D. provide an overview and tips to help support the student’s behavior in general education.

By providing an overview of the BIP, the special education teacher is effectively supporting the student in his least restrictive environment and allowing him to appropriately access the general education curriculum.

397
Q

A kindergarten teacher is working to help her students learn the names and shapes of the letters in the alphabet. Which of the following activities would best support this goal?

A. The teacher begins every day by singing the alphabet with her students.

B. The teacher names a letter and writes it on the board, and the children use playdough to form the shape of the letter at their table.

C. The teacher uses bright, colorful visuals of the alphabet, including capital and lowercase letters to decorate her classroom.

D. The teacher names a letter, writes it on the board, and the class practices making the sounds associated with that letter out loud together.

A

B. The teacher names a letter and writes it on the board, and the children use playdough to form the shape of the letter at their table.

This is a good activity to promote student understanding of the names and shapes of different letters because they will see it, hear it, and have a tactile opportunity to recreate the shape.

398
Q

Even though her students cannot yet read, a preschool teacher labels each of their backpack hooks with their names, labels commonly used items around the room, and has an assortment of books available for them to access independently. Which of the following best explains why this teacher has prioritized making her classroom a print rich environment?

A. to increase student phonemic awareness

B. to develop the student’s ability to recognize, name, and write letters

C. to support and encourage student understanding of the forms and functions of printed language

D. to improve student recognition of irregular, high frequency words

A

C. to support and encourage student understanding of the forms and functions of printed language

A print rich environment is important even before a child learns to read in order to develop an awareness of print.

399
Q

The following description refers to which special education legislation:

A nationwide law that ensures services for students with disabilities. It governs how the state and schools provide early interventions, special education, and related services for three to twenty-one-year-old students with disabilities.

A. No Child Left Behind Act (NCLB)

B. Section 504 of the 1973 Rehabilitation Act

C. Individuals with Disabilities Education Act (IDEA)

D. Family Educational Rights and Privacy Act (FERPA)

A

C. Individuals with Disabilities Education Act (IDEA)

This description matches the guidelines of IDEA.

400
Q

An inclusion teacher is planning for a writing conference with a third-grade student who has completed their rough draft and is ready to begin revising her work. Which of the following should the teacher incorporate into their writing conference to support the students’ revision process?

A. Highlight the first letter of each sentence to check for capitalization.

B. Identify at least two places in the story that would benefit from additional details, elaboration, and/or sensory language.

C. Ask the student to read their essay allowed, underlining spelling errors as they go.

D. Circle any homophones and ask the student if the correct version of the words has been selected.

A

B. Identify at least two places in the story that would benefit from additional details, elaboration, and/or sensory language.

This would help students improve their writing as part of the revision process.

401
Q

A third-grade student with an emotional behavior disorder is engaging in high rates of problem behavior in his current placement. The administrators and behavior specialist decide that an ARD meeting should be held to move the student to a more restrictive placement. What necessary steps should the administrators address in the ARD meeting?

A. who the new case manager will be in the more restrictive placement

B. parental consent for the more restrictive placement

C. school district approval for the movement to a more restrictive placement

D. application of his current goals to the more restrictive placement

A

B. parental consent for the more restrictive placement

Parental consent is required for any move to a more restrictive environment. Students should be educated in their least restrictive environment (LRE) unless agreed upon by the ARD committee (which includes parent/guardian).

402
Q

Which characteristics are common to students with visual impairment?

Select all answers that apply.

A. heightened senses to accommodate for the lower functioning sense of sight

B. repetitive actions such as rocking or flailing hands

C. social isolation from their peers

D. stumbling or uncoordinated movement

A

A. heightened senses to accommodate for the lower functioning sense of sight

Students with one or more senses that do not function like a typical person will be more sensitive to other areas. A student with a vision impairment may have a heightened sense of smell, or he/she may rely on hearing as a primary means of gathering information about his/her surroundings.

D. stumbling or uncoordinated movement

A person with a vision impairment may not have strong depth perception or sharp visual acuity, so they may stumble or appear uncoordinated.

403
Q

A third-grade teacher reads the following passage from a story:

“As Jimmy was brushing his teeth before going to bed, he heard a terrible roar come from the garage. Jimmy didn’t know what could be making that terrible noise, but he left a light on in the closet while he slept that night.”

The teacher then asks students questions about their thoughts on the events of the passage and what might be happening. Which of the following would this activity best promote?

A. mapping

B. fact and opinion

C. schema development

D. predicting

A

D. predicting

This is the correct answer because students are being asked to guess, or predict, the causes of the sound. There should be an expectation that the students explain why they make certain predictions.

404
Q

Why is oral reading often used instead of silent reading when assessing a student’s reading skills and determining their reading level?

A. Hearing the frequency and types of errors can help the teacher determine why the student struggled with the passage.

B. Most students comprehend better when reading aloud, so it provides an opportunity to push beyond their comfort zone.

C. Oral reading for the assessment allows the teacher to compare their oral reading ability to their silent reading ability.

D. Pronunciation errors are the most important part of determining student reading level, and they can only be determined by listening to students read aloud.

A

A. Hearing the frequency and types of errors can help the teacher determine why the student struggled with the passage.

Analyzing miscues can help a teacher identify the cause of comprehension issues and provide guidance on the type of focused instruction needed to support individual students.

405
Q

Miss Hoyt teaches fifth-grade science, and she has lab groups with typical students and students with IEPs. She notices that they are struggling to cooperate and complete the lab in a timely manner. What can she do to help her students practice social skills such as sharing and taking turns?

A. Identify one leader in each group who determines how duties will be shared.

B. Assign students with IEPs to one group and ask the co-teacher to stay with them through the lab as a “coach.”

C. Give a lesson to the whole class on sharing and turn taking using a social story.

D. Give each member a defined role and title. Each role has a set of responsibilities, and they will have a new role with each lab.

A

D. Give each member a defined role and title. Each role has a set of responsibilities, and they will have a new role with each lab.

This helps students learn how to participate in a group by identifying tasks that need to be completed and how they can be divided up. Students will practice shared responsibility since each group member will have a different role, and they will practice taking turns by changing role assignments with each lab.

406
Q

A paraprofessional is helping a high school student transition from his wheelchair to the toilet. Which of these tools should be used by the student?

A. gait belt

B. non-skid shoes

C. arm brace

D. leg brace

A

A. gait belt

A gait belt provides a handle for the person assisting to hold when helping a student stand and turn.

407
Q

Before the school year starts, an elementary teacher plans activities to increase phonological awareness. Which of the skills below will likely be more prevalent earlier in the year as students are beginning to explore phonological awareness?

A. onset and rime

B. listening and rhyming

C. phoneme blending

D. syllable awareness

A

B. listening and rhyming

Listening and rhyming comes before syllable awareness, onset and rime, and phoneme blending.

408
Q

A first-grader with Autism Spectrum Disorder and limited verbal speech in a self-contained classroom is unable to effectively wash his hands after using the restroom. The student currently turns the water on and off then returns to his area. To improve the student’s hand-washing skills, the special education teacher should:

A. physically assist the student in washing his hands when he leaves the restroom.

B. verbally direct him to wash his hands when he leaves the restroom.

C. create a task analysis to teach the student how to wash his hands effectively.

D. provide hand sanitizer outside of the restroom for the student to use.

A

C. create a task analysis to teach the student how to wash his hands effectively.

Using a task analysis can be an effective way to teach a multi-step procedure. This involves breaking down the individual steps of the procedure and teaching them while fading prompt levels until the student can complete the task independently.

409
Q

A special education teacher administers a writing quiz for her fourth-grade small group. The students completed the quiz and results showed that many of the students lacked punctuation and capitalization in their writing samples. What next steps should the special education teacher take?

A. Grade the quizzes and allow the students to see where they missed credit.

B. Continue with the current curriculum but take more credit away for missed punctuation.

C. Remind students that they need to use punctuation and capitalization in their sentences.

D. Incorporate sentence structure review into her curriculum and quiz them again in a week.

A

D. Incorporate sentence structure review into her curriculum and quiz them again in a week.

Providing direct teaching on the skill deficit and re-assessing knowledge afterward is an effective means of addressing skill gaps.

410
Q

Students who have autism are usually more successful in classroom environments that:

A. require students to work as individuals to reduce group work.

B. shift and change throughout the day.

C. have clear schedules and routines.

D. have music playing in the background.

A

C. have clear schedules and routines.

Students with autism typically succeed when a classroom uses clear schedules and routines.

411
Q

An early-childhood special education teacher and a general education pre-k teacher are teaching a morning circle time. There are 3 students with disabilities in the class with varying levels of communication. When planning the lesson, the special education teacher should:

A. offer to provide behavior support during circle time.

B. inform the general education teacher of the IEP goals and help plan on how to embed them within instruction.

C. allow the students with disabilities to participate and assess their IEP goals after circle time.

D. provide the general education teacher with a copy of their IEPs at arrival.

A

B. inform the general education teacher of the IEP goals and help plan on how to embed them within instruction.

Preparing general education lessons that can embed IEP goals is an effective way to directly teach the students within a developmentally appropriate framework.

412
Q

Ms. Aguilar is planning a unit on graphing rational functions. Which of the following topics should Ms. Aguilar review before presenting the problem below?

𝑓(𝑥)=𝑥−2/𝑥−3

A. vertical asymptotes

B. multiplying rational expressions

C. the division algorithm of real numbers

D. the laws of exponents

A

A. vertical asymptotes

To graph a rational function, a student must be able to identify whether the graph has any vertical asymptotes.

413
Q

A first-grade teacher takes three small square tiles and sits across from one of her students. She says, “I’m going to say a word and then its sounds: mat … /m/ … /a/ … /t/.” Then she passes the tiles across the desk to the student and says, “Can you think of another one-syllable word with three sounds?” This activity furthers the student’s reading abilities by helping them:

A. realize the basic building blocks of phonics instruction.

B. learn how to blend individual sounds together.

C. understand that words are made up of separate sounds.

D. recognize the most common sounds in English.

A

C. understand that words are made up of separate sounds.

This activity helps the student learn how to segment individual sounds in a word by creating a visual representation of each separate sound. Segmenting and blending are the two most important aspects of phonemic awareness for students to master as they are learning to read.

414
Q

The IEP team has proposed an initial evaluation for a student who is suspected of having an educational disability of autism. Which of the following areas of evaluation may be recommended?

Select all answers that apply.

A. communication assessment

B. achievement assessment

C. gross motor assessment

D. cognitive assessment

A

A, B, D

One of the characteristics of autism is communication deficits. This may include difficulties with expressive, receptive, or pragmatic language.

Achievement testing is often a standard portion of the initial evaluation. This information will be used to demonstrate whether the student’s achievement is commensurate with his/her cognitive ability.

Cognitive testing is often a standard portion of the initial evaluation. This information will be used to demonstrate whether the student’s cognitive ability is commensurate with his/her achievement scores.

415
Q

Which two of these would be appropriate accommodations for a student while maintaining the integrity of the Texas Essential Knowledge and Skills (TEKS)?

Select all answers that apply.

A. A ninth-grade student is working on maps in World Geography. He has a visual impairment, so the maps are enlarged to 20-point font size, and he is provided large-type labels with black print on a white background to place on the countries, rather than writing the name over the color.

B. A seventh-grade student with an intellectual disability is learning about human body systems. He is required to label each body system on a concept map rather than describing each body system and identifying the organs associated with each system.

C. A fifth-grade student with severe dyslexia is working on a research paper. The student will be graded like other students, except his spelling will not count until the final draft. He uses a word processor to assign spelling corrections on rough drafts..

D. A kindergarten student is working on letter recognition. The student has a physical disability that requires her to use a walker. The teacher decides to only require her to learn the capital letters, but not the lowercase letters.

A

A

These are reasonable accommodations that allows the student full access to the curriculum while accommodating for his disability by providing him with enlarged text and labels.

C

This is a reasonable accommodation that allows the student to access the full curriculum while providing a reasonable accommodation that does not penalize him for his disability.

416
Q

What is the primary goal of summative assessments?

A. ranking students

B. valuating teachers

C. making instructional decisions

D. measuring student achievement

A

D. measuring student achievement

Summative assessments are used to evaluate student learning.

417
Q

Aliyah is a first-grade student with a specific learning disability. Mid-year assessment data shows that she is on grade level in mathematics, but recently she has struggled with interpreting data from simple bar graphs. Which of the following steps should Aliyah’s teacher take first?

A. pairing Aliyah with a peer who can assist her with problems related to bar graphs

B. providing Aliyah with tier 3 RTI support related to analyzing graphs

C. having Aliyah retake the mid-year TEKS math assessments to ensure that the results were accurate

D. providing in-class, small group intervention for Aliyah and peers who are struggling with the same concept

A

D. providing in-class, small group intervention for Aliyah and peers who are struggling with the same concept

This is the most appropriate step to take in this scenario. Aliyah is otherwise on grade level in math and likely just needs additional instruction related to bar graphs. This can be achieved through small group instruction with her classroom teacher.

418
Q

A middle school student with a mild intellectual disability has difficulty recalling basic addition and subtraction facts. Which of the following strategies would most likely help the student meet the math standards for their grade level?

A. providing the student with a math fact sheet that can be used while working on assignments or tests

B. allowing the student to use a calculator for basic addition and subtraction facts

C. allowing the student to have additional time to complete assignments

D. providing pull-out support in which the student practices basic math facts

A

B. allowing the student to use a calculator for basic addition and subtraction facts

A basic calculator that can be used for addition and subtraction is an appropriate strategy to use for this student. This will allow the student to solve more complex math problems without having to remember basic addition and subtraction facts.

419
Q

The Code of Ethics of Texas Educators summarizes expectations for teachers personally and professionally. According to the code, can a teacher be held accountable for material posted on personal social media?

A. A teacher can be held accountable if the content is accessed or posted while on school property or while using school materials such as a teacher’s computer.

B. A teacher cannot be held accountable because the teacher is allowed freedom of speech through personal accounts.

C. A teacher can be held accountable only if the Local Education Agency (LEA) has a written policy about personal social media accounts.

D. A teacher can be held accountable because the code refers to conduct both inside and outside of school.

A

D. A teacher can be held accountable because the code refers to conduct both inside and outside of school.

Teachers can be held accountable for any social media posts no matter when or where they are made.

420
Q

Multiple times a year a third-grade teacher administers a formal assessment of her students’ oral reading fluency. While assessing a student’s reading, she notices that the child performs well on the portion of the test that asks the student to read a paragraph, but struggles when reading many of the same words in list form. Which of the following would best describe a strength of this student?

A. using context clues to determine a word

B. sight word memorization

C. structural analysis

D. using morphology to determine word meaning

A

A. using context clues to determine a word

If a student is reading words more easily when they are in a sentence or paragraph compared with reading them in isolation, it is likely that they are using context clues to aid in their decoding and word analysis. For example, a student my see a word like “dentist” in isolation and not recognize it, but easily read it in the sentence, “My tooth hurt so I went to the dentist.”

421
Q

The special education teacher collects data on a student’s progress on phonemic awareness to determine:

A. if the student is working on phonemic awareness activities at home

B. what the general education teacher is working on during reading intervention.

C. if further intervention is needed to meet criteria and plan for future instruction.

D. how many letter sounds the student knows.

A

C. if further intervention is needed to meet criteria and plan for future instruction.

Data collection and assessment are the means with which special education teachers make informed decisions on student progress. This can inform future instruction and allow evidence for further intervention, if needed.

422
Q

Following a school-wide dyslexia screening exam, a first-grade student is flagged for further evaluation. What information should the teacher review for further evidence?

A. scores on standardized reading curriculum and evaluations

B. observe the student in the library with peers

C. inform the parent that the student will need dyslexia services

D. have the student write specific letters and phrases to evaluate their writing

A

A. scores on standardized reading curriculum and evaluations

Screening tools are the first step toward identifying a potential disability. However, following a screening tool, records should be reviewed to determine if the student has frequently struggled with reading to determine if the flagged score could have been a false positive.

423
Q

A fifth-grade student has been evaluated by a Licensed Specialist in School Psychology (LSSP) who determines that he should be diagnosed with an emotional behavior disorder. The student’s parents are concerned about the stigmatization of this diagnosis. The special education teacher should:

A. provide the parent with academic literature on emotional behavior disorders.

B. acknowledge the parents’ concerns and explain the services the student can now receive.

C. ask the LSSP to change the diagnosis to something less stigmatizing.

D. dismiss the parents’ concerns and outline the student’s new services.

A

B. acknowledge the parents’ concerns and explain the services the student can now receive.

Parent concerns should be acknowledged and accepted. While it is ultimately the parent’s decision, explaining the services that can assist the student can help to ease parent anxiety.

424
Q

Which of the following must be acquired before evaluation for special education services?

A. data collection on behavior

B. an ARD meeting to review other evaluations

C. a parent questionnaire to determine method of evaluation

D. consent for individual evaluation

A

D. consent for individual evaluation

Before beginning any evaluation, parental consent for individual evaluation must be obtained.

425
Q

A first-grade student with autism spectrum disorder is having difficulty with classroom transitions. To help the student, the special education teacher should:

A. continue with the current level of support.

B. inform the general education teacher.

C. provide a visual schedule for the student.

D. remove the student from the classroom during transitions.

A

C. provide a visual schedule for the student.

By providing a visual schedule, the student will have a clear picture of upcoming transitions throughout the day. Visual schedules have been shown to provide appropriate transition support for students with autism.

426
Q

Benjamin is a fourth-grade student with an intellectual disability who is currently performing at a first-grade level in math. Benjamin is able to partition two-dimensional shapes into halves and fourths. His general education teacher is currently planning a lesson on comparing fractions with different numerators and denominators. Which of the following modifications should the teacher plan on providing for Benjamin?

A. having Benjamin spend additional time in the special education classroom during this lesson

B. providing alternate assignments in which Benjamin shades different portions of shapes divided into halves and fourths and then circles the shape showing the larger fraction

C. allowing Benjamin to use a calculator to determine the decimal value of each fraction

D. providing one-on-one instruction for Benjamin in which he is taught to use fraction tiles to represent fractions such as 3/8 or 5/12 to identify the larger fraction

A

B. providing alternate assignments in which Benjamin shades different portions of shapes divided into halves and fourths and then circles the shape showing the larger fraction

This is an appropriate modification in this scenario as it correlates to the lesson being taught to other students in the classroom and builds upon Benjamin’s current abilities.

427
Q

A first-grade teacher would like to improve her students’ ability to track print when it appears on consecutive lines in a book. Several students in the class have a good understanding of the alphabet, corresponding letter sounds, and simple decoding, but they struggle to understand longer lines of text. Which of the following strategies could she use as she models reading a short paragraph containing simple vocabulary in front of the class?

A. using her finger to touch each word while reading slowly from left to right on the page

B. highlighting each word of the text as she reads it from left to right on the page

C. identifying unfamiliar words by skimming the text to locate, circle, and define the words

D. setting a timer to demonstrate how fluent readers gain more information from the text

A

A. using her finger to touch each word while reading slowly from left to right on the page

Touching each word one at a time while reading from left to right is a good example of a strategy to help students track text as they read.

428
Q

A fourth-grade student with autism spectrum disorder has difficulty making and maintaining friendships in his general education classroom. To help address this, the special education teacher should:

A. create a social skills small group to practice trading information and conversations.

B. prepare a script for the student to use in his general education classroom.

C. encourage the student to try to make friends with his classmates.

D. address the entire class and re-introduce the student to them.

A

A. create a social skills small group to practice trading information and conversations.

Social skills deficits are common in individuals with autism spectrum disorder. By providing opportunities to directly teach conversation skills, the special education teacher can assist the student in developing the social skills needed to help maintain friendships.

429
Q

A special education teacher is incorporating the use of readers’ theater into her weekly literacy instruction. Which of the following reading skills is this teacher most likely focusing on with this activity?

A. vocabulary

B. metacognition

C. comprehension

D. fluency

A

D. fluency

Readers’ theater incorporates oral reading with an emphasis on prosody.

430
Q

A special education teacher creates a task analysis for loading a dishwasher. This teacher is most likely:

A. providing a visual for appropriate use of the teacher’s lounge dishwasher.

B. providing transition curriculum to students with deficits in independent living skills.

C. conducting research on household tasks.

D. creating a more efficient way for students to assist in cleaning the classroom.

A

B. providing transition curriculum to students with deficits in independent living skills.

Task analyses can be used to efficiently teach individuals with disabilities various multi-step tasks, such as loading a dishwasher or personal hygiene.

431
Q

A third-grade student with no history of seizures begins having a seizure in the special education classroom. In this case, the first thing the teacher should do is:

A. monitor the student during the seizure.

B. instruct a staff member or administrator to call 911.

C. remove the other students from the classroom.

D. remove any items from around that student that could be harmful.

A

B. instruct a staff member or administrator to call 911.

Students with a history of seizure may have a plan in place that does not require a 911 call; however, any student with no history of seizures should receive emergency medical attention.

432
Q

Which of the following guarantees free and appropriate public education for individuals with disabilities in the United States?

A. Individuals with Disabilities Education Act (IDEA)

B. Mills v. Board of Education of the District of Columbia

C. Brown v. Board of Education

D. Larry P. v. Riles

A

A. Individuals with Disabilities Education Act (IDEA)

The Individuals with Disabilities Education Act (IDEA) guarantees that all students with disabilities can have access to free and appropriate public education.

433
Q

A fourth grader, Vincent, is struggling with divisibility rules. He cannot consistently remember and apply all of the rules when given a number which frustrates him. The teacher feels that his behavior is becoming increasingly disruptive during math time. He has a behavior intervention plan (BIP) and accommodations. What should the special education teacher recommend?

A. Create hands-on activities that will increase his concrete understanding of the rules.

B. Request a committee meeting so that the accommodation of using a calculator can be added.

C. Provide only questions that test his ability to use division rules until he has mastered it.

D. Address the issue now while continuing with the current planned curriculum.

A

D. Address the issue now while continuing with the current planned curriculum.

This does need to be addressed, but Vincent cannot miss other concepts due to frustration with this one. He can be provided a rules sheet when working division problems to assist him until he has memorized the facts. The reacher should also review Vincent’s BIP and accommodations to ensure that the plan is followed with fidelity, especially during math time.

The issue here is memorization of the facts, not the understanding of them. Concrete examples will not help him master this.

434
Q

What is the difference between reliability and validity in test scores?

A. Reliability refers to test scores and validity refers to accommodations.

B. Reliability refers to consistency and validity refers to accuracy.

C. Reliability and validity are the same thing.

D. Reliability refers to accuracy and validity refers to consistency.

A

B. Reliability refers to consistency and validity refers to accuracy.

This is correct: reliable test scores provide the same information regardless of the test and valid test scores provide an accurate assessment of how many answers were correct.

435
Q

During the initial transition meeting, the student expresses interest in a hospitality career. Which of the following would be the appropriate next step?

A. Inform the student that a hospitality career is not for him.

B. Inform the parent to research possible job availability in local restaurants and hotels.

C. Engage the student to research skills needed to work in the hospitality career.

D. Collaborate with special education personnel to schedule time for the student to work in the campus coffee shop to gain experience and skills needed to transition to an integrated employment opportunity in the hospitality field.

A

D. Collaborate with special education personnel to schedule time for the student to work in the campus coffee shop to gain experience and skills needed to transition to an integrated employment opportunity in the hospitality field.

The key words are “collaboration, experience, skills, and integrated employment.”

436
Q

A third-grade class is reading a biography of a famous inventor. Before reading, the teacher shows images of some of the inventions and the class discusses how they have interacted with the inventions in their lives. While reading, the teacher pauses to ask herself, “Why did he switch from copper to aluminum for the outside? Let me read this again.” Then reviews the text aloud to find the answer. The teacher’s actions while reading best depict her knowledge of which factor of reading comprehension?

A. monitoring for comprehension while reading

B. demonstrating vocabulary knowledge while reading

C. engaging background knowledge while reading

D. self-correcting for accuracy while reading

A

A. monitoring for comprehension while reading

Students need to know how to check for comprehension and ask self-monitoring questions while reading independently. Teachers can encourage this skill by modeling the skill during whole-group readings.

437
Q

A special education teacher receives instructions from her principal on how to file a section of IEP paperwork. These instructions are contrary to guidelines given by the special education coordinator and the state education agency. The teacher should:

A. file the paperwork using the principal’s instructions but inform the principal of the contradicting policy.

B. file the paperwork according to the principal’s instructions and inform the special education coordinator.

C. file the paperwork according to the principal’s instructions.

D. refuse to file the paperwork using the principal’s instructions, citing the district and state policy.

A

D. refuse to file the paperwork using the principal’s instructions, citing the district and state policy.

While the principal is the teacher’s direct superior, filing the paperwork would violate guidelines from local and state entities. By citing these policies, the teacher can respectfully inform the principal of the violation.

438
Q

A fourth-grade student receives services for an emotional disability due to a diagnosis of generalized anxiety disorder. His anxieties seem to be heightened when dealing with unexpected changes to his routine. Which of the following positive behavioral supports would be most appropriate for him?

A. reviewing a daily schedule with him and highlighting any events that are atypical

B. letting him know when the fire drills will occur

C. having a teacher escort him throughout the day when there is an assembly or schedule change.

D. excusing him from school events such as guest speakers or assemblies, to maintain his routine

A

A. reviewing a daily schedule with him and highlighting any events that are atypical

Reviewing the daily schedule and identifying atypical events gives him a chance to process the changes before they occur and be aware of what is coming.

439
Q

A second-grade student is being monitored for reading using a multi-tiered Response to Intervention model. The student has been receiving tier 2 supports in his general education classroom for six months and is continuing to struggle on standardized assessments. The next steps for the teacher should be:

A. suggest implementing Tier 1 supports

B. allow more time to see if the Tier 2 supports will be effective

C. adjust the content of the standardized assessments to fit the student’s needs

D. refer the student for evaluation and possible Tier 3 supports

A

D. refer the student for evaluation and possible Tier 3 supports

If progress is not made over a period of time using Tier 2 supports, referral for more services and possible evaluation for special education is needed (i.e. Tier 3).

440
Q

A fifth-grade student with an intellectual disability is having difficulty completing a narrative writing assignment. The special education teacher is asked to provide appropriate writing supports for the student. The teacher should provide:

A. a dictionary.

B. a graphic organizer to help the student outline the assignment.

C. a writing assignment on a different topic.

D. a list of commonly misspelled words.

A

B. a graphic organizer to help the student outline the assignment.

A graphic organizer can be used to assist in narrative writing assignments. This support would be an appropriate provision within the least restrictive environment.

441
Q

Elena is a fifth-grade student with a specific learning disability in reading. Elena’s reading fluency level is slightly below grade level, but she has significant deficits in reading comprehension skills and responds to reading comprehension questions on a second grade level. Based on this information, what would be an appropriate objective to meet Elena’s current needs in the area of reading?

A. When given a third-grade-level text, Elena will respond to questions pertaining to the main idea, characters, and setting with 80% accuracy.

B. When given a second-grade level text, Elena will read at a rate of 120 words per minute with less than 2 errors.

C. When given a fifth-grade-level text, Elena will accurately identify the beginning, middle, and end of the story.

D. When given a fifth-grade level text, Elena will respond to questions pertaining to the main idea, characters, and setting with 100% accuracy.

A

A. When given a third-grade-level text, Elena will respond to questions pertaining to the main idea, characters, and setting with 80% accuracy.

Elena reads fluently but struggles with reading comprehension. Since her reading comprehension skills are currently at a second grade level, this objective is most appropriate.

442
Q

A first-grade student with a specific learning disability in language is struggling to understand letter-sound correspondence. Which of the following would be the most appropriate activity to support this students’ letter-sound understanding?

A. singing the alphabet song

B. making a letter sound while tracing the letter with a pencil and paper

C. shaping their body into an object whose name starts with a given sound

D. arranging the class into alphabetical order based on the students’ last names

A

B. making a letter sound while tracing the letter with a pencil and paper

This is the most appropriate activity because students will be identifying a specific letter with a specific sound.

443
Q

Every Student Succeeds Act (ESSA), No Child Left Behind (NCLB), and Individuals with Disabilities Education Act (IDEA) have changed the landscape of special education services. What is the greatest impact in relation to placement of special education students that has occured?

A. Students are being placed in general education for elective courses, while they continue to attend small, specialized classrooms with other similarly-disabled peers for core content areas.

B. Students are being placed in general education classrooms with typical, grade-level peers at a higher rate.

C. Students are being placed in small, specialized classrooms with other similarly-disabled peers at a higher rate.

D. Students are being placed with grade-level peers for science, social studies and electives, but math and language arts are separate as they are foundational for other subjects.

A

B. Students are being placed in general education classrooms with typical, grade-level peers at a higher rate.

This is true, as the least restrictive environment calls for students to participate with non-disabled grade-level peers as much as is possible. Students also have full access to the state-mandated curriculum.

444
Q

A special education teacher receives a request from a local law enforcement agency. They provide her with a subpoena that requests all communication (emails, texts, call logs) with one of her student’s guardians. The special education teacher should:

A. provide the agency with the requested communications.

B. provide communication from district devices but not her personal cell phone.

C. not provide the agency with any communications.

D. request that she receive parental consent before releasing the communications.

A

A. provide the agency with the requested communications.

When a law enforcement agency produces a subpoena for records, the teacher must turn them over regardless of parent consent or device.

445
Q

Mr. Larry provides in-class support for 4th- and 5th-grade students at Treasure Valley Elementary. Many of the students that he supports do not turn in their homework, get pencils, or begin their morning work without individual prompts, so he creates a visual checklist for them that is on the corner of their desks. Each morning, he simply taps the checklist as he walks by their desks and they begin to complete tasks on the list. The greatest benefit of this approach is that it:

A. increases independent skills and places more responsibility on the student.

B. does not interrupt the general education teacher.

C. decreases Mr. Larry’s workload so that he can focus on other morning tasks.

D. provides students with an example so that they can create their own checklists for other classes.

A

A. increases independent skills and places more responsibility on the student.

This approach allows the students to independently complete routine tasks. As students demonstrate mastery, Mr. Larry may phase out his individual prompts to begin tasks.

446
Q

Norm-referenced tests are those where

A. Anecdotal records are collected to determine student achievement.
B. The student’s parents are interviewed to determine student achievement
C. The student’s performance is compared to an external criterion to determine achievement
D. The student’s performance is compared to his or her peers to determine achievement.

A

D. The student’s performance is compared to his or her peers to determine achievement.

Norm-Referenced tests are used to see a student’s performance and compared to their peers (or age group) to see their progress/achievement

447
Q

Which of the following behaviors demonstrated by a 2-year-old child is the clearest indicator that the motor development of that child may be impaired?

A. Difficulty coordinating hands and eyes
B. Inability to climb stairs with alternating feet
C. Frequently falling while running
D. Making involuntary hand movements

A

Option D is correct because involuntary hand movements in a 2-year-old child can be a result of developmental delays. Options A, B, and C are incorrect because having difficulty with hand-eye coordination, being unable to climb stairs with alternating feet, and falling down while running are developmentally appropriate behaviors for many 2-year-old children.

448
Q
  1. A fifth grader has diabetes and manages her illness through insulin injections. Which of the following symptoms should indicate to a teacher that this student may be experiencing insulin shock?

A. Trembling and irritability
B. A high fever
C. Muscle and joint pain
D. An increased level of thirst

A

Option A is correct because trembling and irritability are symptoms of insulin shock — a condition that occurs when a person with diabetes forgets to eat after an insulin shot and ends up with too much insulin in the blood. Options B, C, and D are incorrect because high fever, muscular pain, and increased thirst are not typical symptoms of insulin shock.

449
Q

Which TWO of the following characteristics are common to students with both autism spectrum disorder and a hearing impairment?

A. Displaying social isolation or awkwardness
B. Exhibiting difficulty with word endings or quiet speech sounds
C. Manifesting difficulty with comprehending contextual or alternate meanings of words
D. Engaging in repetitive activities or rituals

A

Options A and C are correct because displaying social isolation or awkwardness and manifesting difficulty comprehending contextual or alternate meanings of words are both characteristics of students with autism spectrum disorder and students who have a hearing impairment. Option B is incorrect because exhibiting difficulty with word endings or quiet speech sounds is a characteristic of students with a hearing impairment. Option D is incorrect because engaging in repetitive activities or rituals is characteristic solely of students with autism spectrum disorder.

450
Q

Ms. Smith stops the class after just a few minutes of mathematics instruction, which is taking place at the board. She asks two questions requiring simple student responses on note cards. After quickly reviewing the cards, Ms. Smith returns to instructing at the board. Which type of assessment is the teacher using?

A. Summative
B. Formal
C. Formative
D. Alternate

A

Option C is correct because a formative assessment refers to a wide variety of methods that teachers use to conduct in-process evaluations of students’ comprehension and learning needs to adjust their instruction. Option A is incorrect because a summative assessment measures students’ progress at the end of a unit. Option B is incorrect because formal assessments are generally scored using percentiles, stanines, or standard scores. After being scored, they provide data beyond just the score. Option D is incorrect because an alternate assessment measures students’ performance using portfolios and presentations.

451
Q

Alyssa is a third grader who has been assessed and found to be significantly below her grade-level peers in mathematics calculation and problem solving. To indicate that Alyssa qualifies for special education as a student with a specific learning disability in mathematics, Alyssa’s evaluation must also show that she exhibits a

A. communication deficit when describing math concepts.
B. discrepancy between achievement and intellectual ability.
C. lack of ability to match concrete models with numbers or problems.
D. shorter attention span and a lesser ability to focus than her grade-level peers exhibit.

A

Option B is correct because the specific criterion for a learning disability, other than low achievement, is a discrepancy between achievement and intellectual disability. Option A is incorrect because it does not specifically relate to a learning disability. Option C is incorrect because it is not part of the specific criteria for determining a learning disability. Option D is incorrect because this may relate to ADHD, but it does not relate to a specific learning disability.

452
Q

Which of the following statements best describes how the objectives in a student’s Individualized Education Program (IEP) should relate to the Texas Essential Knowledge and Skills (TEKS)?

A. The IEP objectives should be identical to those of the TEKS, with modifications in grade level only.
B. The IEP objectives should be developed independently and then correlated with the TEKS.
C. The IEP objectives should reflect the TEKS as closely as possible.
D. The IEP objectives should be developed to meet student needs, with no reference to the TEKS.

A

Option C is correct because according to the Individuals with Disabilities Education Act (IDEA), a student with a disability is expected to be involved and progressing in the general education curriculum presented in the TEKS. Option A is incorrect because the IEP objectives must reflect the appropriate grade level of the student and can be tweaked to reflect the student’s strengths and weaknesses. Option B is incorrect because IEP objectives must be developed in correlation with the TEKS. Option D is incorrect because the IEP objectives must relate to the TEKS to ensure that the student has access to the general education curriculum to the maximum extent possible.

453
Q

Which of the following strategies will be most effective for providing job training to a student with intellectual disabilities for work as a server in a fast-food restaurant?

A. Pairing the student with an employee who can work beside the student and model the required tasks
B. Making a list of job tasks for the student to refer to as necessary throughout his or her work shift
C. Carefully explaining the steps of the job to the student and checking on the student at regular intervals
D. Simulating the work environment and practicing the job sequence in the classroom

A

Option A is correct because the student will learn the job of a fast-food server best through hands-on training and by having the tasks broken down into smaller, more manageable tasks. Option B is incorrect because a student with an intellectual disability may not be able to follow a long list of tasks. Option C is incorrect because a student with an intellectual disability will most likely not be able to accurately follow such verbal directions. Option D is incorrect because participating in a classroom simulation is not as effective as learning in the workplace.

454
Q

Which THREE of the following scenarios best describe instructional activities that are appropriate for the student and aligned with the Texas Essential Knowledge and Skills (TEKS)?

A. Sixth-grade reading students are expected to independently read two nonfiction articles and compare and contrast the authors’ points of view. Tazio, a student with a learning disability, is provided with a digital graphic organizer that includes pictorial representations to help him complete the assignment.

B. Ninth-grade biology students are expected to create a food chain representative of a local ecosystem. The teacher encourages students to use drawings and photographs to show the transfer of energy from one living thing to another. Maria, a student with a severe visual impairment, is allowed to write a song about the predator-prey relationship in a local food chain and teaches her class the lyrics.

C. Third-grade math students are expected to solve one- and two-step problems using data from bar graphs and pictographs. After modeling how to create a bar graph about favorite foods, the teacher asks Neda, a student with dyscalculia, to shade in the number of students whose favorite food is pizza.

D. Tenth-grade United States history students learn about major historical figures in the Civil War era. Cooperative groups are expected to study an influential person and write and perform a eulogy for that person. Alex, a student with a mild orthopedic impairment, reads aloud the eulogy that he and other group members prepared.

E. Twelfth-grade English students are assigned a classical work of fiction. They are expected to analyze how the work is shaped by the narrator’s point of view and then support their ideas with evidence from the text. Robert, who has severe dyslexia, listens to the text via audiobook and draws and labels a scene from the book with the help of a paraprofessional.

A

Option A is correct because special education students are guaranteed curriculum access by using digital graphic organizer. Option B is correct because allowing visually impaired students to present projects in a nonvisual format ensures that they have access to the TEKS-based curriculum. Option D is correct because participating in the research and writing process and the group presentation allows students equitable access the curriculum. Option C is incorrect because the expectation of shading the graph does not meet the standard of TEKS and denies the student access to the curriculum. Option E is incorrect because, while the use of audio is an appropriate modification, the required assignment does not meet the TEKS requirement.

455
Q

Early in the school year, the students in a fifth-grade class have been demonstrating exclusionary behavior. There are two dominant cliques, and there are several children, including two with disabilities, who are generally treated as outcasts. When considering how to respond, the classroom teacher should be aware that

A. in such situations, it is generally best to let the students work out the issues among themselves rather than to intervene in their social relationships.
B. the most effective response would be to ask the parents of the students in the cliques to apply disciplinary measures if their children continue the exclusionary behavior.
C. his or her response should be aimed primarily at bolstering the resilience and self-assertiveness of the children with the lowest social status.
D. it is imperative to institute a program specifically designed to build a learning community in which all students feel safe and accepted.

A

Option D is correct because it is the teacher’s responsibility to create a safe classroom environment that is conducive to learning. Option A is incorrect because children need to learn how to accept everyone. Option B is incorrect because if the cliques happen in the classroom, the teacher needs to address the issue and the parents should be involved only if needed. Option C is incorrect because the classroom teacher is responsible for all the students in the classroom and should address the whole class and not just students who display low self-esteem.

456
Q

An adult is preparing to carry a small preschooler with multiple disabilities. Which of the following practices will best ensure the adult’s safety during the lift?

A. Prompting the child to hold on to the adult during the lift to the greatest extent possible

B. Maintaining a straight back and lifting with the legs

C. Performing the lift as quickly as possible in a way that is consistent with the child’s safety and comfort

D. Initiating the lift by bending from the waist

A

Option B is correct because keeping the back straight and lifting with the legs will help prevent back injuries in the adult and will ensure the safety of the child. Option A is incorrect because the child may not be able to hold on to the adult. Option C is incorrect because lifting the child too quickly may be potentially dangerous for the adult and may result in an injury. Option D is incorrect because bending from the waist may result in back injuries to the adult.

457
Q

When choosing an assistive technology for a student with a disability, a teacher should most importantly consider which of the following?

A. Will the technology reinforce the student’s strengths and eliminate the need for remediation?

B. Will the technology help the student function as he or she would without having a disability?

C. Will the technology only be considered for the student if he or she has an intellectual disability?

D. Will the technology be suitable for the student’s individual needs and be appropriate for the situation?

A

Option D is correct because the student’s specific needs and the setting in which the technology will be used should be considered when choosing any assistive technology device. Option A is incorrect because the role of assistive technology is not to eliminate the need for remediation but to provide support. Option B is incorrect because it is unrealistic to expect any assistive technology device to cure or eliminate the student’s disability. Option C is incorrect because the scenario does not mention that the student has an intellectual disability. Assistive technology can benefit students with a variety of disabilities, not just those with intellectual disabilities.

458
Q

A middle school special educator has been teaching students with learning disabilities how to use chapter headings, subheadings, indexes, and glossaries in their textbooks when doing content-area reading. The greatest benefit of the approach the educator is using is that it

A. promotes the students’ use of critical-thinking skills.

B. enables the students to synthesize new information quickly.

C. encourages the students to relate new information to prior knowledge.

D. gives the students tools for taking control of their own learning.

A

Option D is correct because teaching students how to use the chapter headings, subheadings, indexes, and glossaries in their textbooks empowers them to self-monitor and control their own learning. Option A is incorrect because learning how to use the different elements of a textbook does not promote critical thinking. Option B is incorrect because using the different elements of a textbook will not help students synthesize information quickly. Option C is incorrect because using the textbook tools will not encourage students to connect new information to prior knowledge.

459
Q

A special educator is teaching Monica, a 12 year old with multiple disabilities, how to identify pennies, nickels, dimes, and quarters. Every day, the teacher gives Monica 20 coins to identify. With the help of a chart showing each coin and its name, Monica is asked to use her speech-generating communication device to identify each coin. The results after one week of instruction are shown below.

Day Number of Coins Identified Correctly
Monday 5
Tuesday 6
Wednesday 4
Thursday 3
Friday 4

Based on the assessment’s results, the special educator’s next step should be to

A. try a different instructional approach.

B. repeat the same activity with Monica the following week.

C. state the value of the 20 coins.

D. give Monica a reward every time she identifies a coin correctly.

A

Option A is correct because the current instructional strategy does not seem to be helping the student, and a different approach may prove more helpful to the student. Option B is incorrect because repeating the same activity and the same teaching style is most likely not going to be effective. Option C is incorrect because stating the value of the coin will give away the answer. Option D is incorrect because if Monica struggles with the concept of identifying coins, increasing positive reinforcement will most likely not help.

460
Q

Which of the following instructional strategies will most likely support the needs of a gifted elementary student who is eligible to receive special education services?

A. Focusing on talent development while providing supports for deficits

B. Providing shortened assignments with specifically defined tasks

C. Limiting external stimulation while presenting an advanced curriculum

D. Assigning classroom assignments that can be completed independently

A

Option A is correct because interventions should focus on developing the talent while attending to the disability. Option B is incorrect because simplifying tasks may increase frustration for some gifted ADHD students who would better handle more open-ended and intriguing tasks. Option C is incorrect because decreasing stimulation may be counterproductive with some gifted ADHD children, who as a group tend to be intense and work better with a high level of stimulation. Option D is incorrect because providing assignments that can be completed independently will not foster either gifted or ADHD students.

461
Q

Ten months ago, 16-year-old Andrew suffered a head injury in an automobile accident and was left legally blind. Since the accident, Andrew seldom leaves his house. A few friends occasionally come over to listen to music, but Andrew fears that they will get bored and stop visiting. Of the following, Andrew’s special educators’ best response to the situation would be to

A. suggest a variety of new and interesting activities that can be done at home and that his friends are likely to enjoy.

B. help Andrew develop proficiency with electronic media that will both entertain him and allow interactions with others from home.

C. ask Andrew what social activities he would like to participate in outside the home and help him develop the skills for doing so.

D. arrange for Andrew to begin socializing with peers who are blind or visually impaired.

A

Option C is correct because by helping Andrew explore his own interests and develop certain skills based on his interests, he will be able to participate in social activities with his peers and also gain self-esteem. Options A, B and C are incorrect because they are restrictive and will not allow Andrew to develop social skills that are based on his own interests.

462
Q

Julio, a sixth grader with a learning disability, immigrated with his family to the United States a year ago. Although Julio speaks English well, he never speaks to his classroom teacher unless spoken to. Concerned about this behavior, the classroom teacher suggests to the special education teacher that Julio may need social skills instruction. Which of the following factors is most important for the special educator to take into account when considering the classroom teacher’s suggestion?

A. Teachers can misperceive and misunderstand students’ behaviors by interpreting them from a single perspective.

B. Self-confidence with regard to schoolwork depends primarily on a student’s belief in his or her ability to succeed through effort.

C. Low self-esteem can be a sign of abuse or neglect, and it is a teacher’s responsibility to report all instances of suspected child abuse.

D. General education teachers are often unclear about the special educator’s role and make unfounded demands for assistance.

A

Option A is correct because cultural differences can easily be misinterpreted as problematic behavior. Different cultures have different attitudes toward teachers. Speaking to the teacher only when spoken to may be a cultural preference and may not be related to a lack of social skills. Options B and C are incorrect because there is no evidence in the scenario that the student lacks self-confidence or is being abused at home. Option D is incorrect because the special education teacher should first evaluate the teacher’s request before making a judgment about whether the student’s attitude reflects a lack of social skills.

463
Q

A second-grade student with an emotional behavioral disorder (EBD) is in an inclusion class. The student has violent outbursts that are frequent and seem to increase with time. Which of the following actions is most appropriate for the special education teacher to take to address the student’s behavior?

A. Amending the student’s Individualized Education Program (IEP) to include new behavior goals

B. Imposing stringent consequences to deter the onset of undesirable classroom behaviors

C. Referring the student to the school counselor for redirection during the behavioral occurrence

D. Conducting a functional behavioral assessment (FBA) to evaluate the reason for the behaviors

A

Option D is correct because by conducting a functional behavioral assessment, the teacher will discover the triggers for the student’s increasingly violent behavior. Option A is incorrect because the IEP will not be amended unless the teacher discovers the trigger for the behavior and establishes a behavior plan that works. Option B is incorrect because imposing a strict consequence without flexibility may escalate the negative behavior. Option C is incorrect because referring the student to the school counselor for redirection each time the behavior happens is a temporary solution and will not help to identify the triggers.

464
Q

For a third grader with attention-deficit/hyperactivity disorder who exhibits disruptive behavior and a low attention span, which of the following behavioral intervention strategies is most likely to help the student achieve classroom success?

A. Arranging for the student to complete an assignment independently at the back of the classroom

B. Redirecting the student to focus on classroom rules each time the negative behavior occurs

C. Decreasing the duration of any activity by breaking it down into smaller, more manageable sections

D. Ignoring the negative behavior each time it occurs in the hope that it will pass

A

Option C is correct because breaking down the activity into more manageable sections will allow the student to remain on task and complete assignments successfully without disruption. Option A is incorrect because asking the student to complete assignments independently at the back of the classroom will not help the student remain on task or allow the student to seek assistance from the teacher easily. Option B is incorrect because reminding the student of the classroom rules several times during a lesson may disrupt the class and is therefore not an appropriate strategy. Option D is incorrect because ignoring a negative behavior will escalate the behavior and disrupt the whole class.

465
Q

Of the following, the most effective means of providing program continuity for children transitioning from preschool to kindergarten is to offer

A. a preschool program that stresses the development of independent work habits and the ability to follow directions.

B. a developmentally appropriate program in both preschool and kindergarten that is responsive to individual differences.

C. a preschool environment that strongly emphasizes the development of literacy skills.

D. an environment in both preschool and kindergarten that engages children in collaborative planning and problem solving.

A

Option B is correct because preschool students need a developmentally appropriate program in kindergarten that will improve their self-esteem and develop their cultural identities. Option A is incorrect because preschool students are not mature enough to work independently. Option C is incorrect because preschool students should learn a variety of skills, not just literacy skills. Option D is incorrect because preschool- and kindergarten-age students are not mature enough to collaborate and solve problems on their own.

466
Q

At an Admission Review Dismissal (ARD) committee meeting for a 4-year-old student with a mild intellectual disability, it was decided that the student will transition from a self-contained preschool classroom to an inclusion kindergarten classroom. Which of the following actions is most appropriate for the current special education teacher to take to make sure that the student has a smooth transition?

A. Finding opportunities for the student to interact with nondisabled peers to foster the acquisition of social skills

B. Pre-teaching the kindergarten curriculum to help the student keep up with nondisabled peers academically

C. Concentrating instruction in the area of reading so that the student will have mastered the alphabetic principle prior to entering kindergarten

D. Providing the student with the schedule of the new classroom so that the student becomes accustomed to the new routine

A

Option A is correct because for a four-year-old student with an intellectual disability, learning social skills is crucial in order to be successfully included with typically developing, same-aged peers. Option B is incorrect because a focus on academic skills will not prepare the student to function socially with peers in the kindergarten environment. Option C is incorrect because such mastery is not required of students without disabilities until the end of kindergarten. Option D is incorrect because even though a visual schedule will help the student adjust to the new classroom routine, the preschool teacher is not required to provide one. This is the responsibility of the kindergarten teacher.

467
Q

Ms. Lewis, a kindergarten teacher, shows an unfamiliar storybook to a student. She asks the student to point to the title of the book and open the book to the first page of the story. After the teacher reads a few pages, she asks the student to point to the words. The teacher is most likely assessing the student’s awareness that

A. different conventions are associated with different genres of literature.

B. printed text corresponds to oral language.

C. letters correspond to individual speech sounds.

D. reading is an interactive process between the reader and the text.

A

Option B is correct because by pointing to the words on a page, the student associates printed words with oral language. Grasping this connection is important for language development. Option A is incorrect because pointing to the words in a text will not help the student become familiar with genre collection. Option C is incorrect because the student is not working on letter-sound correspondence. Option D is incorrect because an interactive activity during reading would involve the teacher asking questions and the student answering them.

468
Q

A special education teacher prepares a group of students to listen to a story about a whale by asking them what they know about whales and writing the students’ responses on the board. This activity is likely to facilitate comprehension of the story primarily because it will

A. prompt students to learn more about the topic.

B. encourage the students to use metacognitive strategies.

C. integrate oral and written vocabulary skills.

D. connect new information to the students’ background knowledge.

A

Option D is correct because the students’ background information about whales will help them acquire new information about the topic. Option A is incorrect because prompting the students to learn more about the topic does not require the teacher to check for prior knowledge. Option B is incorrect because metacognitive strategies require higher-order thinking, which will come later in the lesson. Option C is incorrect because integrating oral and written vocabulary skills is not an example of using prior knowledge to learn more about a new topic.

469
Q

Ms. Ortiz is a special educator who teaches in a middle school resource room. At the beginning of each school year, she has each student complete an interest inventory such as the one shown below.

What is your favorite animal?
What is your favorite game?
What is your favorite sport?
What do you like to do after school?
What is your favorite school subject?
What famous person would you most like to meet?
What place would you like to visit?

In planning reading instruction for students with disabilities, Ms. Ortiz could best use the information from this informal assessment to

A. determine students’ instructional reading levels.

B. suggest themes for units to the students’ English language arts teachers.

C. help students select books for independent reading.

D. determine gaps in students’ vocabulary and concept development.

A

Option C is correct because an inventory of the students’ interests will help the students select books for independent reading. Option A is incorrect because an inventory of the students’ interests will not help the teacher determine the instructional levels of the students. Option B is incorrect because the teacher does not need an inventory of the students’ interests in order to plan unit lessons for English language arts. Option D is incorrect because an inventory of the students’ interests will not pinpoint the deficiencies in their vocabulary and concept development.

470
Q

Which of the following activities, performed by young children while a story is being read to them, would most likely encourage them to listen for meaning?

A. Tapping their feet to the rhythm of the words

B. Miming the action of the story

C. Raising their hands when they hear the main character’s name

D. Making a clay sculpture of their favorite character

A

Option B is correct because students miming the action of the story while the teacher reads encourages them to reflect on a significant moment in the story and understand its meaning. Option A is incorrect because tapping feet while the story is being read will distract other students. Options C and D are incorrect because raising hands when the name of a character is spoken and making a clay sculpture of a character are not effective strategies to test for understanding of the meaning of the story.

471
Q

A first-grade teacher who is working with a group of beginning readers gives each student a set of word cards. On each card is printed a word that the students have already learned to read (e.g., “he,” “she,” “sees,” “loves,” “has,” “the,” “a,” “dog,” “cat,” and “pail”). The teacher shows the students how to arrange the cards to create a statement (e.g., “she sees the cat”). Students then create their own statements and read them aloud. One goal of this activity is to promote students’ reading development by reinforcing word recognition skills. In addition, this activity can be expected to promote students’ writing development by

A. helping them learn to view writing as a useful tool for communication.

B. promoting their recognition of similarities and differences between written and oral language.

C. building their understanding of basic syntactic structures.

D. helping develop their understanding of the value of writing conventions.

A

Option C is correct because an interactive writing activity will help the students understand how to create complete sentences using familiar words. Options A and B are incorrect because the teacher is presenting the basics of writing, not demonstrating different ways of communication. Option D is incorrect because the activity is focusing on basic syntactic structures, not writing conventions.

472
Q

A sixth-grade teacher has made the following notes about a student’s reading performance.

Rebecca’s oral reading speed and accuracy are about average for the class. Her errors, which tend to occur when she encounters unfamiliar polysyllabic words, usually consist of substituting real words or nonsense words that are structurally similar to the printed words rather than words that are semantically or syntactically correct.

Rebecca’s performance on oral and written comprehension questions that are based on silent and oral reading selections is also average for the class; however, her miscues are numerous and sometimes seem to interfere with her comprehension.

Based on the teacher’s notes about Rebecca’s reading performance, Rebecca would benefit most from instruction to help her

A. recognize high-frequency words with regular and irregular spellings.

B. use context clues and monitor her comprehension as she reads.

C. apply knowledge of phonics to decode unfamiliar words.

D. improve reading fluency and vocabulary skills.

A

Option B is correct because using context clues will help Rebecca improve her comprehension skills. Options A, C, and D are incorrect because decoding and learning new words out of context are not effective strategies for helping Rebecca improve her comprehension skills.

473
Q

A teacher regularly analyzes the attempted spellings of emergent readers. In addition to providing information about students’ spelling development, the teacher’s approach would best help the teacher assess students’

A. level of reading fluency.

B. use of word-identification strategies.

C. ability to apply phonics skills.

D. knowledge of comprehension strategies.

A

Option C is correct because analyzing the invented spelling of emergent readers will allow the teacher to assess the students’ understanding of phonemic awareness and to determine whether they are able to apply phonemic awareness skills to decode words. Options A, B, and D are incorrect because analyzing attempted spellings of emergent readers, will not enable the teacher to assess the students’ reading fluency and word identification skills or to check their reading comprehension levels.

474
Q

To promote students’ reading fluency, a fifth-grade teacher plans activities in which students and their assigned partners engage in repeated oral readings. When the teacher assembles the reading materials for the activities, the teacher should assign each pair of students passages from a text that

A. both students are capable of reading aloud with no more than 5 word-recognition errors per 100 words of text.

B. the students have previewed and selected themselves.

C. both students are capable of reading aloud with no more than 25 word-recognition errors per 100 words of text.

D. the students have been reading in connection with content-area study.

A

Option A is correct because 5 out of 100 word-recognition errors implies that the students are fluent readers and they can read independently. Choosing a book that both the students can read independently allows them to take turns reading and provide each other with feedback as a way to monitor comprehension. Option B is incorrect because for the students to succeed in reading, the teacher has to choose a book at the appropriate reading level of the students. Option C is incorrect because 25 out of 100 word-recognition errors implies that the text selected is too difficult and is not at the independent reading level of the students. Option D is incorrect because if the book is selected in connection to a content area, the reading level may not be at the independent reading level of the students.

475
Q

A middle school teacher plans the following activities in connection with a field trip to view a photojournalism exhibit at a local museum.

-Before the trip, students study how elements of design and photographic techniques express ideas and communicate meaning.
-The teacher gives students a list of questions to read and consider as they view the exhibit.
-After the trip, students work in small groups to write answers to the questions.
-In a whole-class discussion, students share their reactions to the exhibit and their groups’ answers to the questions.

Which of the following additional activities would best help the teacher informally assess students’ understanding of the way visual images and elements of design create meaning?

A. Students work in small groups to prepare their own exhibits by using photographs from newspapers and magazines to tell a story.

B. Each student researches one photojournalist featured at the exhibit and presents a brief report on the photojournalist’s work in the field.

C. Students write an essay about the photojournalism exhibit at the museum, analyzing a particular photograph they liked.

D. Each student writes a simulated magazine article and creates a drawing or illustration to accompany the article.

A

Option A is correct because letting the students work cooperatively in small groups to create a photo exhibit will provide them the opportunity to share ideas and allow the teacher to assess their understanding of the concept of photojournalism by telling a story through images. Options B, C, and D are incorrect because these tasks focus on independent research and writing, which is not the objective of the lesson.

476
Q

A reading teacher asks students to read a short passage from a text and then close their eyes for a minute to visualize what they just read. The activity will help the students to

A. understand the meaning of new vocabulary words.

B. increase phonemic awareness.

C. improve reading comprehension.

D. increase reading fluency.

A

Option C is correct because creating visual images increases students’ reading comprehension and provides an anchor to the text. Option A is incorrect because there is no evidence that reading short passages increases understanding of vocabulary within the passage. Option B is incorrect because visualization of a text does not increase phonemic awareness. Option D is incorrect because visualization of a text does not help in reading fluency.

477
Q

A survey has found that about 12 of every 500 airline flights are canceled due to bad weather. This ratio is equivalent to which of the following?

A. 1/24 of the flights

B. 1/240 of the flights

C. 0.24% of the flights

D. 2.4% of the flights

A

Option D is correct because 12 of 500 is equivalent to 2.4%. Options A and B are incorrect because they do not use the correct formula for finding the percentage. Option C is incorrect because the wrong fraction is used to compute the wrong percent

478
Q

A special education teacher provides math instruction in the resource room for individuals and small groups of students who have learning disabilities. When teaching the students, the teacher should follow which of the following instructional guidelines?

A. Emphasize the use of mathematics in science and technology.

B. Focus primarily on mathematical concepts that are likely to be useful in everyday life.

C. Emphasize the role of deductive reasoning in mathematics.

D. Connect mathematical concepts to concrete examples with which the students are already familiar.

A

Option D is correct because concrete examples using manipulatives help students with disabilities understand the abstract symbolic language of mathematics. Option A is incorrect because the concept of mathematics in science and technology is highly abstract for students with disabilities. Option B is incorrect because functional mathematics, such as telling time and counting money, are mostly used with students with mild intellectual disabilities or students who are preparing for vocational training. Option C is incorrect because the role of deductive reasoning in mathematics is an abstract concept that will be confusing for students with disabilities.

479
Q

A third grader with special needs has been having trouble memorizing basic arithmetic facts. The student’s classroom teacher asks the special education teacher’s advice on how to help the student. In general, which of the following suggestions is most important for the special education teacher to give the teacher?

A. Suspend for the present all mathematical activities other than those aimed directly at memorizing arithmetic facts.

B. Modify math instruction to focus primarily on the use of manipulatives to build conceptual understanding.

C. Address the problem now so that lack of automaticity will not impede more advanced math learning later on.

D. Discontinue memorization of facts in favor of teaching the student to use a calculator efficiently and accurately.

A

Option C is correct because the teacher should introduce a variety of strategies such as flash cards and math games to suit the student’s learning style and provide math fact sheets as an accommodation to help the student solve more advanced math problems. Option A is incorrect because the student needs to learn all the math concepts in the curriculum and not just focus on memorization of facts. Option B is incorrect because using manipulatives to build conceptual understanding does not address the issue of the student’s inability to learn facts. Option D is incorrect because the use of calculators will simply make the student dependent on the tool and not encourage the student to use logical reasoning.

480
Q

In general, math instruction for students with learning disabilities in mathematics should be characterized by

A. the development of cognitive strategies rather than a mastery of facts.

B. the development of math process skills.

C. instructional activities limited to hands-on operations with concrete objects.

D. a balanced program of concepts, skills and problem solving.

A

Option D is correct because the general education curriculum consists of a balanced program of concepts, skills, and problem solving, so students with learning disabilities should have access to the same curriculum as their nondisabled peers. Option A is incorrect because mathematics instruction should focus on both the concepts of cognitive strategies and mastery of facts. Option B is incorrect because mathematics instruction should focus on concepts, process skills, and problem solving. Option C is incorrect because even though hands-on operations with concrete objects are important, math instruction cannot be limited to just these activities.

481
Q

Joyce and Tyrone, two third graders, are counting how many gallon and quart milk containers have been collected for a class science project. As they count, Tyrone tells Joyce that the gallon container is twice as big as the quart container. Based on his comment, Tyrone needs instruction in understanding the concept of capacity. The teacher can best teach him by

A. asking him if he can think of some method of measurement that will test his statement.

B. pointing out to him that the word “quart” sounds like the word “quarter” and asking him what that suggests about the relative sizes of the two containers.

C. explaining to him that doubling the linear dimensions of a rectangular solid quadruples the volume.

D. having him use a computer to generate two- and three-dimensional models of the containers in question and then identify similarities and differences among the models generated.

A

Option A is correct because letting the student measure liquid using various methods will allow the student to learn the concept of measurement through hands-on experiments. Option B is incorrect because the best way to teach the concept of volume and capacity in mathematics is through hands-on experimentation and not by guessing. Option C is incorrect because the fundamental rule of connecting a rectangular volume with its linear dimension is a challenging concept for third graders. Option D is incorrect because using a computer to generate two- or three-dimensional models is an advanced skill that is not introduced in third grade.

482
Q

Which of the following is the most appropriate reason to use curriculum-based assessments?

A. To confirm the students’ grade level

B. To identify skills that need reteaching

C. To compare with last year’s test scores

D. To determine the students’ intellectual ability

A

Option B is correct because a curriculum-based assessment is a direct assessment that links instruction with assessment. It is used to help teachers develop goals for instruction and to evaluate the students’ progress in the curriculum. Option A is incorrect because placement tests are conducted to assess the students’ grade level. Option C is incorrect because standardized assessments help with the comparison of students’ scores in different years. Option D is incorrect because a diagnostic assessment is used to determine the intellectual ability of the student.

483
Q

Which of the following is the best activity for reviewing the concept of fractions with fourth-grade students?

A. Using pictures to model unit fractions of a whole

B. Drawing pictures of fractions from memory

C. Writing fractions using numbers

D. Comparing fractions using a variety of materials

A

Option D is correct because using a variety of concrete materials to reinforce the concept of fractions helps students develop a better understanding of the concept. Option A is incorrect because simply using pictures to model unit fractions does not help students learn the concept thoroughly. Option B is incorrect because drawing pictures from memory is a skill requiring knowledge of an abstract concept and does not help students review fractions. Option C is incorrect because writing fractions using numbers is an important skill but does not help to reinforce the concept of fractions.

484
Q

Which of the following is a developmentally appropriate activity for a kindergartner to establish basic number sense?

A. Completing one-to-one correspondence exercise

B. Generalizing skills and applying them to new situations

C. Arranging ten objects from smallest to largest

D. Using a calculator to solve simple mathematics problems

A

Option A is correct because by the age of five, children are able to link the number of objects to the numeral. Option B is incorrect because generalizing skills and applying them to a new situation is a highly advanced mathematical concept and is not age appropriate for kindergarteners. Option C is incorrect because arranging objects from smallest to largest is a skill for younger children who are in preschool. Option D is incorrect because using a calculator to solve math problems is not age appropriate for a kindergarten class.

485
Q

Which of the following is most commonly found on students’ Individualized Education Programs (IEPs)?

A. Speech or language impairment

B. Emotional disturbance

C. Specific learning disability

D. Intellectual disability

A

Option C is correct because according to educational research statistics, children with specific learning disabilities are the highest in number compared with the other disability categories. Options A, B and D are incorrect because each of these disability categories has a much smaller percentage of students compared with students with learning disabilities.

486
Q

In which of the following ways has the movement toward accountability in education most significantly affected students who receive special education services?

A. Special education students must now achieve Individualized Education Program (IEP) goals before graduation.

B. Special education students’ grades must reflect the standards applied to general education students.

C. Fewer special education students pass end-of-course assessments.

D. More special education students are participating in statewide testing.

A

Option D is correct because Individuals with Disabilities Education Act (IDEA) mandates that students with disabilities to participate in statewide assessments with their nondisabled peers, and districts are required to develop alternate assessments for students who cannot participate in regular state or district-wide assessments. Districts must also provide accommodations in both instruction and assessment as needed by students with disabilities. Option A is incorrect because special education students do not need to achieve their IEP goals before graduation. Rather, they must participate in statewide testing. Option B is incorrect because students with disabilities receive accommodations and modifications based on their IEPs, so their grades do not reflect the standards applied to general education students. Option C is incorrect because students with disabilities must participate in statewide testing regardless whether they pass end-of-course assessments.

487
Q

Ms. Fredericks, who is teaching first grade for the first time, asks Mr. Cardenas, the school’s special education teacher, why a new student in her class needs a full and individual evaluation (FIE) even though a doctor has provided a note indicating that the student has been diagnosed with diabetes. Which of the following is the most appropriate response from Mr. Cardenas?

A. New students may not be identified for special education without going through the Response to Intervention (RTI) process.

B. Students with diabetes do not fall under a disability category covered by the Individuals with Disabilities Education Act (IDEA).

C. Public schools do not have to consider evaluation data from personnel not associated with educational institutions.

D. Formal evaluations by law must determine whether the student has a need for special education and related services.

A

Option D is correct because determining educational need is part of the process of identifying students who are eligible for services. Option A is incorrect because RTI is not a required process for identification of a student with a disability under IDEA. Option B is incorrect because diabetes could be a condition requiring services under the Other Health Impairment (OHI) category. Option C is incorrect because schools must consider all relevant evaluation data provided to them.

488
Q

Which of the following is most effective for a teacher to use when working with linguistically diverse students with disabilities in second grade?

A. Listening to class discussions and asking questions

B. Presenting vocabulary that is needed in the learning context

C. Having the students dictate their responses to test questions

D. Introducing mnemonic devices to improve memorization

A

Option B is correct because presenting the key vocabulary of the lesson to the students makes it easier for those who speak other languages to focus on the main idea of the lesson. This strategy will also ease any anxiety the students feel. Options A, C, and D are incorrect because listening to class discussions, having students dictate responses to test questions, and introducing mnemonic devices will not make the lesson more accessible to students.

489
Q

A beginning special educator teaches middle school students from diverse backgrounds in a resource room. At the beginning of the school year, the teacher reviews a list of rules for the resource room with each group of students and hangs the list on the classroom wall. One rule gives directions about the appropriate noise level: “Work quietly without disturbing others.”

Several times during the first weeks of school, students start helping each other with their work instead of focusing on their own. Consequently, the teacher has given out warnings to these students for breaking the rule. When reflecting upon the students’ behavior, the teacher needs to be aware that

A. students from some cultures embrace a belief of interdependence rather than independence.

B. rules should be consistently enforced to create a secure environment for students.

C. students are more likely to follow rules that are accompanied by some form of reinforcement.

D. resource rooms typically do not require the same degree of structure that general classrooms do.

A

Option A is correct because some students may come from families of other cultures that value interdependence or have a group identity rather than the individualistic identity that is more prominent in the United States. Options B, C and D are incorrect because these considerations do not address the difference in value systems between cultures.

490
Q

After an Admission Review Dismissal (ARD) committee meeting for a student with an intellectual disability, Ms. Salina, the special education teacher, is contacted by Mr. Davis, a case worker from a local state agency that provides services for individuals with intellectual disabilities. He reports to Ms. Salinas that the student’s parents want him to review the Individualized Education Program (IEP) developed at the ARD meeting, and he wants to come to the school to look at the document. Which of the following is the most appropriate response to Mr. Davis’s request?

A. Teachers can only provide copies of the IEP document to members of the ARD committee.

B. Outside agencies involved in providing services can review educational documents only at the school campus.

C. Community agency case workers can review the IEP document only with written permission from the parent.

D. School district personnel can only provide written summaries of ARD committee meetings.

A

Option C is correct because educational records can be released with the written permission of the parent or adult student. Option A is incorrect because the IEP document can be shared outside of the meeting with written parent permission. Option B is incorrect because the document can be shared off campus with parent permission. Option D is incorrect because summaries of the ARD would still be considered educational records requiring parental permission for release.

491
Q

During mathematics lessons, a special educator has students work in groups of two or three. Which of the following strategies would be most effective for the educator to use to help the small groups of students learn the social skill of sharing?

A. Giving the groups a time limit to solve a set of problems

B. Reviewing the answers to all of the problems at the end of class

C. Having the students work with the same group for several days

D. Providing each group with only one set of materials

A

Option D is correct because providing limited materials necessitates sharing during a cooperative learning activity. Option A is incorrect because setting a time limit will not teach the students social skills. Option B is incorrect because reviewing the answers to all the problems at the end of class does not allow the students to practice their sharing skills. Option C is incorrect because having the students work in the same group will not necessarily teach sharing as a social skill.

492
Q

Mr. McClure, a special education paraprofessional assigned to an elementary resource room, has a tendency to come in late to class. His supervisor, Mr. Washington, would like to discuss the issue with him in a way that would elicit the paraprofessional’s genuine cooperation. Which of the following types of statements would be most effective for Mr. Washington to use to meet his goal?

A. A reminder about Mr. McClure’s responsibilities as they are stated in his contract

B. A description of what the supervisor observes Mr. McClure doing and the effect his actions have on the class

C. A summary highlighting the supervisor’s impressions of Mr. McClure’s need for change in behavior

D. A promise that if Mr. McClure is punctual all week, the supervisor will let him leave early on Fridays

A

Option B is correct because Mr. McClure arriving late every day to the classroom disrupts the routine, and many students with disabilities need a stable learning environment in order to succeed. Option A is incorrect because the paraprofessional is already aware of the responsibilities stated in the contract. Options C and D are incorrect because sharing impressions and promising to let Mr. McClure leave early every Friday are both unprofessional and unrealistic.

493
Q

According to the Individuals with Disabilities Education Act (IDEA), which of the following people must be present at a third-grade student’s annual Admission Review Dismissal (ARD) committee meeting?

A. A special education teacher, a general education teacher, a parent, and a school administrator

B. A special education teacher, a general education teacher, a parent, and an occupational therapist

C. A general education teacher, a physical therapist, a school administrator, and a guidance counselor

D. A special education teacher, a general education teacher, a parent, the student, and an interpreter

A

Option A is correct because a special education teacher, a general education teacher, a parent, and a school administrator are mandated to participate in the student’s ARD committee meeting. Option B is incorrect because the student may or may not receive occupational therapy, and the school administrator is excluded. Option C is incorrect because the student may or may not receive physical therapy or have a guidance counselor, and the school administrator is excluded. Option D is incorrect because a school administrator is excluded. In addition, IDEA does not require third-grade students to attend the ARD committee meeting, and the scenario given does not require an interpreter.

494
Q

A new high school student is being assessed. He reads a 500 word text. He misreads 35 words. At what level is he reading?
a. Instructional level.
b. Independent level.
c. Unsatisfactory level.
d. Merit level

A

A: Instructional level. In one minute, a student who reads with 95-100% accuracy is at an independent reading level. A student who reads with 90-95% accuracy is at an Instructional level. A student who reads with less than 90%accuracy is at a frustration level.

495
Q

According to the Assistive Technology Act, assistive devices are:
a. Electronic devices that support learning such as computers, calculators, student responders, electronic self-teaching books and electronic reading devices.
b. Any mechanical, electrical or electronic device that helps teachers streamline efficiency.
c. Any device that could help a disabled student in school or life functions.
d. Experimental, high-tech teaching tools that teachers can obtain by participating in one of 67 government funded research projects.

A

C: Any device that could help a disabled student in education or life functioning. The Assistive Technology Act of 1998 is the primary legislation regarding assistive technology for disabled students and adults. The act funds 56 state programs concerned with the assistive technology needs of individuals with disabilities. Assistive devices include wheelchairs, hearing aids, glare-reduction screens, Braille devices, voice-recognition software, screen magnifiers and a wealth of other tools.

496
Q

In the first week of school, a resource teacher asks her high school students to make lists of things they know how to do well. How is this activity most useful to the teacher?
a. It establishes a feeling of success in her students from the start.
b. It is an informal assessment of their writing skills and gives the teacher an idea of each student’s interests and abilities.
c. It is a formal assessment of prior knowledge.
d. It invites further discussion of each student’s unique contributions and will help the class bond with mutual respect.

A

B: It is an informal assessment of their writing and also gives the teacher an idea of each student’s interests, abilities and skills. This assignment gives the teacher an idea of her students’ writing abilities at the beginning of the year. She can return to this piece of writing during the school year to assess progress.

497
Q

In the above example, how could the teacher use the students’ lists in her lesson planning?
a. On the last day of school, she can return the lists and ask the students to add the new skills they’ve learned, so they can see how far they’ve come.
b. She can have the students exchange lists so they can find other students who share the same interests.
c. She can use the lists when planning independent reading and research projects for each student.
d. All of the above.

A

D: All of the above. The writing prompt is multipurpose. The teacher can use it in a number of ways, including planning independent reading and research projects for each student, inviting students to share their writing to find others with the same interests and as a way of demonstrating to each student their academic growth at the end of the school year.

498
Q

A seventh grader with mild intellectual disabilities is having considerable trouble with algebra. His stepfather is trying to help, but the more he drills the boy, the less he seems to understand. The teacher suggests:
a. He continues drilling and enhances with pop quizzes. It may take the student longer to understand algebraic terms, expressions and equations, but with hard work he will eventually learn them.
b. He calls a moratorium on at-home algebra work. The student is becoming less willing to work at school and the teacher is concerned he is losing confidence due to failure at home.
c. He continues drilling but breaks the study sessions into no more than 3 five-minute periods per day.
d. He substitutes fun activities for math drills. Incorporating algebra blocks, math games, and applications of algebra to real-life situations will make math more fun and more relevant.

A

D: He substitutes more enjoyable algebra activities for math drills. Incorporating manipulatives such as algebra blocks, math games and applications of algebra to real-life situations, will make math both more fun and more relevant. When both parent and child are enjoying the work, they will accomplish morein a shorter period of time and the child will feel happy and successful, which encourages her to embrace further learning opportunities.

499
Q

Augmentative and Alternative Communication (AAC) devices, forearm crutches and a head pointer are assistive devices that might be used by a student with:
a. Severe intellectual disabilities.
b. Cerebral palsy.
c. Tourette syndrome.
d. Minor skeletal birth defects.

A

B: Cerebral palsy. Cerebral palsy is an umbrella term that groups neurological childhood disorders that affect muscular control. It does not worsen over time and the cause is located in damaged areas of the brain that control muscle movement. Depending upon the severity of the disorder, a child with cerebral palsy might benefit from an AAC device to help in speaking, forearm crutches to assist in walking or a head pointer for a child whose best motor control is his head.

500
Q

A four year old child has difficulty sorting plastic cubes, circles and triangles by color and shape, doesn’t recognize patterns or groups and doesn’t understand the relationship between little/big, tall/short, many/few. The child enjoys counting, but does not say the numbers in proper order nor recognize the meaning of different numbers. This child most likely:
a. Is exhibiting signs of intellectual disabilities.
b. Is developing within an acceptable range.
c. Has dysgraphia.
d. Has dyscalculia.

A

D: Dyscalculia. Dyscalculia defines a range of difficulties in math, such as the inability to understand numbers’ meanings, measurements, patterns, mathematical terms and the application of mathematic principals. Early clues include a young child’s inability to group items by size or color, recognize patterns or understand the meaning or order of numbers.

501
Q

Response to Intervention (RTI) is:
a. Parents, classroom teacher, special education teacher and other caring persons stage an intervention to express how a student’s socially unacceptable behavior upsets them.
b. An opportunity for a student to openly and freely respond to specific interventions without fear of reprimand.
c. A strategy for diagnosing learning disabilities in which a student receives research-supported interventions to correct an academic delay. If the interventions do not result in considerable improvement, the failure to respond suggests causal learning disabilities.
d. A formal complaint lodged by a parent or guardian in response to what they consider an intrusion by a teacher into private matters.

A

C: A strategy for diagnosing learning disabilities in which a student with an academic delay receives research-supported interventions to correct the delay. If the interventions do not result in considerable academic improvement, the failure to respond suggests causal learning disabilities.

502
Q

Sixth graders Alfie and Honesty ride the same bus. Honesty constantly teases Alfie. Alfie is embarrassed because he believes she is berating him. The bus driver told their teacher it was possible that Honesty is actually interested in Alfie, but doesn’t express it well. The best form of conflict resolution would be for the teacher to:

a. Take Honesty aside and explain boys don’t like overly aggressive girls.

b. Take Honesty aside and teach her less embarrassing methods of getting a boy’s attention.

c. Explain to Alfie that Honesty probably teases him because she likes him and he should take it as a compliment.

d. Suggest to Alfie that if he is disturbed by Honesty’s teasing, he have a calm, assertive conversation with her and tell her he doesn’t like it and insist she stop.

A

D: Suggest to Alfie that if he is disturbed by Honesty’s teasing, he might have a calm, assertive conversation with her in which he tells her he doesn’t like it and insist she stop. By encouraging Alfie to act on his own, it shows him he has primary responsibility for taking care of himself. By offering social strategies, he learns a set of skills that will serve him throughout life. If Honesty continues to tease him, he can ask a teacher to step in, but doing so without his invitation is inappropriate.

503
Q

A special education teacher is creating a developmental history for a high school student. She wants to know when the teen reached certain behavioral, academic and developmental milestones. She should consult:
a. The student’s previous teachers. This information should be in the file.
b. The student. Involving him in the process will make him more interested in his progress.
c. The student’s doctor and therapist. These professionals know how to elicit and document this information.
d. The parent or guardian because he or she has known the student from the beginning.

A

D: The student’s parent or guardian, who has known the student throughout his life, is the correct answer. When compiling a developmental history it’s best to consult people who have had a close personal relationship with the student over his lifetime. They are the most likely to possess the greatest amount of information regarding the student’s development over time.

504
Q

When transitioning from one subject to another and when she becomes anxious, a student always taps her front tooth 5 times then opens and closes her eyes 11 times before leaving her desk. The child most likely has:
a. Repetitive Disorder
b. Obsessive Compulsive Disorder
c. Anxiety Disorder
d. Depression

A

B: Obsessive Compulsive Disorder (OCD). Children and adults with OCD typically engage in a series of highly ritualized behaviors that are rigidly performed when they feel stressed. Behaviors include tapping, snapping fingers, blinking, counting and so forth.

505
Q

By law, a child with a disability is defined as one with:
a. Intellectual disabilities, hearing, speech, language, visual, orthopedic or other health impairments, emotional disturbance, autism, brain injury caused by trauma or specific learning disabilities and needs special education and related services.
b. Intellectual disabilities, emotional disturbance, autism, brain injury caused by trauma or specific learning disabilities who needs special education and related services.
c. A child who is unable to reach the same academic goals as his peers, regardless of cause, and needs special education and related services.
d. The term “disability” is no longer used. The correct term is “other ability”.

A

A: Intellectual disabilities, hearing, speech, language, visual, orthopedic or other health impairments, emotional disturbance, autism, brain injury caused by trauma, or specific learning disabilities who needs special education and related services. Children with one or more of these conditions are legally entitled to services and programs designed to help them achieve at the highest level of their ability.

506
Q

Which classroom environment is most likely to support a student with ADHD?
a. Students with ADHD become bored easily so a classroom with distinct areas for a multitude of activities will stimulate her. When she loses interest in one area, she can move to the next and continue learning.
b. Students with ADHD are highly aggressive and easily fall into depression. The teacher needs to provide a learning environment in which sharp objects such as scissors, tacks or sharpened pencils are eliminated. This ensures greater safety for both student and teacher.
c. Students with ADHD are highly creative. A room with brightly colored mobiles, a multitude of visual and physical textures (such as striped rugs and fuzzy pillows) and plenty of art-based games will stimulate and encourage learning.
d. Students with ADHD are extremely sensitive to distractions. A learning environment in which visual and audio distractions have been eliminated is best. Low lighting, few posters and a clean whiteboard help the student focus.

A

D: Students with ADHD are extremely sensitive to distractions. A learning environment in which visual and audio distractions have been eliminated is best. Low lighting, few posters and a clean whiteboard will help minimize distractions.

507
Q

A resource teacher notices one of her students has made the same reading error numerous times the past few days. She decides the student wrongly believes that ‘ou’ is always pronounced as it is in the word through.She corrects this misunderstanding by showing the student word families containing words like though, ought, ground. This strategy is called:
a. Corrective feedback
b. Positive reinforcement
c. Consistent repetition
d. Corrective support

A

A: Corrective feedback. Corrective feedback is offered to a student in order to explain why a particular error is, in fact, an error. Corrective feedback is specific; it locates where and how the student went astray so that similar errors can be avoided in the future.

508
Q

A kindergarten teacher has a new student who will not make eye contact with anyone so she doesn’t appear to be listening. She often rocks back and forth and does not stop when asked or give any indication she has heard. She avoids physical contact. Sometimes the teacher must take her arm to guide her from one place to another. Occasionally the student erupts, howling in terror and fury. The most likely diagnosis is:
a. Asperger’s Syndrome
b. Obsessive-Compulsive Disorder
c. Autism
d. Antisocial Psychosis

A

C: Autism. Autistic children are typically very withdrawn, avoid eye contact and are not responsive to verbal or physical attempts to connect. Some autistic children fall into repetitive behaviors that are very difficult to arrest or prevent. These behaviors include rocking, spinning and handshaking.

509
Q

A special education teacher shows parents of a dyslexic child a study that examined brain scans of dyslexic and non-dyslexic readers. The study demonstrated that dyslexics use (the) __________ side(s) of their brains while non-dyslexics use (the) _______ side.
a. Both, the left.
b. Both, the right.
c. Left, right.
d. Right, left.

A

A: Both, the left. Research using MRIs show dyslexics use both sides of their brains for activities such as reading, while non-dyslexics use only the left side.

510
Q

A student with ______________ has a great deal of difficulty with the mechanical act of writing. She drops her pencil, cannot form legible letters and cannot decode what she has written.
a. A nonverbal learning disorder
b. Dyslexia
c. Dyspraxia
d. Dysgraphia

A

D: Dysgraphia. Dysgraphic individuals cannot manage the physical act of writing. While many dysgraphics are highly intelligent and able to express themselves cogently, they have extreme difficulty holding a writing implement and shaping letters.

511
Q

A resource room teacher has a middle school student recently diagnosed with depression. The student has been put on an antidepressant. The teacher knows the student may develop certain transitory reactions to the medication. One reaction might be:
a. Extreme sleepiness.
b. Increased, persistent thirst.
c. Anxiety, coupled with an urge to verbalize a continuous inner dialogue.
d. Inappropriate anger.

A

B: Extreme, persistent thirst. Although there are a number of antidepressants available, most of them share the side effect of a dry, cottony mouth that lasts for a few weeks at the beginning. The student is likely to ask for water frequently because this type of thirst isn’t easily quenched. The teacher and the student should understand this side effect will ease and disappear with time.

512
Q

Reading comprehension should be evaluated:
a. Every two months using various informal assessments. Done more than twice a year, assessments place undue stress on both student and teacher and do not indicate enough change to be worth it.
b. With a combination of informal and formal assessments including: standardized testing, awareness of grades, systematically charted data over a period of time and teacher notes.
c. With bi-weekly self-assessment rubrics to keep the student aware of his progress.
d. By testing the student before reading a particular text to determine which vocabulary words he already knows and can correctly use.

A

B: With a combination of informal and formal assessments including standardized testing, awareness of grades, systematically charted data over a period of time and teacher notes. Comprehension and vocabulary cannot be sufficiently assessed with occasional, brief studies. Continuous observation, high-stakes and standardized testing, attention to grades and closely tracking the outcomes of objective-linked assessments are interrelated tools that, when systematically organized, offer a solid understanding of students’ strengths and weaknesses.

513
Q

A diabetic first grader is very pale, trembling and covered in a fine sweat. The teacher attempts to talk to the child, but the girl’s response is confused and she seems highly irritable She is most likely experiencing:
a. Diabetic hypoglycemia.
b. Lack of sleep.
c. Hunger.
d. Diabetic hyperglycemia

A

A: Diabetic hypoglycemia. Diabetic hypoglycemia, also known as insulin reaction, occurs when blood sugar falls to a very low level. It is important to treat it quickly or the diabetic could faint, in which case an injection of glucagon is administered.

514
Q

An intellectually disabled teen has been offered a job by an elderly neighbor. The neighbor wants the teen to work alongside her in the garden twice a week. They will plant seeds, transplant larger plants, weed, lay mulch, water and fertilize. Later in the season, they will cut flowers and arrange bouquets, pick produce and sell them at the neighbor’s roadside stand. The neighbor, the teen’s mother and special education teacher meet to discuss the proposal. The plan is:
a. Tentatively accepted. Because the teen is excited about having a job, her mother and teacher reluctantly agree. They both know the girl is likely to lose interest quickly and caution the neighbor that if she truly needs help she may want to look elsewhere. However, no one wants to disappoint the girl and all decide the experience will be good for her.
b. Rejected. Despite the teen’s insistence that she can manage these tasks, her mother and teacher believe that she cannot. They fear trying will set her up for failure.
c. Tentatively rejected. The teacher and her mother are very uncomfortable with the neighbor’s offer. They suspect the elderly woman is simply lonely or may be a predator who has selected an intellectually disabled victim because such children are particularly vulnerable.
d. Enthusiastically accepted. The adults discuss a background check and the possibility that the teen might discover gardening is not for her and want to quit. However, this is most likely to happen early in her employment, giving the neighbor sufficient time to find another helper. The teacher is pleased because the girl will learn new skills through modeling and repetition. The mother is pleased because the experience will add to the girl’s self-esteem and show her she is capable of learning. The elderly neighbor is pleased because she is compassionate and truly needs help. The girl is delighted the neighbor recognizes her potential and sees her as valuable.

A

D: Enthusiastically accepted. The adults discuss a background check and the possibility the teen might discover gardening is not for her and want to quit. However, this is most likely to occur early in her employment, giving the neighbor sufficient time to find another helper. The teacher is pleased because the girl will learn new skills through modeling and repetition. The mother is pleased because the experience will add to the girl’s self-esteem as well as show her she is capable of learning. The elderly neighbor is pleased because she is both compassionate and truly needs help. The girl is delighted the neighbor recognizes her potential and sees her as valuable.

515
Q

Dr. Gee reads the following sentence to a group of 5th graders: “The turquoise sky is reflected in the still lake. Fat white clouds floated on the lake’s surface as though the water was really another sky. It was such a beautiful day. The students were to write the word “beautiful” in the blank. One student wrote ‘pretty’ instead. This suggests:
a. The student doesn’t know the meaning of the word ‘beautiful’.
b. The student is highly creative and believes he can substitute a word with a similar meaning.
c. The student did not know how to spell ‘beautiful’.
d. The student did not hear what the teacher said. He heard ‘pretty’ instead of ‘beautiful.’

A

C: The student did not know how to spell ‘beautiful’. It is doubtful the student heard “pretty” instead of beautiful since the two sound nothing alike. It is equally unlikely he doesn’t know the meaning of the word ‘beautiful’ since his substitution, ‘pretty’, is a synonym for beautiful. It is likely this child is creative, but that alone wouldn’t be sufficient reason to replace one word with another. The most logical answer is that he simply didn’t know how to spell ‘beautiful’. He does know that some words mean almost the same thing, and since he already knew how to spell ‘pretty’, he incorrectly believed a synonym would be acceptable.

516
Q

Autism Spectrum Disorder is also known as:
a. Pervasive Spectrum Disorder
b. Asperger’s Syndrome
c. Variable Developmental Disorder
d. Artistic Continuum Syndrome

A

A: Pervasive Spectrum Disorders (PSD) is another name for Autism Spectrum Disorders (ASD). PSD causes disabilities in language, thought, emotion and empathy. The most severe form of PSD is autistic disorder. A much less severe form is Asperger’s Syndrome.

517
Q

A third grade boy is new to the school. His teacher has noticed he happily plays with other children, redirects his attention without upset when another child rejects his offer to play and doesn’t mind playing on his own. However, the boy doesn’t pay attention when academic instruction is given. He continues to speak with other children, draws, or distracts himself. The teacher reminds him repeatedly to listen and follow instruction. When he does not, she moves him to a quiet desk away from the others. When isolated, the boy puts his head on the desk and weeps uncontrollably, or stares at a fixed spot and repeats to himself, “I hate myself, I hate myself. I should be dead.” During these episodes, the teacher cannot break through to the student; his disconnection seems complete. The teacher has requested a conference with his parents, but they do not speak English and have not responded to her offer of a translator. The teacher should:
a. Establish a consistent set of expectations for the child. He needs to understand there are appropriate times for play and for learning.
b. Isolate the boy first thing. His behavior suggests manipulation. By third grade children fully understand they are expected to pay attention when the teacher is speaking. The boy is punishing the teacher with tears and repetitive self-hate, consciously or unconsciously attempting to make the teacher feel guilty.
c. Immediately refer him to the counselor. The boy is exhibiting serious emotional distress suggesting abuse or neglect at home or outside of school.
d. Recognize the child’s highly sensitive nature; offer comfort when he acts out self-loathing. Carefully explain why he must learn to pay attention so he will use reason instead of emotion when making future choices.

A

C: Immediately refer him to the counselor. The boy is exhibiting serious emotional distress suggesting either abuse or neglect at home or elsewhere. While his behavior may seem manipulative, the fact that the boy is unreachable once he’s in the highly charged emotional state in which he repeats, “I hate myself” suggests emotional trauma. The fact the child is socialized with peers, playing with them when invited and not taking rejection personally, suggests his emotional distress may be caused by an adult who has convinced him he is unworthy. A trained counselor is the best choice.

518
Q

A student with Asperger’s Syndrome is most likely to display which set of behaviors?
a. He is confrontational, argumentative and inflexible.
b. He is fearful, shy and highly anxious.
c. He is socially distant, focused on certain subjects to the point of obsession and inflexible.
d. He is flighty, tearful and exhibits repetitive, ritualized behavior.

A

C: He is socially distant, focused on certain subjects to the point of obsession and inflexible. Asperger Syndrome is a mild form of autism. Children with this disorder typically do interact with teachers, other adults and sometimes other children; however, the interaction is rather remote and without emotional expression. They are also very focused on subjects of great interest to the abandonment of all others. When asked to redirect focus, Asperger children often become emphatically obstinate, refusing to shift focus.

519
Q

A special education teacher working with a group of third graders is about to begin a unit on birds. She asks the children what they know about birds. They tell her birds fly, lay eggs and build nests. She asks the students to draw a picture of a bird family. Some children draw birds in flight; one draws a mother bird with a nest of babies; another draws an egg with the baby bird inside the egg. These pre-reading activities are useful because:
a. They help assess prior knowledge.
b. They establish a framework in which to integrate the new information.
c. They create a sense of excitement and curiosity.
d. All of the above.

A

D: All of the above is correct. This project gives the teacher the opportunity to evaluate what students already know, establishes a scaffold of accessible information to which the students can integrate new information and creates a sense of curiosity and excitement in the students, which encourages them to learn.

520
Q

Verbal dyspraxia is:
a. Trouble with the physical act of writing.
b. Refusal to speak.
c. Misplacing letters within words.
d. A motor skill development disorder which includes inconsistent speech errors.

A

D: is correct: A motor skill development disorder which includes speech errors that don’t clearly follow a pattern and so appear to be inconsistent. An example is a student who can pronounce /p/ when it is followed by a long i, as in pine, but not when followed by an ou diphthong, as in pout. Verbally dyspraxic individuals are unable to correctly place the tongue, lips and jaw for consistent sounds that can be organized into syllables. Dyspraxia appears to be a brain disorder in which the area that controls production of particular sounds is damaged.

521
Q

A resource room teacher has a small group of second and third graders who are struggling with reading comprehension. A useful strategy would be to:
a. Present a list of vocabulary before students read a particular text.
b. Ask students to create a play about the story.
c. Read a story aloud. Ask students to raise their hands when they hear an unfamiliar word.
d. Have each child keep a book of new vocabulary words. Whenever an unfamiliar word is seen or heard the student should enter the word in her personal dictionary.

A

B: Ask students to create a play about the story as the teacher reads aloud. This activity grounds the students in thestory action as it is occurring. Acting it out insures understanding; otherwise, the students will most likely stop the teacher and ask for clarification. Furthermore, by acting it out, students are incorporating understanding physically. They will be more likely to retain the story and be able to comprehend the meanings incorporated in it.

522
Q

Tourette syndrome is characterized by:
a. Facial twitches, grunts, inappropriate words and body spasms.
b. Inappropriate words, aggressive behavior and tearful episodes.
c. Facial twitches, grunts, extreme shyness and refusal to make eye contact.
d. Refusal to make eye contact, rocking, spinning of objects and ritualized behavior.

A

A: Twitches, grunts, inappropriate words, body spasms. Children and adults with Tourette syndrome are rarely aggressive nor are they reluctant to make eye contact or otherwise engage others. Tourette syndrome is characterized by explosive sounds, sometimes in the form of inappropriate words, more often just as meaningless syllables; muscular twitches of the face or elsewhere in the body and the complete inability to control these spasms. Tourette sufferers often also suffer from Obsessive Compulsive Disorder.

523
Q

A second grader finds it impossible to remain in her seat. She wanders around the room, sprawls on the floor and rolls back and forth when asked to do math problems and jumps up and down when waiting in line. When the teacher tells her to sit down, she rolls her eyes in apparent disgust and looks to other students for support. When she finds a student looking back, she laughs and makes a face. The teacher has noticed when a reward is attached to good behavior; the girl is consistently able to control her actions for long periods of time. But when reprimanded without the promise of a reward, she becomes angry, tearful and pouts. This child is most likely manifesting:
a. Tourette’s Syndrome
b. Attention Deficit Hyperactivity Disorder
c. Lack of sufficiently developed behavior and social skills
d. Psychosis

A

C: Lack of sufficiently developed behavior and social skills. The child may or may not be hyperactive, but the fact that she can control her behavior for extended periods if a reward is involved suggests the child is overly indulged outside of class. In addition, she appears to act out in an effort to seek peer admiration; this excludes the possibility of Tourette syndrome and Attention Deficit Hyperactivity Disorder. In the first case, she would be unlikely to seek approval. In the second, she would be unlikely to be able to control herself under certain circumstances. There is nothing in her behavior to suggest psychosis.

524
Q

ADHD refers to:
a. Attention Deficit Hyperactivity Disorder
b. Anxiety/ Depression Hyperactivity Disorder
c. Aggression-Depression Hyperactivity Disorder
d. Atkinson, Draper and Hutchinson Disability

A

A: Attention Deficit Hyperactivity Disorder. Children with ADHD exhibit a myriad of symptoms including: disorganization, easily distracted and frustrated, defensive, immature, impulsive, often interrupts conversations and hyperactive behaviors.

525
Q

Rate, accuracy and prosody are elements of:
a. Reading fluency
b. Reading comprehension
c. Math fluency
d. Algebraic function

A

A: Reading fluency. Fluent readers are able to read smoothly and comfortably at a steady pace. The more quickly a child reads, the greater the chance of leaving out a word or substituting one word for another, i.e., wink instead of sink. Fluent readers are able to maintain accuracy without sacrificing rate. Fluent readers also stress important words in a text, group words into rhythmic phrases and read with intonation (prosody).

526
Q

When a diabetic student goes into insulin shock, she should:
a. Call her parents to come get her.
b. Drink a soda or eat some hard candy.
c. Drink a high-protein shake.
d. Put her head on the desk and wait for the episode to pass.

A

B: Drink a soda or eat some hard candy. Diabetes is a metabolic disorder that prevents proper processing of food, resulting in a lack of enough insulin for the blood to transport sugar. Insulin shock, also known as hypoglycemia, is typically brought on by a diabetic’s failure to take insulin or to eat often enough. It is a serious condition that must be dealt withimmediately.

527
Q

Strategies to increase reading fluency for English Language Learners include:
a. Tape-assisted reading.
b. Reading aloud while students follow along in their books.
c. Asking parents to read with the child each evening.
d. A and B.

A

D: A and B. Any opportunity for an ELL to hear spoken English while simultaneously seeing it in print will help facilitate reading fluency.

528
Q

How could a teacher effectively use a picture book of folk tales from an English Language Learner’s country of origin?
a. Share the book with other students to educate them about the ELL’s culture.
b. Ask the ELL to rewrite the folk tale, modernizing it and using the United States as the setting.
c. Ask the student to tell one folk tale in her native language. The teacher writes key English words and asks the child to find them in the book. When the child finds the words, they read them together.
d. None of the above. It’s best to encourage the student to forget about her first country as quickly as possible to help her acclimate. Folktales from her country will only make her homesick.

A

C: Asking the student to tell her one folk tale from the book in her native language. The teacher writes key English words and asks the child to find those words in the book. When the child finds the words, they read them together. The teacher might also suggest the child write the new English words in a notebook and extend the lesson by having thechild write sentences using the words.

529
Q

Asking the student to tell her one folk tale from the book in her native language. The teacher writes key English words and asks the child to find those words in the book. When the child finds the words, they read them together. The teacher might also suggest the child write the new English words in a notebook and extend the lesson by having thechild write sentences using the words.

What kind of load does the previous example employ?
a. Cognitive load
b. Language load
c. Bilingual load
d. Cultural load

A

D: Cultural load. Cultural load is concerned with how the relationship between language and culture can help or hinder learning. By using a familiar folk tale, the student is given the opportunity to learn new words in a culturally familiar context.

530
Q

A teacher asks an English Language Learner to do a picture walk through a book and describe what he thinks the story is about. This helps the student’s awareness of the story’s __________.
a. Deeper meaning
b. Theme
c. Context
d. Events

A

D: Events. A picture walk invites a reader to prepare for the act of reading the text by previewing the illustrations in order to understand the events that will unfold in the story.

531
Q

Language load is:
a. The weight of books in a child’s backpack or the damage to the child’s body the weight may cause in the future.
b. The vocabulary a child has upon entering a new classroom.
c. The number of unrecognizable words an English Language Learner encounters when reading a passage or listening to a teacher.
d. The number of languages a person has mastered.

A

C: The number of unrecognizable words an English Language Learner encounters when reading or listening. Language load is one of the barriers ELLs face. Rephrasing, dividing complex sentences into smaller units and teaching essential vocabulary before the student begins reading areall strategies which can lighten the load.

532
Q

When working with English Language Learners, teachers should:
a. Avoid idioms and slang; involve students in hands-on activities; reference students’ prior knowledge; speak slowly.
b. Speak slowly; use monosyllabic words when possible; repeat each sentence three times; use idioms but not slang.
c. Use monosyllabic words when possible; repeat key instructions three times but not in a row; reference students’ prior knowledge; have students keep a journal of new vocabulary words.
d. Use both idioms and slang; reference students’ prior knowledge; speak at a normal rate of speed; involve students in hands-on activities.

A

A: Avoid idioms and slang; involve students in hands-on activities; reference students’ prior knowledge; speak slowly. Informal use of speech such as idioms and slang are likely to confuse ELLs. Involving students in hands-on activities such as group reading and language play makes the experience more meaningful and more immediate. New knowledge can only be absorbed by attaching it to prior knowledge so referencing what students already know is essential. Speaking slowly to English Language Learners is important since they are processing what is being said at a slower rate than a native speaker.

533
Q

Word recognition is:
a. One of the building blocks of learning.
b. Useful only to fluent readers.
c. Culturally based.
d. Especially important to English Language Learners and students with reading disabilities.

A

D: Especially important to English Language Learners and students with reading disabilities. Word recognition is the process of identifying a word’s meaning and pronunciation. While it is important to all readers, it is essential to ELLs.

534
Q

Assessing silent reading fluency can best be accomplished by:
a. Having the student summarize the material to determine how much was understood.
b. Giving a written test which covers plot, theme, character development, sequence of events, rising action, climax, falling action and outcome. A student must test at a 95% accuracy rate to be considered fluent at silent reading.
c. Giving a three minute Test of Silent Contextual Reading Fluency four times a year.
d. Silent reading fluency cannot be assessed. It is a private act between the reader and the text and does not invite critique.

A

C: Giving a three minute Test of Silent Contextual Reading Fluency four times a year. This test presents a student with a string of text in which no spaces between words appear; punctuation is also removed. The student must divide one word from another by marking where division should occur. When presented with a strand such as: Thesmalldogherdedthefluffysheepintothebarn would ideally be sectioned as The/small/dog/herded/the/fluffy/sheep/into/the/barn. The more words a student accurately separates, the higher the silent reading

535
Q

Explicit instruction includes:
a. Clarifying the goal, modeling strategies and offering explanations geared to a student’s level of understanding.
b. Determining the goal, offering strategies and asking questions designed to ascertain if understanding has been reached.
c. Reassessing the goal, developing strategies and determining if further reassessing of the goal is required.
d. Objectifying the goal, assessing strategies and offering explanations geared to a student’s level of understanding.

A

A: Clarifying the goal, modeling strategies and offering explanations geared to a student’s level of understanding. Well-organized teaching that offers simple steps and frequent references to previously learned material defines explicit instruction.

536
Q

An ORF is:
a. An Oral Reading Fluency assessment.
b. An Occasional Reading Function assessment.
c. An Oscar Reynolds Feinstein assessment.
d. An Overt Reading Failure assessment.

A

A: An ORF is an Oral Reading Fluency assessment. An ORF, also called Curriculum-Based Measurement (CBM) is a one-minute assessment in which the student reads a grade-level text aloud. The test supervisor notes errors the reader doesn’t self-correct and the number of words read correctly.

537
Q

The four types of Bilingual Special Education Instructional Delivery Models include:
a. Bilingual Support Model, Coordinated Services Model, Integrated Bilingual Special Education, Bilingual Special Education Model.
b. Bilingual Instructional Education, Coordinating Instruction, Disintegrative Support, Bilingual Special Education Model.
c. Integrated, Disintegrative, Bilingual Support, Corresponding Services.
d. Special Instructional Education, Bilingual Instruction, Bilingual Delivery, Special Support Education.

A

A: Bilingual Support Model, Coordinated Services Model, Integrated Bilingual Special Education, Bilingual Special Education Model. The Bilingual Support Model teams bilingual paraprofessionals with English-speaking special educators to assist with the IEP implementation. The bilingual assistant gives instruction in areas specified in the IEP. In the Coordinated Services Model, the team consists of an English speaking special education teacher and a bilingual educator. The Integrated Bilingual Special Education model is applied in districts with bilingual special education teachers who can give instruction in the native language, English as Second Language (ESL) training and transition assistance as the student gains proficiency. The Bilingual Special Education Model integrates all school personnel who focus on bilingual special education instruction and services. All professionals have been trained in bilingual special education.

538
Q

The Individuals with Disabilities Education Act (IDEA) requires that members of an IEP team include:
a. All teachers involved with the student, the parent(s) or guardian and the student (if appropriate).
b. The classroom teacher, a special education teacher, the parent(s) or guardian, a representative of the local education agency knowledgeable about specialized instruction, someone to interpret instructional implications, the student (if appropriate) and other people invited by the parents or the school.
c. The classroom teacher, a special education teacher, the principal or AP and the parent(s) or guardian.
d. All teachers involved with the student, the principal or AP, the parent(s) or guardian and the student (if appropriate).

A

B: The classroom teacher, a special education teacher, parents or guardian, a representative of the local education agency knowledgeable about specialized instruction, someone to interpret instructional implications, the student if appropriate and other people invited by the parents or the school. IDEA defines the IEP team as a group of people responsible for developing, reviewing and revising the Individualized Education Program for a disabled student.

539
Q

At the beginning of each month, a student reads a page or two from a book he hasn’t seen before. The resource teacher notes the total number of words in the section and the number of times the student leaves out or misreads a word. If the student reads with more than a 10% error rate, he is:
a. Reading with full comprehension.
b. Probably bored and his attention is wandering.
c. Reading at a frustration level.
d. Missing contextual clues.

A

C: Reading at a Frustration reading level. At a Frustration reading level, a student is unable to unlock meaning from a text regardless of teacher support or strategies. The reader is at this level when he has less than 90% accuracy in word recognition and less than 50% in comprehension, retelling a story is illogical or incomplete and the student cannot accurately answer questions about the text.

540
Q

A Cloze test evaluates a student’s:
a. Reading fluency
b. Understanding of context and vocabulary
c. Phonemic skills
d. Ability to apply the Alphabetic Principle to previously unknown material.

A

B: Understanding of context and vocabulary. A Cloze test presents a reader with a text in which certain words are blocked out. The reader must determine probable missing words based on context clues. In order to supply these words, the reader must already know them.

541
Q

A Kindergarten teacher is showing students the written alphabet. The teacher pronounces a phoneme and one student points to it on the alphabet chart. The teacher is presenting:
a. Letter-sound correspondence
b. Rote memorization
c. Predictive Analysis
d. Segmentation

A

A:Letter-sound correspondence. Letter-sound correspondence is the relationship between a spoken sound and the letters predictably used in English to transcribe them.

542
Q

A ninth grade special education teacher is giving students strategies for taking a Maze test. There will be several paragraphs in which some words have been blanked out. There are five possible answers for each blank. The best approach is to:
a. Read all answers; mark out any that are illogical. ‘Plug in’ the remaining words and mark out those that are grammatically incorrect or do not sound right. Think about context clues.
b. Read the first answer. If it is logical and sounds correct, select that word and move on to the next question.
c. Cover the answers and try to guess what the correct word is. Look at the five choices. Select the one closest in meaning to the word guessed.
d. Do the first and last questions then one in the middle. Look for a pattern and select the remaining words accordingly.

A

A: Read all answers; mark out any that are illogical. Next ‘plug in’ the remaining words and mark out those that are grammatically incorrect or do not sound right. Finally, think about context clues. Maze and Cloze tests are related. In a Cloze test, the reader uses context clues and familiar vocabulary to decide which words have been leftout of a text. A Maze test supplies a number of possible answers and the reader must select the correct one. Possible answers can be syntactically possible but illogical, syntactically impossible but make semantic sense and both illogical and grammatically impossible.

543
Q

A resource teacher wants to design a lesson that will help first and second graders learn sight words so all the students can read their lists. She should teach them how to:
a. Divide sight words into syllables. Considering one syllable at a time provides a sense of control and increases confidence.
b. Recognize word families. Organizing similar words allows patterns to emerge.
c. Sound out the words by vocalizing each letter. Using this approach, students will be able to sound out any sight word.
d. Memorize their lists by using techniques such as songs, mnemonic devices and other fun activities. By definition, sight words cannot be decoded but must be recognized on sight.

A

D: Memorize their lists by using techniques such as songs, mnemonic devices and other fun activities. By definition, sight words cannot be decoded but must be recognized on sight.

544
Q

Phonological awareness activities are:
a. Oral
b. Visual
c. Both A and B.
d. Semantically based.

A

A: Oral. Phonological awareness is the understanding of the sounds within a spoken word. While phonological awareness contributes to fluent reading skills, activities designed to develop an awareness of word-sounds are, by definition, oral.

545
Q

It is important to teach life skills to developmentally delayed students to prepare them for life after school. Which of the following skills sets should these students be taught?
a. Count money, plan meals, grocery shop, recognize safety concerns.
b. Count money, order delivery meals, dating skills, how to drive.
c. How to drive, style and hygiene tips, social strategies, dating skills.
d. Stock market investment, hairdressing, house painting, pet care.

A

A: Count money, plan meals, grocery shop, recognize safety concerns. These are among the most basic life skills developmentally delayed students must master. Other life skills include specific occupational skills, home maintenance, clothes selection and care, food preparation and personal hygiene.

546
Q

A special education teacher has done intervention with an eighth grade student with a reading disability. The student can now successfully use tactics to understand the meanings of unfamiliar words, knows words such as crucial, criticism and witness have multiple meanings and considers what she already knows to figure out a word’s meaning. These features of effective reading belong to which category?
a. Word recognition
b. Vocabulary
c. Content
d. Comprehension

A

A: Word recognition. Elements of word recognition include strategies to decode unfamiliar words, considering alternate word meanings to decode a text and the ability to apply prior knowledge to determine a word’s meaning.

547
Q

Emergent writers understand letters represent sounds, words begin with a sound that can be written as a letter and writing is a way one person captures an idea another person will read. Emergent writers pass through the following stages:
a. Scripting the end-sound to a word (KT=cat); leaving space between words; writing from the top left to the top right and from top to bottom of the page.
b. Scripting the end-sound to a word (KT=cat); writing from the top left to the top right and from top to bottom of the page; separating the words from one another with a space in between.
c. Leaving space between the initial letters that represent words; writing from the top left to the top right and from top to bottom of the page; scripting the final sound of each word and the initial sound (KT=cat).
d. Drawing a picture beside each of the initial sounds to represent the entire word; scripting the end-sound to a word (KT=cat); scripting the interior sounds that compose the entire word(KAT=cat)

A

A: Scripting the end-sound to a word KT=cat; leaving space between words; writing from the top left to the top right and from top to bottom of the page. Each of these steps is progressively more abstract. Scripting the end-sound to a word helps a young writer recognize words have beginnings and endings. This naturally leads to the willingness to separate words with white space so they stand as individual entities. Once this step is reached, the child realizes English, writing progresses from left to right and from the top of the page to the bottom.

548
Q

As defined by the Individuals with Disabilities Education Act (IDEA), Secondary Transition is a synchronized group of activities that are:
a. Results-oriented and include post-school activities, vocational education, employment support and adult services and considers the individual’s strengths, preferences and interests.
b. Socially structured and consider the individual’s strengths, preferences and interests and vocational requirements.
c. Designed to support vocational training, results-oriented and have a strong social component.
d. Selected by the parent(s) or guardian because the student cannot choose for himself.

A

A: Are results-oriented, includes post-school activities, vocational education, employment support, adult services and considers the individual’s strengths, preferences and interests. Additional activities that compose Secondary Transition are instruction, related services, community experiences, the development of employment and other post-school adult living objectives and, if appropriate, acquisition of daily living skills and functional vocational evaluation.

549
Q

A resource teacher can facilitate the greatest achievement in emergent writers who are scripting initial and final sounds by:
a. Suggesting they write a book to build confidence, teach sequencing, and encourage them to deeply explore ideas.
b. Asking they read their stories to other students.
c. Inviting a reporter to write about her emergent writers.
d. Inviting parents or guardians for a tea party at which the children will read their stories aloud.

A

B: Asking they read their stories to other students. Emergent writers scripting initial and final sounds will gain the most immediate and relevant satisfaction by moving around the room, reading what they’ve written to other students.

550
Q

At what point should the teacher in the above example offer the children picture books and ask them to read to her?
a. When the children are able to script initial sounds, end sounds and interior sounds. She should wait until this point to avoid frustration.
b. After the teacher has read the picture books several times, the children can ‘practice reading’ to her, while learning to handle books, turn pages, and pay attention to context clues.
c. After the children have learned the sight words.
d. From the first day of school. Picture walks help young readers understand books are arranged sequentially. Pictures provide narrative coherence and contextual clues. Holding a book and turning pages also gives young readers a familiarity with them.

A

D: From the first day of school. Picture walks give young readers the idea books are arranged sequentially. Pictures provide narrative coherence and context clues. Holding a book and turning pages gives young readers a familiarity with them.

551
Q

How can a teacher teach spelling effectively?
a. Students who have an understanding of letter-sound association do not need to be taught to spell. If they can say a word, they can spell it.
b. Students who have an understanding of letter-sound association and can identify syllables and recognize when the base word has a Latin, Greek or Indo-European ancestry don’t need to be taught to spell. They can deduce what is most likely the correct spelling using a combination of these strategies. A teacher who posts charts organizing words into their ancestor families, phonemic units and word-sound families is efficiently teaching spelling. The rest is up to the student.
c. Students who spell poorly will be at a disadvantage for the rest of their lives. It is essential students spend at least 15 minutes a day drilling spelling words until they know them forward and backward. The teacher should alternate between students individually writing a new word 25 times and the entire class chanting the words.
d. Students should be taught writing is a process. By applying spelling patterns found in word families, the spelling of many words can be deduced.

A

D: Students should be taught that writing is a process. By applying spelling patterns found in word families, the spelling of many words can be deduced.

552
Q

A special education teacher gives a struggling reader a story with key words missing: The children were hungry. They went into the ______. They found bread, peanut ______ and jelly in the cupboard. They made __________. They __ _ the sandwiches. Then they were not _______ anymore.The student is able to complete the sentences by paying attention to:
a. Syntax. Word order can provide enough hints that a reader can predict what happens next.
b. Pretext. By previewing the story, the student can determine the missing words.
c. Context. By considering other words in the story, the student can deduce the missing words.
d. Sequencing. By ordering the ideas, the student can determine the missing words.

A

C: Context. By considering the other words in the story, the student can deduce the missing words. Referring to other words when a reader encounters an unfamiliar or missing word, can often unlock meaning.

553
Q

Collaborative Strategic Reading (CSR) depends upon which two practices?
a. Cooperative learning and reading comprehension.
b. Reading and metacognition.
c. Reading comprehension and metacognition.
d. Cooperative learning and metacognition.

A

A: Cooperative learning and reading comprehension. CSR is group of four reading strategies that students with learning disabilities can use to decipher and understand texts. Small groups of studentsat various reading levels support one another by going through the strategies as they read aloud or silently. Before reading, the group previews, applying prior knowledge and prediction. Next readers target words or syllables they didn’t understand calledclunksand apply a number of strategies to decode the clunks. Third, students get the gistby determining the most important character, setting, event or idea. Finally, the students wrap it upby creating questions to discuss their understanding of the text and summarize its meaning.

554
Q

Before being assigned to a special education classroom, a student must:
a. Agree to the reassignment.
b. Have an Individualized Education Plan developed.
c. Have an Independent Education Policy developed.
d. Be seen by an educational psychologist to confirm her diagnosis.

A

B: Have an Individualized Education Plan written for her. An IEP is a requirement of law. The plan, written by a team of individuals including her classroom teacher, the special education teacher, her parent s, the studentif appropriate and other interested individuals, establishes objectives and goals and offers a time-line in which to reach them.

555
Q

When asked a question, the new student answers with as few words as possible. He prefers to draw airplanes over and over again rather than play with the other children. The classroom teacher isn’t sure how to help the child. The special education teacher suggests the teacher:
a. Leave the child alone. He is likely adjusting to the new situation and will come out of his shell soon enough.
b. Remind other children in the class to include the new student.
c. Observe the child over the course of a week or two. Draw him into conversation to determine if vocabulary is limited. Note how the child interacts with others in the class. Does he initiate conversation? If another child initiates, does he respond?
d. Refer him to the school counselor immediately. It is likely the child is suffering from serious problems at home.

A

C: Observe the child over the course of a week or two. Draw him into conversation to determine if vocabulary is limited. Note how the child interacts with others in the class. Does he initiate conversation? If another child initiates, does he respond? Once the teacher has observed, she is in a better position to offer information to the special education teacher or counselor and to determine her best course of action.

556
Q

A special education teacher feels some of his strategies aren’t effective. He asked a specialist to help him improve. The specialist suggests he:
a. Begin a journal in which he considers strategies he has used. Which seemed to work? Which didn’t, and why?
b. Meet with the specialist to discuss the teacher’s goals.
c. Permit the specialist arrive unannounced for observation. This will prevent the teacher from unconsciously over-preparing.
d. Set up a video camera and record several student sessions to review. They can effectively collaborate at that time.

A

B: Meet with the specialist to discuss the teacher’s goals. It isn’t possible to determine if strategies are effective or determine a future course unless the teacher has a firm grasp of his goals and expectations.

557
Q

An eighth grade student is able to decode many words and has a borderline/acceptable vocabulary, but his reading comprehension is quite low. He can be helped with intervention offering:
a. Strategies to increase comprehension and develop vocabulary.
b. Strategies to increase comprehension and learn to identify syntax.
c. Strategies to improve understanding of both content and context.
d. Strategies to develop vocabulary and improve understanding of both content and context.

A

A: Strategies to increase comprehension and to build vocabulary. He should receive instruction focused on just the areas in which he is exhibiting difficulty. Improved vocabulary will give him greater comprehension skills. Strategies focused on enhancing comprehension together with a stronger vocabulary will provide the greatest help.

558
Q

Research indicates oral language competency in emergent readers is essential because:
a. It enhances students’ phonemic awareness and increases vocabulary.
b. The more verbally expressive emergent readers are, the more confident they become. These students will embrace both academic and independent reading levels.
c. Strong oral language skills invite students to consider a plethora of ideas. The more they ask, the richer their background knowledge.
d. It demonstrates to students their ideas are important and worth sharing.

A

A: It enhances students’ phonemic awareness and increases vocabulary. Strength in oral language helps emergent readers because reading relies largely upon the ability to decode words with knowledge about what sounds the letters represent. A large vocabulary helps the reader recognize words whose sounds are properly decoded but whose meanings aren’t familiar. Unfamiliar words slow reading fluency.

559
Q

A teacher has shown a mentally challenged student a website that integrates music and video clips with a variety of educational games about a topic the student has shown interest in. The student is initially intimidated and fears interacting with the program might result in her breaking the computer. The teacher reassures her she cannot harm the machine and shows the girl how to manipulate the mouse and keyboard. The teacher reminds the student what she already knowsabout the subject. As the student becomes more comfortable with the mouse, she focuses on the images and sounds, at times responding to the program conversationally, telling it what she knows about dinosaurs. The teacher is using the computer along with which teaching strategy?
a. Modular instruction.
b. Scaffolding.
c. Linking.
d. Transmutation.

A

B: Scaffolding. Scaffolding is an umbrella teaching approach which offers a multitude of supports. Scaffolding includes prior knowledge, mnemonic devices, modeling, graphs, charts, graphic organizers and information needed prior to starting the lesson such as vocabulary or mathematical formulas.

560
Q

A student has been identified with a cluster of learning disabilities. She will be joining a special education classroom. She is understandably nervous about making the change to a different teacher and group of classmates. In order to help her make the transition, the child should:
a. Have a party to which her new classmates are invited along with some friends from the fifth-grade class she is leaving.
b. Prepare to begin classes with her new teacher the next day. Once the decision has been made, nothing will be gained by postponing the inevitable.
c. Be brave and understand life will be full of transitions. This is an opportunity to learn new skills that will serve her well in the future.
d. Visit the classroom, meet the teacher and her new classmates and be given the opportunity to ask questions about the change she is about to make.

A

D: Visit the classroom, meet the teacher and her new classmates and be given the opportunity to ask questions about the change she is about to make. When she is able to visualize what the classroom looks like, meet the people that will become her new educational ‘family’ and have her concerns and questions addressed, she will feel more confident about the transition.

561
Q

A student is taking a reading test in which several words have been replaced with blanks. Below each blank is a series of three possible answers. The student chooses the right answer from each set. The student is taking:
a. A Cloze test, which is a type of Maze test.
b. A Maze test, which is a type of Cloze test.
c. A multiple-choice quiz.
d. A vocabulary test incorporating a type of multiple-choice quiz.

A

B: A Maze test, which is a type of Cloze test. A Cloze test offers a text with key words blanked out and the student must determine the most likely words based upon context and his vocabulary. A Maze test offers a number of possible answers and the student must read very carefully in order to make the correct selection.

562
Q

A high school student is not a strong reader. She loses her place often and misreads key words. She doesn’t try to correct her errors, even when they make no sense. She can give only a rudimentary summary of what she read. Which type of instructional focus would be most beneficial?
a. Well-organized coaching in decoding, sight words, vocabulary and comprehension several times a week. The more systematic the lessons, the better the chance the intervention will succeed.
b. Weekly instruction on one area of reading; more, and the student will become overwhelmed and likely shut down.
c. Instruction aimed at helping her become self-motivated and disciplined in her approach to learning.
d. Strategies to help her understand the general meaning so that she can gather context clues.

A

A: Well-organized coaching in decoding, sight words, vocabulary and comprehension several times a week. The more systematic the lessons, the better the chance the intervention will succeed.

563
Q

A teacher has a student with dyscalculia who has trouble organizing addition and subtraction problems on paper. She can best help him by:
a. Encouraging memorization of number families. Committing them to memory is the only way.
b. Demonstrating a problem in different ways. Write a problem on the board: 11 -3. Gather 11 books and take 3 of them away. Draw 11 x’s on the board and erase 3.
c. Use graph paper to help him organize. Show him how to write the problems, keeping each number in a box aligned with other numbers.
d. Make a game of addition and subtraction problems. Divide the class into groups and let them compete to see which group can solve the most problems.

A

C: Use graph paper to help him organize. Show him how to write the problems, keeping each number in a box aligned with other numbers. This will help him determine which numbers are in the ones group, the tens group, the hundreds group and so on.

564
Q

A child has been losing strength in her muscles over a period of time. The loss is very gradual, but the teacher is concerned and would like the child to see a doctor. The possible diagnosis is:
a. Cerebral Palsy
b. Muscular Dystrophy
c. Muscular Sclerosis
d. Spastic Muscular and Nerve Disorder

A

B: Muscular dystrophy. There are 20 types of muscular dystrophy, a genetically inherited disease that frequently first manifests in childhood. By contrast, muscular sclerosis almost never appears in childhood. Cerebral palsy is not a deteriorating disease, as is muscular dystrophy.

565
Q

A middle school student is preparing to transition from a self-contained special education classroom to a general education classroom. This transition should be made:
a. With proper preparation. A student this age needs to acclimate socially and can best do so with the same group of students in every class.
b. At the beginning of the next school year so the student doesn’t have a stigma when joining the new group.
c. One class at a time with the special education teacher supervising academic and social progress.
d. By transitioning into classes he is most interested in because he is most likely to succeed with subjects he cares about. The confidence he gains from academic success will support him as he transitions into classes he’s less interested in.

A

C: One class at a time, with the special education teacher supervising his academic and social progress. It is important to make this transition slowly, to permit the special education teacher to remain in the student’s life as both academic and emotional support and the student to adjust to her larger classes and students she doesn’t know as well.

566
Q

The four required activities described by the Assistive Technology Act (AT ACT) of 1998 are a public awareness program, coordinating activities among state agencies, technical assistance and training and:
a. Specialized training for special education teachers and support.
b. Outreach to underrepresented religious groups, ethnicities and urban populations.
c. Outreach to underrepresented and rural populations.
d. New technologies training on a quarterly basis for special education teachers and support.

A

C: Outreach to underrepresented and rural populations. The four required activities of the AT ACT of 1998 are: a public awareness program, coordinate activities among state agencies, technical assistance and training and outreach to underrepresented and rural populations.

567
Q

Behavior problems in special education students are most effectively handled with:
a. Zero tolerance.
b. Positive Behavioral Support (PBS)
c. Acceptance and tolerance
d. Positive Behavioral Control (PBC)

A

B: Positive Behavior Support. The Individuals with Disabilities Education Act of 1997 is the recommended method of dealing with behavioral problems in children with disabilities.

568
Q

A teacher suspects one of her kindergarteners has a learning disability in math. Why would the teacher suggest intervention to the child’s concerned parents rather than assessment as the first step?
a. She wouldn’t; assessment should precede intervention.
b. She wouldn’t; kindergarteners develop new skills at radically different rates. Suggesting either intervention or assessment at this point is premature. The teacher would more likely observe the child over a three month period to note her development before including the parents about her concern.
c. Assessing a young child for learning disabilities often leads to an incorrect conclusion because a student must be taught the subject before it’s possible to assess her understanding of it. Intervention teaches the child specific skills to correct her misconceptions. If the intervention fails, assessment is the next step.
d. Assessment at this stage is unnecessary and wastes time and money. Since an assessment that resulted in a diagnosis of a learning disability would recommend intervention to correct it, it is more efficient to proceed directly to intervention.

A

C: Assessing a young child for learning disabilities often leads to an incorrect conclusion because a student must be taught the subject before it is possible to assess her understanding of it. Intervention teaches the child specific skills to correct her misconceptions. If the intervention fails, assessment is the next step. Many experts recommend such assessment should not be undertaken until a child is at least six years of age.

569
Q

A high school student struggles with applied math problems. He is given the following word math problem. He selects a. 55 hours.
A train travelled from point A to point B in 3.5 hours. The same train travelled from point B to point C in 2.75 hours. Another train left point C 1.25 hours after the first train arrived at point C. This train travelled to point D in 45 minutes. The first train returns to point B in only 2.5 hours. How long did it take to travel from point A to point D?
a. 55 hours
b. 97.60 hours
c. 8.25 hours
d. 19.75 hours
The student most likely:
a. Knows he lacks the skills to solve word problems. He arbitrarily selected the first answer without attempting to solve the problem.
b. Tried to solve the problem. He aligned all the numbers as they appeared so that 45 minutes were added with 5 in the ones column and 4 in the tens column. He also added 2.5 hours, which isn’t required to solve the problem.
c. Tried to solve the problem by estimating and chose the most likely answer.
d. Selected the correct answer.

A

B: Tried to solve the problem. He aligned all the numbers as they appeared so that 45 minutes were added with 5 in the ones column and 4 in the tens column. He also added 2.5 hours, which isn’t required to solve the problem.

570
Q

The teacher in the above example wants to give the student something he can manipulate to arrive at the correct answer. The student should be given:
a. Graph paper so he can properly align the numbers.
b. A blank page to make a visual representation of the problem.
c. A calculator.
d. A digital clock that can be manually moved forward.

A

D: A digital clock that can be manually moved forward. This student’s error is the result of two misunderstandings. The first is that 45 minutes is represented as 5 in the ones column and 4 in the tens column, but that 2.5 hours is represented with .5 in the tens column and 3 in the ones column. The second misunderstanding is the student includes the red herring 2.5 hours, which should not be in the equation. With a digital clock that can be manually moved forward, the student can begin at 0:00 and move the clock forward as he adds the numbers. The teacher can remind him 45 minutes is not 45 hours and it is irrelevant the first train returned to point B in 2.5 hours.

571
Q

The teacher knows there are several ways to arrive at the correct answer. She also knows if she explains them to the student, he will not only be able to understand where he made errors, but he might also see the relationships between various methods of solving applied math problems. The teacher should show the student how to:
a. Estimate the answer; convert all of the numbers to minutes and divide the answer by 60 to determine the number of hours and minutes; determine the percentage of an hour 45 minutes is, and write that number as .75; create a graph.
b. Estimate the answer; add the relevant numbers by removing the decimal points, then insert a decimal point in the answer; use a calculator to avoid problems in addition.
c. Estimate the answer; remove all decimal points; add all of the numbers; divide by 60 using a calculator; make a graph;
d. Estimate; create a graph; use a digital clock; add the relevant numbers together by removing the decimal points, then insert a decimal point in the answer.

A

A: Estimate the answer; convert all of the numbers to minutes and divide the answer by 60 to determine the number of hours and minutes; Determine the percentage of an hour 45 minutes is, and write that number as .75; create a graph.

572
Q

The answer in the above word problem is:
a. 8.75 hours
b. 97.60 hours
c. 8.25 hours
d. none of the above

A

C: 8.25 hours. 3.5 hours+2.75 hours=6.25 hours+1.25 hours=7.5 hours +.75 hour=8.25 hours.

573
Q

IDEA requires that students identified with learning disabilities or other special needs be educated in _________ learning environment appropriate for theirneeds.
a. The safest
b. The least restrictive
c. The most appropriate
d. The most desirable

A

B: Least restrictive. IDEA requires the least restrictive environment (LRE) appropriate to a child’s needs is the proper learning environment so children are not unnecessarily isolated from non-disabled children. The student’s IEP team is responsible for determining the LRE.

574
Q

Ella, a high school student with some mental challenges, is a verbal linguistic learner. This means she learns best using:
a. Oral and written materials.
b. Materials in both her native language and English.
c. Songs combined with movements.
d. All types of visual aids

A

A: Oral and written materials. Verbal linguistic learners love language in all its forms. They often enjoy tongue twisters, mnemonic devices, poetry, word games and crossword puzzles.

575
Q

Howard Gardner’s theory of Multiple Intelligences organizes learners into what types of intelligences?
a. Verbal linguistic, mathematical, musically attuned, visual special, body embraced, interpersonal, naturistic, existential.
b. Emphatic, recessive, aggressive, assertive, dogmatic, apologetic, determined, elusive.
c. Verbal linguistic, mathematical logical, musical, visual spatial, body kinesthetic, interpersonal, naturistic, existential.
d. Dramatic, musical, verbal, mathematical, dance-oriented, sports-oriented, scientific, socially concerned.

A

C: Verbal linguistic, mathematical logical, musical, visual spatial, body kinesthetic, interpersonal, naturalistic, existential. Harvard Professor Howard Gardner cites his theory of multiple intelligences, also called learning styles, as an answer to how teachers can most effectively reach all their students. It is especially important to recognize the learning styles of students with learning disabilities and design lessons for those students accordingly.

576
Q

Jacob, a high school student, destroyed his motorcycle in an accident on a rainy night. He did not appear to be seriously injured. A previously excellent student, after the accident he became extremely moody and defiant. His school work became spotty. While on occasion he does quite well, more often he fails to turn in homework or doesn’t write down the assignment. These behaviors are often found in:
a. All teenagers and should not be cause for alarm.
b. Emotionally troubled teens; sometimes these teens ‘act out’ by putting themselves in danger, such as riding a motorcycle in bad weather.
c. Traumatic brain injury.
d. Students learning they are responsible for their own actions. Jacob is most likely angry his motorcycle was destroyed and is expressing his reluctance to take responsibility by behaving like a child.

A

C: Traumatic brain injury. Although Jacob appears to have escaped unhurt, his dramatic change in classroom behavior suggests he may be suffering from TBI. Other symptoms of TBI are: hyperactivity, impulsivity, memory and communication problems, sexually uninhibited, improper language, failure to recognize social cues, inability to focus, and physical problems such as balance.

577
Q

Lead teaching, learning centers / learning stations, resource services, team teaching and consultation are all used in:
a. Innovative teaching
b. Strategic teaching
c. Collaborative teaching
d. Self-contained classrooms

A

C: Collaborative teaching. Classrooms with a lead teacher often include a specialized teacher to listen to the lesson then work with special needs children. Other methods are: learning centers or stations in which collaborating teachers are responsible for different areas, assigning special needs students into a resource room, team teaching and/or consultation by the special education teacher to the classroom teacher.

578
Q

A special education teacher has a child who doesn’t understand the relationship between ones, tens and hundreds. He is a Bodily Kinesthetic learner. The teacher should:
a. Draw a colorful chart and put the numbers in the appropriate columns.
b. Teach him how an abacus works.
c. Create a song and dance about the numbers families.
d. Show him the relationship using Monopoly money.

A

B: Teach him how an abacus works. An abacus gives both a visual/tactile demonstration of how numbers work and allows a child who processes information through hand/body movement to physically experience numerical relationships.

579
Q

Binh, a high school senior, is concerned his school records might contain inaccurate information. He has requested them. By law, the school must:
a. Obtain permission from his parents first.
b. Provide the records within 7 days.
c. Provide the records within 45 days.
d. The school can refuse; by law, they own the records and may share them as they see fit, regardless of requests

A

C: Provide the records within 45 days. The Family Educational Rights and Privacy Act (FERPA) is a federal law that addresses student rights regarding their records. Among FERPA regulations is the requirement that a student be given records within 45 days of making the request.

580
Q

Identifying specific skills deficient in special education math students is important so the teacher can decide how to remediate. Problems can include an inability to recall math facts, understand mathematical operations and formulas and how rules are used in solving problems or focusing on attention to details. Such students might be:
a. Able to solve math problems when they haven’t been taught an operation required to do so.
b. Unable to locate errors in their own work.
c. Able to solve math problems in another language.
d. Unable to count higher than 100.

A

B: Unable to locate errors in their own work. This is the only logical answer. Answers a, c and d do not make sense in context.

581
Q

What steps are taken to identify specific skill deficits in math?
a. Standardized assessment tests, examining areas of weakness in student work to determine patterns, teacher observations, interviews with student.
b. Standardized assessment tests, examining areas of weakness in student work to determine patterns, teacher observations, interviews with parent(s).
c. Teacher observations coupled with examining areas of weakness in student work are sufficient.
d. None of the above.

A

A: Standardized assessment tests, examining areas of weakness in student work to determine patterns, teacher observations and interviews with the student. At this point the teacher is well-prepared to plan instruction.

582
Q

A fifth-grade lead teacher and the special education teacher have scheduled a parent conference to discuss the behavior problems of the student. They anticipate the boy’s mother will be anxious and defensive as she has been at previous conferences. The best approach for the teachers to take is to:
a. Draw the parent out about issues in her own life so that she will feel reassured and trusting. Point out possible connections between the mother’s emotions about her own life and her son’s behaviors and reactions.
b. Be very firm with the mother, explain the penalties and disciplines her son can expect if the behavior continues and stress neither the parent nor the child has input regarding punishment.
c. Stress the teachers will not do anything without the parent’s approval since they do not want to face liability issues.
d. Begin by welcoming the mother and telling her about her son’s academic improvements. Stress the teachers, the mother and the child share goals for the student’s success. Explain the behavior problems and ask if the mother has any insights to share.

A

D: Begin by welcoming the mother and discussing her son’s academic improvements. Stress that the teachers, the mother and the child share goals for the student’s success. Explain the behavior problems and ask if the mother has insights to share. It’s important to keep communication open.

583
Q

At the beginning of the week, a special education teacher asked a group of students to generate a list of verbs that make visual or sound pictures. She suggests students think of verbs that mean ways of walking, talking, eating, sitting and playing. The students spend the remainder of the week compiling the list. They notice interesting verbs as they read books, remark on less common verbs they hear in conversation or on television and locate interesting verbs in signs, magazines and other printed materials. One child begins to draw pictures to illustrate some of the verbs. Two children collaborate to create a play in which they demonstrate some of the verbs in a dance. A boy writes a song incorporating the list of verbs. The project is extremely successful. At the end of the week the students have created the following list:
TIPTOE, SCOOT, MUMBLE, MUNCH, LEAP, SPIN, DIVE, POUNCE, GLIDE, SLITHER, MOAN, WHISPER, GRUMBLE, NIBBLE, SHRILL, HOLLER, PERCH, LEAN, STOMP, MARCH, GIGGLE, HOP, STRUT, SLOUCH, GULP, HOWL, WHINE, SLURP, CROUCH, DRIBBLE, DROOL, HOOT, YELP, YOWL, GROWL, WHISTLE, SHRIEK, SNICKER, INSULT, COMPLIMENT, PLEAD, BARK, WIGGLE, TWIST, SLINK, TODDLE, TRUDGE, WANDER, STROLL.
The teacher’s goal is to:
a. Enhance students’ understanding of theme by encouraging them to make connections between categories of verbs.
b. Enhance students’ vocabulary by encouraging them to find examples in the world around them.
c. Enhance students’ understanding of context by encouraging them to explore verbs for contextual clues.
d. Enhance students’ sense of curiosity by directing their attention to a number of different resources they may not have considered.

A

B: Enhance students’ vocabulary by encouraging them to find examples in the world around them. Often children have richer vocabularies than they realize. This project simultaneously encourages students to remember words they already know and to learn other words with similar meanings.

584
Q

In the previous example, how could the teacher extend the lesson and apply it across the curriculum?
a. Create a Word Wall with the words the students collected.
b. Have students work on a class dictionary, putting the words in alphabetical order and explaining what they mean.
c. Ask students to create a chart noting which verbs have 1, 2 or 3 syllables, which verbs contain double letters, which verbs are also nouns and which verbs have common word-endings.
d. All of the above.

A

D: All of the above. There is often a multitude of ways a teacher can apply skills and information learned in one lesson to other subjects. In this case, vocabulary building is enhanced with a word wall; logic and reasoning skills are developed by putting the words into alphabetical order then
carefully considering how to define them; and both math skills and word recognition ability are improved by creating a chart demonstrating a variety of ways one can categorize a list of words.

585
Q

A classroom teacher has a student with learning disabilities that affect her ability to do math. The teacher consults with the special education teacher and decides she will modify the work the child is given by reducing the number of problems, let her have extra time to finish, and provide her with a multiplication chart. The teacher is:
a. Giving the student an unfair advantage. Letting her have extra time should be sufficient.
b. Giving the student an unfair advantage. Providing a multiplication chart should be sufficient. With that, she should get her work done on time.
c. Making appropriate modifications. Each child is different. In this case, she consulted with the special education teacher and concluded the child needs multiple supports.
d. Modifying the student’s work because it makes it easier on the teacher. There is less to explain and less to grade.

A

C: Making appropriate modifications. Each child is different. In this case, she has consulted with the special education teacher and concluded the child needs multiple supports.

586
Q

Explain the philosophy of inclusion.
a. All children should be included in decisions affecting their education.
b. Children with special needs are as much a part of the educational community as any other child and necessary services that allow these students to participate in the learning community should be provided.
c. Parents are part of a child’s learning community and should be included in academic decisions.
d. All teachers and support persons, including Para pros, translators and other assistants, should be allowed to participate in academic decisions

A

B: Children with special needs are as much a part of the educational community as any other child and necessary services that allow these students to participate in the learning community should be provided.

587
Q

The ADA is:
a. The Americans with Disabilities Act.
b. The Anti-Discrimination Act.
c. The American Diabetes Association.
d. The Alternatives to Discrimination Act.

A

A: The Americans with Disabilities Act. The ADA is a federal act prohibiting discrimination based on disability in the areas of employment, state and local government, public accommodations, commercial facilities, transportation and telecommunications.

588
Q

A teacher working with students who have math disabilities has had success with a variety of multi-sensory techniques including:
a. Estimating, converting fractions, multiplication families, graphic organizers.
b. Graphic organizers, math textbooks, multi-step problems, converting fractions.
c. Memorizing tables, drawing graphs, converting fractions, charting information.
d. Power point presentations that include music, manipulatives, graphic organizers, clapping games.

A

D: Power point presentations that include music, manipulatives, graphic organizers and clapping games. Multi-sensory techniques include visual, audio, tactile and kinesthetic approaches to teaching.

589
Q

The development of an IEP is a(n) _____________ process.
a. Indirect.
b. Collaborative.
c. Mathematical.
d. Single.

A

B: Collaborative. The creation of an Individualized Education Plan (IEP) involves classroom and special education teachers, family members, the student (if appropriate) and other interested parties who collaborate in the student’s best interests.

590
Q

A middle school Language Arts teacher begins each class with 10 minutes of journal writing. Students are free to write about whatever they choose. She reminds them this is the perfect place to react to something they’ve read, write about a problem and try to think of solutions, track a project they’ve undertaken and otherwise interact honestly with themselves. The teacher should periodically:
a. Collect the journals and select an entry to edit; this will show the student how his writing can improve.
b. Suggest new and innovative ways students can use their journals, including automatic writing, found poetry, lists, and collages.
c. Collect and review the journals to identify students at risk for drugs, alcohol or sexual abuse.
d. Say nothing about the journals during the school year. They are intensely private and discussing them in any way with the students violates trust.

A

B: Suggest new and innovative ways students can use their journals including automatic writing, found poetry, lists and collages. While journals are intensely personal and should never be read without the student’s permission, teachers can certainly inspire students to use the journals to explore their ideas in innovative ways.

591
Q

A high school student has been diagnosed with ODD. Some of the manifestations of the diagnosis are:
a. Obsessive and compulsive activities such as hand washing, counting and ritualistic behaviors.
b. He is self-occupied, depressed and disorganized. He keeps to himself, is considered odd by his classmates and could be suicidal.
c. The student is overly occupied with others, defending them from imagined slights and determined they recognize his concern as real rather than psychotic.
d. He goes out of his way to annoy others, is defiant and goes into childish rages in which he blames others.

A

D: He goes out of his way to annoy others, is defiant and goes into childish rages in which he blames others. This student has been diagnosed with Oppositional Defiant Disorder, a psychiatric disorder characterized by noncompliance, tantrums, extremely irritating conduct, refusal to follow rules, argumentative behavior and blaming others.

592
Q

A teacher is introducing a new subject to her special education student. She reminds the student what she knows about the subject, offers a graphic organizer with which she can organize her learning, teaches key vocabulary and models an activity the student will undertake in her study. The teacher is providing:
a. Building Blocks.
b. Strategic Framing.
c. Multiple Learning Styles Techniques.
d. Scaffolding.

A

D: Scaffolding is an umbrella teaching approach that offers a learner a multitude of supports and encourages her to reach out in many directions to enhance learning. Elements of scaffolding include prior knowledge, mnemonic devices, modeling, graphs, charts, graphic organizers and information the student will need prior to undertaking the lesson, such as vocabulary or mathematical formulas.

593
Q

The Family Educational Rights and Privacy Act (FERPA) is a federal law that addresses student rights regarding their records. Among the rights the law protects are the right to:
a. Obtain requested records within 45 days; request amendment of inaccurate information or information that violates the student’s privacy; be notified before personal information is shared with third parties; file a complaint with the U.S. Dept. of Education should the school fail to fulfill these requests.
b. Obtain records within 45 days and make amendments at the request of parents only.
c. Obtain records within 45 days and make amendments at the request of both parents and student. Neither parent(s) nor students can obtain or amend documents alone.
d. Be notified of all requests for personal information by third parties.

A

A: Obtain requested records within 45 days; request amendment of inaccurate information or information that violates the student’s privacy; be notified before personal information is shared with third parties; file a complaint with the U.S. Department of Education should the school fail to fulfill these requests. FERPA is a federal law that addresses student rights regarding records.

594
Q

A criterion-referenced test would be most useful in assessing which of the following capabilities?

A

How well a student masters specific learning objectives

595
Q

which of the following types of informal assessment is most appropriate for gathering information on the frequency with which specific student behaviors occur?

A

anecdotal record

596
Q

If a student is having a epileptic seizures, what is the first step should be taken when a student is experiencing an epileptic seizure?

A

cushion the head to protect the brain from trauma when the head strikes against the floor

597
Q

when the progress of a student is being monitored which of the following is an advantage of curriculum-based measurement ( cbm ) over norm referenced tests?

A

CBMs can be used at more frequent intervals than norm-referenced tests

598
Q

Leon goes to private school and struggles in class. What steps would he need to take in order to be evaluated from the public school that he used to attend?

A

the same as for any other special education referral

599
Q

9 year old student diagnosed with emotional disturbance transferred schools. difficulty relating to peers and adults, demonstrating quiet, withdrawn behavior and rarely making eye contact. They conducted a (ARD) Admission Review Dismissal designs behavioral interventions for the student it is most critical to…

A

Focus first on what the student is most motivated to change

600
Q

Lou’s comprehensive cognitive ability was calculated to be 93 SS, with a 90% confidence interval of 89 to 98. Which of the following is a true statement?

A

the students cognitive ability score will fall between 89-98, 90 times out of 100

601
Q

which of the following is the most appropriate use of a norm referenced test?

A

to compare a students performance to that of age=level or grade-level peers

602
Q

behavior sampling would most likely be used by a special education teacher to…

A

collect data about the duration of frequency of a particular behavior exhibited by a student

603
Q

an 11 year old took a all content standardized test in 5th grade. He scored a overall score of 6.5. which of the following is an accurate interpretation of this information?

A

Since he has mastered content typically covered through the 5th month of sixth grade, he should be considered to be performing at sixth grade level.

604
Q

Derek a student with intellectual disability just joined a teacher class and compared with his classmates Derek is less likely to

A

develop leaning stratigies without teacher intervention

605
Q

10th grader diagnosed with speech and language disorder does not participate in classroom discussion and teacher ask SPED teacher what she should do to get the student more involved

A

she needs to increase the wait time for student responses

606
Q

which of the following is the best way to ensure that a student with an autism spectrum disorder (ASD) feels comfortable in the school environment?

A

Providing a predictable school routine

607
Q

Second grade student who has been diagnosed as having attention-deficit/hyperactivity disorder (ADHD) which of the following is assign of ADHD that the student is most likely to display?

A

Forgets to turn in homework on a daily basis

608
Q

which of the following is the most common relative strength of students with a learning disability and students with attention-deficit/hyperactivity disorder (ADHD)?

A

ability to attend to more then one idea at a time

609
Q

10 grader with a learning disability is in an inclusion class struggles to pass her history class and cannot remember information from the lecture. what is the best strategy to help her be successful on the test?

A

modeling strategies for taking notes to use as a reference when studying

610
Q

6th grader with mild intellectual disability has severe outbursts throughout the day. which of the following actions is most appropriate for the teacher to address the behavior issue?

A

Conducting a functional behavioral assessment (FBA)

611
Q

which of the following accommodations is best for a teacher to make to help 6th grader with (ADHD) increase on task behavior and perform well on an assessment?

A

Limiting the number of test questions on each page

612
Q

a special education teacher will be meeting weekly w middle school students in a social skills improvement group. which of the following is the most appropriate way for the teacher to help the students increase their self-advocacy?

A

having students set personal goals and track their progress

613
Q

Ms. Chang begins to reward maya for the times she behaves well in class. the teacher approach can best be described as an example of which of the following?

A

operant conditioning

614
Q

5th grader in an inclusion classroom he will be provided a modified version of the state mandated assessment for all 3 content areas. His IEP goals should

A

show evidence of adapted content requirements in all three subject areas

615
Q

Preschooler with autism spectrum disorder needs help adjusting to changes and transitions during the school day. to best help him what is the best way to provide him with which of the following

A

a positive reinforce each time he completes a transition successfully

616
Q

Ethan with dysgraphia had a (ARD) committee meeting his teacher explains explains that the quality of his written answers is far below that of his oral answers. which of the following accommodations will best meet Ethan’s need?

A

using an augmentative and alternative communication device

617
Q

a student with autism spectrum disorder in a co-taught class. what is the most appropriate way for teacher to provide scaffolding for Matthew during instruction?

A

placing Matthew in a small group setting supervised by the special education teacher

618
Q

student with limited English proficiency struggles with receptive and productive oral language and reading comprehension skills. what is the best precautions to take in the ARD to determine if he needs special education services?

A

assessing the student formally and informally in both the second language and primary language

619
Q

Niki is a third grader with specific learning disability in reading fluency. She reads on a first grade level but understands 3rd grade read aloud. what’s the best way to help her increase her reading levels with certain assignments

A

pre-teaching academic vocab found in the texts of the reading assignments

620
Q

Which of the following is the most appropriate reason for teaching students who have learning disabilities in the area of mathematics reasoning to use a mnemonic-based strategy for problem solving

A

Having a strategy to follow will decrease the possibility for confusion during calculations

621
Q

In order to develop a hypothesis about why a student displays a certain type of behavior in a particular setting, the student must be observed directly and which of the following information recorded

A

Setting, time, behavior

622
Q

Which of the following types of assistive technology would be most appropriate for a student who has writing difficulties due to fine-motor-skill delays

A

Speech-recognition software

623
Q

A student of average ability who is receiving special education services is diagnosed with attention deficit/hyperactivity disorder (ADHD). Which of the following is a classroom management technique that will best support the needs of the student

A

Establishing clear classroom rules and consistent routines

624
Q

They particularly struggle with acquisition of content-area vocabulary. Which of the following learning strategies is most appropriate for Ms. Bettor’s students?

A

Link the key words to familiar, acoustically similar words and provide a visual cue associating the word to its definition

625
Q

A ninth-grade student with dyslexia has an Individual Education Program (IEP) goal to improve his independent writing skills. Which of the following is the most appropriate accommodation the IEP team can recommend for the student

A

A peer tutor to work one-on-one with the student in developing his ideas and correcting his mechanics and spelling

626
Q

Mary, a student who read above grade level and is hard of hearing, chose to write about the turtle’s shell pattern and eating habits. Which of the following modifications would be most appropriate for Mary?

A

Giving her the assignment in writing

627
Q

A special education teacher is considering transition goals for a student who will turn 16 by the end of the year to ensure they contain appropriate information. Which of the following must be included in the individual transition goals?

A

Services and post-school outcomes for the student

628
Q

Rachel, a student with a visual impairment, Rachel’s parents ask that the school purchase an electronic reader that has large font and text-to-speech capabilities. Which of the following is the most appropriate response by the ARD committee to provide to the parents

A

Considering the parents’ recommendation and deciding as a team the most appropriate technology Rachel should use to have success in the classroom

629
Q

Which of the following general instructional approaches would most significantly help an English-language learner with a receptive language disorder grasp concepts taught in the classroom?

A

Providing new information in context

630
Q

To best facilitate successful collaboration between general education and special education teachers within a co-teaching setting, teachers will primarily need..

A

Additional planning time

631
Q

A kindergarten student who experienced prenatal exposure to drugs and who exhibits self-regulatory problems is likely to be best served in a classroom which..

A

Routines, activities, and caretakers are consistent and predictable

632
Q
A
632
Q

Sonia w/ average intelligence cerebral palsy: When setting up the classroom, Sonia’s teacher should consider which of the following to provide Sonia with the necessary accessibility in which she requires?

A

Making aisles between desks and tables wide and uncluttered

633
Q

Sonia w/ average intelligence cerebral palsy: Which of the following related service providers can best help perform everyday activities such as brushing her teeth and tying her shoes?

A

Occupational therapist

634
Q

Sonia w/ average intelligence cerebral palsy: Which of the following assistive technology devices is most appropriate for Sonia to use in the classroom?

A

A voice-to-text software program

635
Q

Samuel diagnosed w a learning disability (LD) is at his best when learning from lectures or classroom discussions but struggles with printed materials. What assistive device should be considered when developing his IEP?

A

Augmentative communication software

636
Q

A 8th grade student has a degenerative medical condition affecting his limbs but not his fingers. Which of the following accommodations would be most appropriate to support this student’s independence in the task of note taking?

A

A portable word processor or computer

637
Q

Which of the following would be the most appropriate language arts goal for a 17-year-old who receives instruction within a life skills curriculum

A

Finding a doctor’s telephone number in a telephone book

638
Q

Challenging behaviors that are not socially motivated do not require intervention from others in the environment. These behaviors may serve the function of obtaining or escaping internal stimulation. Which of the following is a non-socially motivated challenging behavior

A

Stealing from a classmate

639
Q

A special education teacher is working with a ninth grader with a learning disability on study strategies to use in content-area classes. The teacher could best promote the student’s application of such strategies by..

A

Giving her a chart that associates strategies with specific types of assignments

640
Q

Which of the following types of skills are most useful to the greatest number of students who are receiving special education services and are explicitly taught to students to help them function independently in the community

A

Problem-solving skills

641
Q

students are doing a project where they interview people: One student in the class has speech that is very difficult to understand. The most appropriate way to include this student in the interviewing process would be to..

A

Help him create a written version of his questions for the interviewee to refer to during the interview

642
Q

students are doing a project where they interview people: How does this activity best promote the functional living skills of students who have special needs?

A

It involves them in meaningful interactions with people in a community setting

643
Q

student w behavior disorder has been showing aggressive behaviors and in the ARD they want to conduct a functional behavioral assessment. Which of the following procedures would most likely be included in this assessment?

A

Conducting interview to determine the precise nature and timing of the student’s problem behavior

644
Q

Which of the following statements by a special educator best exemplifies the use of positive behavior techniques to promote a student’s self-management skills?

A

“Logan, you are sitting so quietly writing the sentences as I assigned.”

645
Q

high school senior meets with sped teacher, job coach and job supervisor after getting into a loud argument at work. Which of the following would be the best strategy for using this incident to foster Lisa’s social growth?

A

Asking Lisa to describe what happened and guiding her to analyze how and why it occurred

646
Q

An early intervention teacher can best support the transition process of children and their families who are moving from infant/toddler programs to preschool by..

A

Ensuring that the new program will continue to follow guidelines for developmentally appropriate instructional practices

647
Q

When planning social skills instruction from culturally and/or linguistically diverse background, which of the following should be a special educator’s most important consideration

A

The similarities and differences among students’ cultural backgrounds

648
Q

Which of the following is a basic premise of an effective functional skills curriculum for secondary school students with intellectual disabilities?

A

Students should learn job-related and life skills specific to many different types of communities (e.g., rural, urban)

649
Q

A school’s special education staff has begun collaborating with local employers to develop and institute a program of community-based vocational training. Students will spend several weeks at each site receiving training in a variety of jobs. Such a program would be especially useful for..

A

Helping students develop a high degree of proficiency at a wide range of jobs

650
Q

Students in a second-grade class will be participating in a number of art activities over the course of the year. Which of the following would be the best way to adapt these activities for 8-year-old Susan, a student with severe delays in fine motor development?

A

Ensuring that a wide range of art materials, including some that the student can easily manipulate, are always available

651
Q

7 year old 2nd grader has Oppositional defiant disorder (ODD). However he is performing at grade level for all his classes and his gen ed teacher sees that he is struggling w a poetry writing unit. Which of the following modifications would be most effective in supporting him in this area?

A

Providing explicit, scaffolded instruction with opportunities for him to make his own choices

652
Q

Alternate keyboards, optical head pointers, and voice recognition software would be most helpful to students who have difficulty

A

Physically accessing standard technology

653
Q

Postgraduation transition planning for a student is likely to be most successful if planning is based primarily on which of the following?

A

The student’s personal goals and priorities

654
Q

a 10th grade student has down syndrome and hopes to work w the animals at the zoo and live in a apartment w her cousin after graduation. Which of the following will be the most appropriate transition statement on her individualized education program (IEP)?

A

Jana will participate in a curriculum that addresses functional daily living and work skills

655
Q

In preparing for a classroom with general and special education students, the general education teacher may have to make adjustments for students with disabilities. Which of the following actions should the teacher take?

A

The general education teacher should review each student’s Individualized Education Program (IEP) with the special education teacher

656
Q
A
656
Q

Of the following, the most effective means for creating consistency from preschool through kindergarten is to provide students..

A

A developmentally appropriate program in both preschool and kindergarten that is responsive to individual differences

657
Q

a fourth-grade student with a Specific Learning Disability (SLD), is in an inclusion classroom. She refuses to work in collaborative groups and when the teacher asks her why she is not willing to participate, Karen responds that she feels stupid. Which of the following would be most appropriate for the inclusion teacher to implement prior to collaborative activities?

A

Discussing the topics with Karen before beginning the activity

658
Q

Which of the following learning goals would most likely be addressed in functional academic skills instruction for a high school student with multiple disabilities?

A

Follow directions for recipes

659
Q
A
660
Q

In which of the following stages of literacy development is phonemic awareness included?

A. Word pattern readers and spellers

B. Syllables and Affixes

C. Emergent Readers and Spellers

D. Alphabetic Readers and Spellers

A

C. Emergent readers and spellers

This is the first stage and includes phonemic and phonological awareness